Jump to content

Wikipedia:Reference desk/Science

From Wikipedia, the free encyclopedia

This is an old revision of this page, as edited by 78.146.162.232 (talk) at 11:56, 22 May 2009 (→‎Lilac bushes). The present address (URL) is a permanent link to this revision, which may differ significantly from the current revision.

Welcome to the science section
of the Wikipedia reference desk.
Select a section:
Want a faster answer?

Main page: Help searching Wikipedia

   

How can I get my question answered?

  • Select the section of the desk that best fits the general topic of your question (see the navigation column to the right).
  • Post your question to only one section, providing a short header that gives the topic of your question.
  • Type '~~~~' (that is, four tilde characters) at the end – this signs and dates your contribution so we know who wrote what and when.
  • Don't post personal contact information – it will be removed. Any answers will be provided here.
  • Please be as specific as possible, and include all relevant context – the usefulness of answers may depend on the context.
  • Note:
    • We don't answer (and may remove) questions that require medical diagnosis or legal advice.
    • We don't answer requests for opinions, predictions or debate.
    • We don't do your homework for you, though we'll help you past the stuck point.
    • We don't conduct original research or provide a free source of ideas, but we'll help you find information you need.



How do I answer a question?

Main page: Wikipedia:Reference desk/Guidelines

  • The best answers address the question directly, and back up facts with wikilinks and links to sources. Do not edit others' comments and do not give any medical or legal advice.
See also:



May 13

Blood in near vacuum

The article blood says that blood in a vacuum looses its oxygen and turns blue. How long would this take? Would it happen before or after drying, as a matter of fact, would blood dry at all? Thank you. 190.17.201.142 (talk) 01:54, 13 May 2009 (UTC)[reply]

The lack of a source for that statement in the article makes tracking down related information. The Vacuum article has some useful information and lots of references. I also found this article that has some more in-depth information along the same lines. One of the key points is that the body contains the blood, so it does not boil and you do not freeze (at least not right away). With no air in the lungs, the blood becomes oxygen depleted fairly quickly, causing loss of consciousness in about 15 seconds, but you still may have a couple of minutes where recovery is still possible. -- Tcncv (talk) 02:41, 13 May 2009 (UTC)[reply]
I think the pertinent article is probably oxygen dissociation curve. In a vacuum, the air pressure is 0, making the partial pressure of oxygen also 0, so when the vacuum occurs, you drop to the left end of the curve, oxygen dissociates from hemoglobin, dissolved oxygen in the blood proper will also be lost. All of this would occur before any drying would. Blood would turn "blue" only in the sense that venous blood is said to be blue: it's the color of hemoglobin without an associated oxygen molecule. - Nunh-huh 02:50, 13 May 2009 (UTC)[reply]
Our vein and cyanosis articles explain that oxygenated blood is red, and deoxygenated blood is darker-red. The blue color comes from Rayleigh scattering by tissues. Notice that the sentence from the blood article that you referenced has a "citation needed" after it. Ginogrz (talk) 04:06, 13 May 2009 (UTC)[reply]
"Citation needed" is there because I just added it. -- Tcncv (talk) 04:55, 13 May 2009 (UTC)[reply]
Really interesting. So what would happen if I shed blood on a surface in an atmosphere at, say, 10 percent pressure that of the earth surface, for example in protected conditions at 15km altitude ? 190.17.201.142 (talk) 05:23, 13 May 2009 (UTC)[reply]

Runway threshold markings

This is not really a science question, but I figure the aviation folks are more likely to see this here. While touring the taking a tour via Google Earth, I happened to notice differences in the threshold markings on runways at various major airports. Most have twelve bars (six on either side of the centerline, while other very similar runways have eight (four on either side). For example, of Atlanta's four major runways, only 8L/26R has the twelve bar markings. However, this diagram shows that the runway 9L/27R is the same width and longer, but it only has eight bars. Another intersting case is Seattle-Tacoma International Airport here, where both runways have eight bars on the south end, and twelve on the north. Can anyone clarify this? -- Tcncv (talk) 01:56, 13 May 2009 (UTC)[reply]

There are standards - but there are SEVERAL standards - and there doesn't always seem to be a lot of logic as to which standard each airport uses. Runway#Runway_markings explains this to some degree. SteveBaker (talk) 02:42, 13 May 2009 (UTC)[reply]
Thanks. I saw that section, but it does not go into enough detail. I am convinced there is a standard out there somewhere that precisely states when to use eight stripes and when to use twelve. Aviation is two heavily regulated for airports such as Atlanta and Seattle to have mixed markings without a well defined reason. -- Tcncv (talk) 02:56, 13 May 2009 (UTC)[reply]
Apparently there are two configurations for runway threshold markings mentioned by the FAA (See section 2F here). They consist of either 8 parallel bars, or various numbers of parallel bars to indicate runway width (e.g. 8 = 100 ft, 12 = 150 ft). Unfortunately, my quick read didn't reveal why there are two configurations, or when each can be used. And as for why the two systems are mixed at SeaTac, I have no clue. -- Flyguy649 talk 05:04, 13 May 2009 (UTC)[reply]
A bit more. According to this, the 8-bars-no-matter-what configuration was to become invalid as of Jan 1, 2008. So it's possible that the google images are from a time when the airports hadn't yet finished changing over the markings. Just a thought. -- Flyguy649 talk 05:09, 13 May 2009 (UTC)[reply]
Thanks for the excellent info. The transition from configuration A to configuration B would certainly explain things. -- Tcncv (talk) 05:47, 13 May 2009 (UTC)[reply]
As further evidence, looking at the third runway construction in Seattle, it is apparent that the image of the south end of the airport is older that the image at the north end. And in Atlanta, the twelve stripe thresholds appear to be newer pain jobs than the eight stripe thresholds, judging from the tire marks. -- Tcncv (talk) 06:14, 13 May 2009 (UTC)[reply]
It may be that there is finally some effort to reconcile the multiple standards into a single one - but repainting those stripes is not a trivial matter. They look small from the air - but each one of those 'piano keys' is as big as a medium-sized parking lot - burning off and repainting that much paint isn't easy. Closing the runway in order to do the work is also difficult. Other aspects of the differing standards are even tougher to fix - lighting configurations, for example, can involve lights positioned out off the end of the runway on land that the airport authority may not even own...and a typical airport has many hundreds of lights. Some of the complications arise from updated requirements that 'grandfather in' the old standards for these kinds of reasons - others come about from military airfields being repurposed or shared for civilian needs - resulting in conflicting military versus civilian/FAA requirements, etc. It really is something of a mess. SteveBaker (talk) 14:00, 13 May 2009 (UTC)[reply]

Potential energy

We're given three charges, Q1, Q2, and Q3, which are arranged in an equilateral triangle and we're asked to find the total potential energy. Now normally I would have said that U = U1 + U2 + U3 = (k/d)(Q1Q2 + Q1Q3 + Q2Q1 + Q2Q3 + Q3Q1 + Q3Q2). But the answer says that U = (k/d)(Q1Q2 + Q1Q3 + Q2Q3). Why is this? —Preceding unsigned comment added by 65.92.6.148 (talk) 04:00, 13 May 2009 (UTC)[reply]

It appears that you are double-counting some energy. (Note that you're off by a factor of two). This is because the energy of two particles Q1 and Q2 is only counted once; if you also calculate it for Q2 and Q1, you are considering the same system twice. Nimur (talk) 04:34, 13 May 2009 (UTC)[reply]
(ec - partial duplication of prior answer) I'm no expert, but one way to check for consistency between the three change potential energy equation and the two charge potential energy equation is to set one of the charges, say Q3 to zero. When compared with the equation in Potential energy#Electrostatic potential energy, it appears that the textbook answer is correct. In short, don't double count Q1Q2 along with Q2Q1. That effectively is already built into the constant k. -- Tcncv (talk) 04:43, 13 May 2009 (UTC)[reply]
Put simply, the standard potential formula already counts the potential for both of the charges involved; you do not need to calculate and and and then add those three. (Those three quantities are not nonsensical, but they apply only if you consider establishing two of the charges and then bringing in the third. They can't all three be that third charge!) --Tardis (talk) 04:51, 13 May 2009 (UTC)[reply]
Why is that so? Is there some sort of derevation for this? Because the formula kq1q2/r comes from assuming one charge stays put, so why would it also account for the work that's done on it as well? —Preceding unsigned comment added by 65.92.6.148 (talk) 00:35, 14 May 2009 (UTC)[reply]
I managed to derive that for a constant force (such as when we assume gravity to be constant), the equation for U doesn't change whether we assume one of the bodies (eg the earth) says at rest or not. I just said that ΔU=-W_T=-(W_1,2 + W_2,1) = -(F•s_1 + F•s_2)=FΔh_1 - FΔh_2=F(Δh_1-Δh_2)=mgΔd, where W_1,2 is the work done on body 1 by body 2, and • is the dot product. Can someone help me with the derivation for a non-constant force? —Preceding unsigned comment added by 65.92.6.148 (talk) 04:00, 13 May 2009 (UTC)[reply]
It's really just a matter of definition. We could go around calculating the potential energies of each charge separately, but the contribution to A's potential from B is always identical to B's from A, so we might as well call their sum "the potential of A and B" and be done with it. Clearly we cannot then also consider "the potential of B and A" because we've already counted both those interactions. As for deriving it, it should be sufficient to apply Newton's Third Law (and its corollary that the forces are radial) and note that (where we define the scalar F as positive when repulsive). --Tardis (talk) 18:03, 14 May 2009 (UTC)[reply]

To answer this question, we need to look into the fundamentals... what is potential energy of a system ? It is the energy required to assemble the system. Or the energy you would get if you blew up the system. This is basically the "potential" energy that the system has. So first coming for a one charge system. What is its potential energy ? Assuming there is nothing else in the surroundings, it is zero. You didn't have to do any work to put it there. Note that we don't take into account the energy to create a charge... charges are simply given to us and we just put them around. This is wise policy, as the energy of a point charge, is in fact, infinite. So for a system of two charges, we first bring in the first charge. This wouldn't take any work. Now, for the second charge. We bring it in from far away. How far away ? You could say from your house, or from a known landmark like the Eiffel tower or the sun, but the standard practice is to bring it in from infinity. But it is repelled (or attracted) by the first charge.So we integrate to get the work done, which is how we get your famous formula. Now for say three charges, we put the first charge in. Again no work done here. Next we put the second charge in. You know the formula. Now for the third charge, it is repelled (or attracted) by both the first and the second charges. So we plug in the formula for both of them, and add them all up, which is how you get your textbook formula. So once you understand why you get a certain formula, it is easy to build skyscrapers on you strong basement... Rkr1991 (talk) 08:00, 14 May 2009 (UTC)[reply]

Makes sense. But out of curiosity, how would you re-derive the equation for potential energy while taking into account both charges moving (I guess it's more of a math question than anything....). —Preceding unsigned comment added by 65.92.6.148 (talk) 08:40, 14 May 2009 (UTC)[reply]

Now that's interesting... The whole time we were talking about Electrostatics, where we assume everything to be in rest or in very slow motion. Now the question you are asking belongs to the domain of Electrodynamics. Now i need to talk a bit more in the basics, a point which i evidently forgot to mention. I had told that you integrate the differential work from infinity to the required position. The question is, along what path do i integrate ? That is, while getting from infinity to the point, what curve does to charge follow ? The answer is it doesn't mater. You can integrate along any curve want, it doesn't change the answer. This is because the electrostatic force, given by coulomb's law, is a term we call conservative [[1]]. Its line integral is independent of path, and we get the same answer always. Now in the realm of electrodynamics, coulomb's law doesn't hold good. Further, in addition to electric forces, there are also magnetic forces. Both of these are now non conservative. So, our answer would now depend on the path we choose. So, if we get different energies if the charges are all blown up in different directions, there isn't really much meaning in the term potential energy anymore... So in electrodynamics, there is no such term Potential energy. However, there exists a four vector potential term... but we don't really care about that do we ? Also, don't forget to sign you posts, by placing four '~' marks at the end... Rkr1991 (talk) 15:09, 14 May 2009 (UTC)[reply]

MICROPROCESSOR

Deleted. Poster has posted the same homework question on RD/C repeatedly. Tempshill (talk) 15:51, 13 May 2009 (UTC)[reply]

Hypnotic state of mind while watching t.v.

(kids' eyes glass-over, people walk by unnoticed, because of the fast 'flickering' of the tv)

I've heard this several times, but searching for it online, I can't find anything on it.

67.236.121.225 (talk) 06:32, 13 May 2009 (UTC)[reply]

Photosensitive_epilepsy#Symptoms has a subsection about Television, traditionally the most common source of seizures.Cuddlyable3 (talk) 07:49, 13 May 2009 (UTC)[reply]
Immersion (virtual reality)?--Lenticel (talk) 08:03, 13 May 2009 (UTC)[reply]
Hyperfocus is marked as needing work, but the term may help with googling studies. 71.236.24.129 (talk) 08:13, 13 May 2009 (UTC)[reply]
You can get the same effect reading a book, writing or doing other tasks. Any time you "zone out," you're so focused on the task at hand that other stimuli are ignored. — The Hand That Feeds You:Bite 13:21, 13 May 2009 (UTC)[reply]
Also see tetris effect, highway hypnosis and automaticity. Lanfear's Bane | t 13:54, 13 May 2009 (UTC)[reply]

Sound Energy Converter

Is there and if there isn't, is it possible to create a device converting sound into kinetic, electrical, or electromagnetic energy?The Successor of Physics 12:56, 13 May 2009 (UTC)[reply]

See Microphone. -- Coneslayer (talk) 13:12, 13 May 2009 (UTC)[reply]
While we're at it, see geophone and hydrophone and piezoelectric crystal as well. Nimur (talk) 13:21, 13 May 2009 (UTC)[reply]
In the 1880's Thomas Edison invented a device which converted sound waves into vibration, and then into rotary motion, and it could power a drill. He noted that many a mother-in-law's voice had been thought to be capable of drilling holes. Edison (talk) 17:23, 13 May 2009 (UTC)[reply]
... really? That deserves a [citation needed] tag. Nimur (talk) 19:11, 13 May 2009 (UTC)[reply]
They had such a machine on exhibit at the reconstructed Edison Menlo Park lab at Greenfield Village when I last toured the place. I found a reference which (snippet view) refers to the rotary motion being used to run a sewing machine, voice powered. "STITCH! STITCH! KEEP STITCHING DAMMIT!" Treadle powered and later electric powered machines were More conducive to domestic tranquility. "Menlo Park Reminiscences," by Francis Jehl, (writen 1937, Dover reprint 1990) (who was present at Edison's lab in the 1870'2-80's) tells on pages 180-181 of the "phonomotor." On page 180 is an illustration of it from the Scientific American (apparently July 27, 1878). It was the Sci Am reporter who said "Mr. Edison says he will have no difficulty in making the machine bore a hole through a board, but we consider such an application of the machine of very little utility, as we are familiar with voices that can accomplish that feat without the mechanical appliance." The machine used a diaphragm connected via a rubber tube to a spring, which carried a pawl that acted on a ratchet on the flywheel. Certain voice sounds could cause the flywheel to rotate "with considerable velocity." See also [2], [3], [4], [5], [6], and [7]. The notability guideline says "If a topic has received significant coverage in reliable secondary sources that are independent of the subject, it is presumed to satisfy the inclusion criteria for a stand-alone article." Does it appear that this little "philosophical toy" or demonstration gadget qualifies for its own stand-alone article, since it caught the fancy of scientific journals and science writers in various countries? Edison (talk) 21:13, 13 May 2009 (UTC)[reply]
Sounds like a great idea for an article...and once you get it into a halfway reasonable shape - you should definitely send a quote from it to the "Did You Know?" section of the Wikipedia front page. SteveBaker (talk) 22:41, 13 May 2009 (UTC)[reply]
Certainly a microphone fits the bill. It produces alternating current electricity from sound. SteveBaker (talk) 22:41, 13 May 2009 (UTC)[reply]
Thanks, guys! I forgot even about simple stuff like microphones! I must have gone mad!The Successor of Physics 13:03, 14 May 2009 (UTC)[reply]

megawatt to kg, dm and minute

can anyone tell me the answer to these four questions: (i dont need hints)

1. convert a power of one megawatt on a system whose fundamental units are 10 kg, 1 decimeter and 1 minute. 2. find the value of 60 joule/min on a systemwhich has 100g.cm and 1 min as fundamental units. 3. write the dimensions of a/b in the relation v is velocity and t is time. 4. write the dimensions of a * b in the relation E= b - x2/at, where E is energy, x is distance and t is time. —Preceding unsigned comment added by 122.50.139.37 (talk) 13:17, 13 May 2009 (UTC)[reply]

If we do your homework for you (instead of helping you do it yourself), will we get your course credit? -- Coneslayer (talk) 13:23, 13 May 2009 (UTC)[reply]


i have already solved these questions. i just need to verify my answers (there's no one else left but you people, cos my friends are all out of town and i'm on vacation. i need to check my answers)

If you show us your work, we'll be happy to confirm the answers. This is a core tenet of our do your own homework policy. — Lomn 13:45, 13 May 2009 (UTC)[reply]
I think in that case, what might work better, would e for you to tell us the answers that you got, and then we could see if they are correct? 65.121.141.34 (talk) 13:46, 13 May 2009 (UTC)[reply]
(EC) You could send us your answers and ask us to check them for you! But we really do have strict rules about not answering homework questions - it's not just people being mean to you. SteveBaker (talk) 13:48, 13 May 2009 (UTC)[reply]
Please do your own homework.
Welcome to the Wikipedia Reference Desk. Your question appears to be a homework question. I apologize if this is a misinterpretation, but it is our aim here not to do people's homework for them, but to merely aid them in doing it themselves. Letting someone else do your homework does not help you learn nearly as much as doing it yourself. Please attempt to solve the problem or answer the question yourself first. If you need help with a specific part of your homework, feel free to tell us where you are stuck and ask for help. If you need help grasping the concept of a problem, by all means let us know.

So I just measured my penis size, why am I not happy with it?

I am almost 8" inches long erect, definitely well over 7", but only 6" exactly in girth. Why am I not satisfied with my penis size?--Pipelinefine (talk) 14:28, 13 May 2009 (UTC)[reply]

The reference desk will not address medical or psychological issues. If you have a medical concern, see a physician. If you have a psychological concern, see a psychiatrist. Nimur (talk) 14:32, 13 May 2009 (UTC)[reply]

the reason probably has to do with your self-esteem (google self steem and impotence to see extreme examples), which is probably made worse by the stigma and discrimination you face for being a troll. 79.122.61.98 (talk) 16:58, 13 May 2009 (UTC)[reply]

Over 7" is above average, I believe, so your lack of satisfaction is presumably about you, not your penis, and since we only have information about your penis we cannot help you. If it is causing you significant distress, then see a psychiatrist (your GP/family doctor can refer you to one), otherwise just try and get over it! --Tango (talk) 17:15, 13 May 2009 (UTC)[reply]
Take it easy, no body else is measuring but you. Length graph. Girth graph. Mac Davis (talk) 03:42, 14 May 2009 (UTC)[reply]
Is this the time to say "It isn't the size of the ship, its the motion of the ocean"? A penis is to sex as a hockey puck is to hockey; it may be a needed component, but your enjoyment and understanding of the game will become far greater when you take a more holistic approach to it. Just as real hockey fans learn to appreciate the off-puck action, you may find that the enjoyment of all games improves when the focus is off of a specific "tool" in the game, but rather on the entirety of the game itself. --Jayron32.talk.contribs 04:07, 14 May 2009 (UTC)[reply]
Well said Jayron32, I hope your ocean is long in motion. Richard Avery (talk) 13:43, 14 May 2009 (UTC)[reply]
If you want information on normal range of variation in sizes, we do have it here in Wikipedia. See the detailed article Penis size. We may not answer medical questions at the desk, but we do have information in the encyclopedia from which you can answer them yourself. 8 x 6 is in the highest few percent, as you can see from the graphs there.DGG (talk) 15:27, 14 May 2009 (UTC)[reply]
Maybe because your penis is too large? If your measurements are accurate your penis is exceptionally above average, probably I expect in the top 1-2%. If this is a problem, I recommend as with most above you seek help from a professional who can help you learn to deal with your exceptionally large penis and whatever complications that arise. Simple advice would be take it slow, learn to communicate and use sufficient lubrication Nil Einne (talk) 12:38, 18 May 2009 (UTC)[reply]

Why does sticking your thumb on a water hose make the water come out faster?

I'm not sure if this is the right place to ask this question, but our article on the List of common misconceptions states (depending on which edit you happen to look at):

It is not true that a nozzle (or a person's thumb) on the end of a garden hose makes the water squirt farther because the same amount of water gets forced through a smaller opening. The rate of flow of water through the hose is not a set constant; in fact, putting one's thumb over the end of the hose reduces the rate of flow. What is constant is the water pressure at the source. When water is flowing, the pressure decreases the farther from the source one gets due to friction between the water and the pipes it's flowing through. The faster the water moves through the pipe, the greater is the friction that cuts down pressure at the output end. A thumb over the end of the hose decreases the flow rate, causing the friction from the source to decrease, causing the remaining water to have more speed.[1]

A discussion is here[8].

Would some editors who know more about friction and pressure help settle this issue? A Quest For Knowledge (talk) 14:56, 13 May 2009 (UTC)[reply]

Take a look at the above section, WP:RD/S#Submarine hull breach. SteveBaker's math is pretty applicable to the hose problem. Note the important distinction between mass flow rate and flow velocity. Reducing the aperture size (with your thumb) will increase the flow velocity but decrease the mass flow rate. Nimur (talk) 15:00, 13 May 2009 (UTC)[reply]

To put the above in layman's terms, covering the end of the hole with your thumb both reduces the amount of water coming out and increases the speed at which it comes out. These are not necessarily contradictory ideas. --Jayron32.talk.contribs 15:09, 13 May 2009 (UTC)[reply]
Jayron32, thank you for putting it in layman's terms but is the increased speed caused by pressure or friction? This seems to be the heart of the issue. A Quest For Knowledge (talk) 15:21, 13 May 2009 (UTC)[reply]
Both change and both are variables that change the velocity. David D. (Talk) 15:55, 13 May 2009 (UTC)[reply]
Having said that if you turn the tap off nothing happens so pressure is the primary issue. David D. (Talk) 15:59, 13 May 2009 (UTC)[reply]
I don't believe that pressure increases. You have the same amount of pressure pushing from the tap, which is forcing a similar amount of volume per second to flow through the hose (the rate will decrease somewhat due to increased flow resistance, analogous to a resistor in an electric circuit, but that is subtracting from the spraying effect, not adding). Because you have the same (or similar) amount of volume going through a smaller area, the water must speed up in a linear, meters/second way (if you think about it, volume flow rate, q, is equal to A*V, decreasing A while keeping a similar q will invariably lead to a higher V). The pressure drop should equalise (after a few seconds of flowing through your thumb) to approximately the same, it's going from whatever it is at the end of the pipe to atmospheric. To get a more mathematical idea about this phenomena, you might find Bernoulli's principle an interesting article. TastyCakes (talk) 16:14, 13 May 2009 (UTC)[reply]
The pressure at the nozzle does change. I agree the pressure at the valve does not. Also bear in mind there is not a constant flow rate in this example. David D. (Talk) 16:33, 13 May 2009 (UTC)[reply]
q is not approximately constant. In fact, for a fixed pressure difference the volumetric flow rate is strongly dependent on the size of the outlet. (Consider a basin full of water, the rate of water flowing out clearly depends on the number and size of the holes provided.) Also, water in a real hose is subject to viscosity, which affects the pressure reaching your thumb and is a factor not considered under Bernoulli. Dragons flight (talk) 16:35, 13 May 2009 (UTC)[reply]
For the purpose of Bernoulli, is water considered inviscid? I have only seen Bernoulli used in reference to gasses such as air, which is about 50-100x less viscous then water. 65.121.141.34 (talk) 16:23, 13 May 2009 (UTC)[reply]
Ya, Bernoulli's principle is true for all incompressible fluids as I understand it. In fact I think it should be simpler with an incompressible fluid than you've probably seen with air. TastyCakes (talk) 16:26, 13 May 2009 (UTC)[reply]
In short, I think saying it's a misconception about the water flow is incorrect. While it is true that the volumetric flow rate is not constant because of changes in friction etc is completely accurate, the concept is true (in my opinion) in that the area of the end of the hose is decreased much more than the volumetric flow rate is decreased, and as such the q = V*A equation dictates an increase in velocity, even though the effect is somewhat lessened by q's own reduction. The pressure will increase at the end of the pipe because the flow rate has reduced the friction loss along the pipe. But I think in your average garden hose the difference will be quite small, since the change in volumetric flow rate would also probably be quite small. It is not the main cause of the effect, in my opinion; the area reduction is. TastyCakes (talk) 16:36, 13 May 2009 (UTC)[reply]
In the laminar, viscous limit, the volumetric flow rate is proportional to A2 and so changes more rapidly than A (see: Hagen–Poiseuille equation). Obviously your thumb is not conducive to laminar flow, but I think your assumption that the change in q is small is still incorrect. Dragons flight (talk) 16:51, 13 May 2009 (UTC)[reply]
mm isn't it P that's proportional to A*r2 rather than proportional to the area squared? And isn't that only for circular cross sections, which wouldn't be true for the thumb over the hose analogy? TastyCakes (talk) 17:00, 13 May 2009 (UTC)[reply]
So HP would be correct for the most part but your average velocity leaving the pipe still follows the simpler formula:
Perhaps you are right that Q is greatly reduced, but it seems quite clear from the equation that in the hose example it is reduced much much less than A, else the water wouldn't come out of the pipe faster. TastyCakes (talk) 17:11, 13 May 2009 (UTC)[reply]
I don't think you're factoring in the resistance along the hose. As the flow rate decreases the resistance along the hose decreases too. The effect of this is an increase of pressure at the nozzle. David D. (Talk) 20:44, 13 May 2009 (UTC)[reply]


That problem cannot be correctly solved by either the Hagen-Poiseuille's equation or the Bernoulli's equation alone. The first one assumes an uninterrupted pipe of circular cross-section which is not true here because the thumb interrupts the flow at the end. The second one neglects friction all together and cannot explain the phenomenon. But a combination of both can solve the problem. Lets stablish three points of interest, , , and . will be a point somewhere in the water reservoir being kept at a constant pressure , will be a point at the end of the hose right before the nuzzle (or thumb) whose pressure is yet to be determined, and will be a point outside the nuzzle kept at atmospheric pressure . Lets define the pressure drops along the hose before and after (that is without the thumb and with the thumb).
, and
.
We can use Hagen-Poiseuille to compute the water flow before and after.
, and
.
To find we use Bernoulli
Where we used the fact that the flow through points and must be the same,
.
and are the cross-section areas at point in the hose and point at the thumb.
Now we place the expression we got for into the expression obtained from Bernoulli's equation.
Where we introduced the quantity . Solving the quadratic equation for we get
, and
.
If we then get and the flow does not change very much. but if we then get which gives a flow proportional to the area and a constant exit velocity. Dauto (talk) 21:03, 13 May 2009 (UTC)[reply]

why do women love chocolate

or is it a myth —Preceding unsigned comment added by 79.122.61.98 (talk) 16:43, 13 May 2009 (UTC)[reply]

Probabily the same reason why men love chocolate. Dauto (talk) 17:10, 13 May 2009 (UTC)[reply]
I'm talking about the stereotype. Either its a myth or women love chocolate more and differently than men. 79.122.61.98 (talk) 17:24, 13 May 2009 (UTC)[reply]
A few studies seem to suggest women do have a higher preference for chocolate than men do, but the reasons for this are not known (PMID 1799282, PMID 12954417). Rockpocket 17:34, 13 May 2009 (UTC)[reply]
Yeah, I prefer chocolate to men any day!--80.3.133.116 (talk) 17:38, 13 May 2009 (UTC)[reply]
Touché, (I've edited to clarify my meaning). Rockpocket 17:54, 13 May 2009 (UTC)[reply]
What are your views on men buying you chocolate? --Tango (talk) 17:59, 13 May 2009 (UTC)[reply]
Our survey says... I specifically linked to the account at confectionerynews to highlight that this survey was commissioned by a chocolate manufacturer. 80.41.120.247 (talk) 19:31, 13 May 2009 (UTC)[reply]
I'd always thought it was just that women tend made a bigger deal about going against whatever fad diet they were currently on and 'being naughty' by eating the oh-so-fattening chocolate that <whoever writes those womens' mags> says is bad. Guys seem to be much less concerned about their weights and figures. --Kurt Shaped Box (talk) 17:49, 13 May 2009 (UTC)[reply]
Well, yeah, but I think there's a little more to it. Since we're dealing in stereotypes, I would think of a guy chowing down on a massive steak or similar "manly" meal rather than a box of assorted chocolate. Matt Deres (talk) 18:11, 13 May 2009 (UTC)[reply]
I seem to recall women reacting more strongly to the serotonin-related effects than men, but maybe I'm misremembering this. I did get the impression that I have read numerous scientific discussions of why women do seem to have a much stronger psychological reaction to chocolate than most men do. --140.247.252.198 (talk) 18:57, 13 May 2009 (UTC)[reply]
Yeah, I remembered reading the same thing (not sure where) but was too afraid to post for fear of being flamed. Google scholar seems to remember it too. SpinningSpark 19:35, 13 May 2009 (UTC)[reply]
Can you link to a specific study showing evidence that chocolate per se affects serotonin (not "carbohydrates" or whatever, as lots of other foods contain this)? The "Chocolate: Food or Drug?" (J Am Diet Assoc.99:1249-1256.(1999)) study is pretty unscientific when it comes to the neurobiology part - acknowledging that PEA doesn't reach the brain (and higher quantities are in cheese) but despite that still speculating that because "7 MDMA abusers [displayed] an intense craving for chocolate [...] it is worth noting that MDMA is structurally related to PEA and may produce similar effects". Also I only see higher cravings for chocolate during pre-menstrual stage, along with a mention of cravings for carbohydrate being affected by serotonin. --Mark PEA (talk) 10:17, 14 May 2009 (UTC)[reply]

what is this "I did get the impression that I have read numerous" sentence?? This phrasing sounds schizophrenic to say the least...79.122.61.98 (talk)

What, you say?The Hand That Feeds You:Bite 21:29, 13 May 2009 (UTC)[reply]
When I write sentences like that it usually means I changed by mind about what I was going to say half way through writing it and didn't go back and change everything so it was consistent. --Tango (talk) 21:44, 13 May 2009 (UTC) [reply]

I fucking hate chocolate, and ice cream, and anything like that

A distinction should be made here between milk chocolate and dark chocolate. Dark chocolate has less sugar than milk chocolate, and perhaps is less preferred by women. In fact, chocolate alone is quite bitter. The Wikepedia article on chocolate says, "Aztecs made it into a beverage known as xocolatl, a Nahuatl word meaning 'bitter water' ". The Wikipedia article is well written, information dense, and interesting, especially if you are interested in making and manufacturing things. It includes a lead to "mouthfeel" which has some good definitions. - GlowWorm.

Perspective of an electron

I was thinking the other day what it would be like to be a person on an electron while it orbits the nucleus of an atom. I tried to calculate it out but it got confusing. What I tried to do was make the proportion of a height of a person (about 2 meters) over the diameter of Earth (12,756,200 meters) equal to X (the height of a person on an electron) over the diameter of an electron (about 5.636x10^-15 M). I got ~8.836x10^-22 meters. After I got here though I couldn't figure out where to go, in other words how I would be able to use this information to help me relate my size relative to Earth to what it would like on an electron. I thought of trying to compare the size of the Sun in Earth's sky to the nucleus of a hydrogen atom in an electrons "sky". If this makes any sense to anyone, I would appreciate some help! Thanks!

And if this is still kinda confusing, I'll try to simplify what I'm trying to do:

As we stand here on earth, we can look out the Sun, the Moon, other planets, and other stars and galaxies. If you notice, electrons, although they don't orbit like Earth does around the Sun, still "orbit" around the nucleus of an atom. I'm curious to see what the "sky" would look like if an electron was earth and we were people on it. How far away would the Nucleus (Sun) be? How large would it look in the "sky"?

Hopefully you can get my drift, and help me out. Thanks again! —Preceding unsigned comment added by 129.21.109.153 (talk) 21:45, 13 May 2009 (UTC)[reply]

I don't know where you got the diameter number for your electron - but it's wrong. Electrons are 'point particles' - they have no size. The nearest thing to a 'size' you could use would be some function of the probability cloud that describes it's position...but that would give you a really odd view of what you're trying to understand. SteveBaker (talk) 22:33, 13 May 2009 (UTC)[reply]
The OP is referrring to the classical electron radius. Although Steve is right; the electron has no actual physical size in the way that macroscopic objects do. Someguy1221 (talk) 22:40, 13 May 2009 (UTC)[reply]
Yes, but the classical radius of the electron isn't a very useful conceplt. Compton wavelength of the electron might be a better choice? Dauto (talk) 23:39, 13 May 2009 (UTC)[reply]
How about, what would be its radius if it were a Kerr–Newman black hole of the same mass, charge, and angular momentum? That would seem to be a limiting value closer than which you couldn't really model it as a point. --Trovatore (talk) 08:11, 14 May 2009 (UTC)[reply]

Oh ok I see what you are saying. I was actually thinking that too, but I got a number for it so I figured it must have a diameter. I suppose this means that this couldn't really happen then, huh? —Preceding unsigned comment added by 129.21.109.153 (talk) 00:33, 14 May 2009 (UTC)[reply]

Some of our models of the atom treat an electron as a discrete little "planet" orbiting the nucleus of the atom. This can be a useful model for explaining some simple aspects of atomic structure, but is fantastically far from reality. Depending on what you are attempting to study about an electron, you can model it as either a) a discrete particle b) a "cloud" surrounding the nucleus of the atom c) a standing wave d) a quantum mechanical wave function. Like any models, these have varying degrees of proximity to reality, as well as varying degrees of "ease of use" and "usefulness" in particular situations. If, however, you stop trying to force an electron to behave like something you have experience with, like a little ball, and instead just accept it for what it is, it makes it easier to understand the more accurate models of electron behavior. --Jayron32.talk.contribs 04:02, 14 May 2009 (UTC)[reply]
Yep. There are some things (especially at the quantum level) that you just can't come to understand by analogy with macroscopic things. You have to resort to these rather unsatisfying ways of coming to terms with one property by itself - but never really getting a 'feel' for what the entire object is like. SteveBaker (talk) 04:52, 14 May 2009 (UTC)[reply]
This is especially true because you wanted to know what things would "look like." Your visual perspective is based on electromagnetic waves of a certain wavelength, visible light. Those wavelengths are about 300 nanometers to 700 nanometers. If you tried to "see" anything smaller than those length scales, you would not be able to form an image. At the very best, you could try to do some kind of interferometry or amplitude detection (LIDAR ?) And every time one photon of light arrived, it would blow your "planet" (electron) into a totally different quantum state (with new position and momentum). I don't think you could "see" anything. Nimur (talk) 13:22, 14 May 2009 (UTC)[reply]
Also worth noting is that while applying classical physics understanding to a quantum mechanical problem produces problems like you described, the reverse is not true. All macroscopic particles obey quantum mechanical equations perfectly, its just that within the realm of real measurement devices, the "classical approximation" is functionally identical to the quantum mechanical reality. In other words, all objects have a corresponding wave function; even YOU do, its just that your wave function is not terribly useful in getting information about your expected behavior. --Jayron32.talk.contribs 18:25, 14 May 2009 (UTC)[reply]

Grumbling Tummy

Childish question, this may seem, but why does your tummy grumble and gurgle? What's it doing? Eating itself? (Well, digesting, anyway).--KageTora (영호 (影虎)) (talk) 22:28, 13 May 2009 (UTC)[reply]

See Borborygmus. (I'll admit, I just learnt that word, and isn't it fantastic?!) --Tango (talk) 23:03, 13 May 2009 (UTC)[reply]
Excellent! Interesting to note that it even has a plural 'borborygmi'. Thanks Tango!--KageTora (영호 (影虎)) (talk) 23:12, 13 May 2009 (UTC)[reply]


May 14

Zoological Society of London - Presidents and Secretaries

I've been working on Zoological Society of London and I have two questions about officers of that society:

  • (1) Zoological Society of London#Presidents - there are seven links there to pages about titles, rather than people. Effectively, those seven links need to be disambiguated to point to the right person. Can anyone here help with working out which earls, lords, marquesses, dukes and viscounts are which?
  • (2) Zoological Society of London#Secretaries - there are two entries missing (the officers of this society serve for three year terms). These two missing entries are currently marked by [...], but if anyone can find out who those two missing entries are, I'd be very grateful.

Thanks. This may be more of a history question - will post a link from over there as well. Carcharoth (talk) 00:04, 14 May 2009 (UTC)[reply]

Why don't you raise this content question here ? Cuddlyable3 (talk) 14:45, 14 May 2009 (UTC)[reply]
I was kind of hoping people here could help. I've sometimes asked questions here about obscure stuff for articles and got answers and sources that have been channelled back into the articles, thus improving them. Sometimes you are more likely to get an answer here than on the article talk page, though you are right in that that should also be tried. See also Wikipedia:WikiProject Reference Desk Article Collaboration and Template:WPRDAC. Carcharoth (talk) 21:17, 14 May 2009 (UTC)[reply]

Nature of electron

Is an electron a particle or is it an em wave ? —Preceding unsigned comment added by 79.75.124.111 (talk) 00:22, 14 May 2009 (UTC)[reply]

See Electron which states that "Electrons have quantum mechanical properties of both a particle and a wave". somehow i thinck u new that! -hydnjo (talk) 00:42, 14 May 2009 (UTC)[reply]
An electron is definitely not an em (electromagnetic) wave. The particle of the electromagnetic field is the photon, not the electron, despite the name. The electron is a different kind of wave. -- BenRG (talk) 01:22, 14 May 2009 (UTC)[reply]
The electron is neither a wave nor is it a particle. It is an electron. When we attempt to explain its behavior using only classical mechanics it becomes necessary to sometimes treat it like a discrete particle, and other times to treat it like a wave, but this is only because classical physics is inadequate for dealing with the realities of electron behavior. Classical mechanics lacks the ability to create an adequate model for fully explaining electron behavior, and so we end up with the unsatisfying paradox of wave-particle duality. Thankfully, quantum mechanics exists, and it takes care of these apparent paradoxes just fine. See also Copenhagen interpretation for some more background on the philosophy behind the quantum mechanics/classical mechanics tension. --Jayron32.talk.contribs 03:54, 14 May 2009 (UTC)[reply]

Future remote DNA sampling

SciFi warning - What could be a future way of remotely sampling someone's DNA for analysis, say, from a meter or so away? Could a laser be used? Thank you.190.17.201.142 (talk) 00:27, 14 May 2009 (UTC)[reply]

Normally you need to extract DNA out of cells, with steps continuing after that. See DNA extraction. Mac Davis (talk) 03:38, 14 May 2009 (UTC)[reply]
There is nothing in our current technology that could be imagined to be used one day for a kind of quick DNA scanning of a cell? 190.17.201.142 (talk) 03:52, 14 May 2009 (UTC)[reply]
You could imagine an extra-large Glonckian scanner. This is a device that superimposes a copy of itself into the subject and collects a sample, then uses quantum teleportation to move the virtual sample (but not the copied Glonckian scanner) back into the cloud chamber of the original scanner. Of course, Glonks are usually mean to scan much smaller molecules, but one might eventually be built with a really large sampling hydrator to capture an entire DNA molecule. Tempshill (talk) 04:08, 14 May 2009 (UTC)[reply]
I suppose one might be able to sample dead skin cells by blowing air-flows over the person in question. Once you have a sample (even a small one) then extracting the DNA and running some kind of classification scan on it could probably be automated and brought down to a couple of minutes. But the trouble with sampling at a distance like that would be lie in being 100% sure that the material you get a hold of did actually come from that person. Cross-contamination is a serious problem for DNA magnification techniques - just one DNA molecule from someone else would be plenty to screw up the results. I suspect that we'd use some other identification technique in the future. Between photographs, super-low-dosage X-rays and other remote sensing techniques - I'm sure you could easily gather enough biometric data to figure out who someone is. SteveBaker (talk) 04:47, 14 May 2009 (UTC)[reply]
you could fire a very tiny projectile, that knocked off a few cells, or jab with a long thin needle very quickly. Perhaps a kind of smart dust or nanobot could accomplish the mission. Graeme Bartlett (talk) 10:30, 14 May 2009 (UTC)[reply]
Think about it. Cuddlyable3 (talk) 14:38, 14 May 2009 (UTC)[reply]

average life of furniture?

Today I was discussing the concept of semi-durable goods with a friend, things like furniture, book shelves, plastic bins you might keep on your desk or for your clothes etc. I have also seen lots of yard sales + things on the side of the road, (old chairs, furniture etc) and It got me thinking about what the average life of most of these goods is. How long does the average ikea table last? Bookshelf ? Chair? etc. Seems to me there is a big range, from 6 mo's for the worst of it up to 100's of years for the best. Anyone have any thoughts on where to get this data? Or ever see it discussed? —Preceding unsigned comment added by 72.230.5.95 (talk) 01:04, 14 May 2009 (UTC)[reply]

Depends on what you mean by lasting. Do repaired items count as lasting? Because then the lifespan of a piece of furnature is indefinate, see the Ship of Theseus paradox for more... --Jayron32.talk.contribs 03:48, 14 May 2009 (UTC)[reply]
Well, things like tables don't ever wear out - they get progressively more marked and chipped and so forth - but the "life" of the object is almost infinite. We have a 200 year old hall table and it's 100% functional. I don't see why it wouldn't be functional in another 200 years. Certainly some classes of furniture can get broken - but that's a random event that's not in any way related to the age of the furniture. So I don't think you can put a lifespan on that - if it breaks, it's a random event that's as likely to happen on the day you buy it as on a day 200 years later - and if it just gets unacceptably "scruffy" then it's down to your tolerance for scruffy furniture - it'll still be perfectly functional when it's so beat up that you decide it's at the end of it's life and replace it. Obviously, upholstered furniture has a more certain life - but you can always re-upholster anything like that as many times as you need to. We had a pair of sofa's that saw hard duty in our family room (they were really seriously abused) and they were terminally scruffy by 15 years old. However, a few hours with a sewing machine and a staple gun - and they looked brand new again. Things like bookshelves literally can last forever. The bookcase in my study was the one that came "free" with a complete set of Encyclopedia Britannica's in 1960. The books are long gone (who needs them when you have Wikipedia?) - but the bookcase lives on.
There are antique pieces of furniture in daily use in some people's home that was made many centuries ago - it's still 100% functional. Just about the only furniture we've even been forced to throw out was a pair of crappy MDF computer desks that both failed due to the poor choice of construction techniques - but my victorian writing desk gets daily use - and will undoubtedly outlive me!
SteveBaker (talk) 04:41, 14 May 2009 (UTC)[reply]
If you are looking for record breakers look at furniture from old tombs (e.g. [9]). This is mostly a question of durable materials. laminated particle board or Medium density fiberboard contains glues and binders that will dissipate or change over time. So it's unlikely our descendants will find many "ancient" examples of that. Despite it's reputation plastic is only durable under ideal storage conditions. Otherwise the plasticizers will get lost over time and the piece will become brittle. Whether a piece of furniture survives depends a lot on what value their owner assigns to it. The Germans have a funny nickname for their bulky trash pick-up days "furniture trading day". (OR One of my ancestors is remembered by reportedly using a Rosewood cabinet as firewood. They had just gotten brand new plywood furniture.) 71.236.24.129 (talk) 08:58, 14 May 2009 (UTC)[reply]
When we lived in Germany (Moenchengladbach) in the late '70s it was also known as the "monthly schrank," a schrank [sp?] being a large item of furniture something like a Welsh dresser. The custom was that anyone looking for 2nd-hand furniture or other items such as stereos would tour the streets and take whatever they wanted before the municipal lorries came round to collect the remainder. There was no social stigma attached, and a lot of the goods put out were good quality and in good condition. 87.81.230.195 (talk) 23:34, 14 May 2009 (UTC)[reply]
Here are hypotheses about the lifetime of semi-durable household goods. They may or may not be possible to quantify by collecting data from sources such as media advertisements, private ads, repair services and municipal trash dumps.
  • An item can have more than one average lifetime
    • The time it stays fashionable
    • The time it is saleable, new or used
    • The time it is useable, including justifiable repairs
    • The time it is collectable.
  • The above lifetimes are different for a high-consumption relatively wealthy society or an economically challenged society. Cuddlyable3 (talk) 14:23, 14 May 2009 (UTC)[reply]
And all bets are off for your wooden furniture if you get termites. 65.121.141.34 (talk) 19:56, 14 May 2009 (UTC)[reply]
I would never touch second-hand furniture for fear that it might have bedbugs (Cimex lectularius). They seem to be spreading rapidly. They are a real horror. I won't even sit on a chair any more in a waiting room or other public place. I no longer go to movies or theaters. When bedbugs become well established in a home, they are almost impossible to get rid of, like cockroaches. At least cockroaches do not suck your blood. Wikipedia has an article on bedbugs. Also see bedbugger.com. - GlowWorm.
If you aren't against bugs in general check out this guy for a natural countermeasure: Scutigera coleoptrata. 71.236.24.129 (talk) 10:21, 15 May 2009 (UTC)[reply]

'Bubbles' in the Sky

You know when you look up into a clear blue sky (could be anything, really, but this is an example), and you see little tiny round 'bubbles', almost like bacteria, but you can't quite look directly at them because they are on the surface of your eye (I'm not mad, everyone I know sees them)? What are they called and, more importantly, what are they? Actually bubbles in the liquid on your eyes? Do they even have a name?--KageTora - (영호 (影虎)) (talk) 02:57, 14 May 2009 (UTC)[reply]

floaters. Dauto (talk) 03:01, 14 May 2009 (UTC)[reply]
Incredible! This is why I love Wikipedia! Thanks!--KageTora - (영호 (影虎)) (talk) 04:30, 14 May 2009 (UTC)[reply]

Name that disease

There was a tropical disease I heard about in Guam that humans would contract when they rubbed their backs against the trunks of palm trees for itch relief. Tiny microorganisms would be left on your body and you would feel very bad and cause your skin to itch more. To my knowledge it did not particularly afflict Guamanians, I just heard about it in the Navy hospital. Can not remember if this was fatal. Can anybody help? I thought it was Chagas disease but after reading the article I am not sure this is the case. Mac Davis (talk) 03:30, 14 May 2009 (UTC)[reply]

Chiggers? --Jayron32.talk.contribs 03:44, 14 May 2009 (UTC)[reply]
Our article I link above mentions several diseases carried by Chigger larvae, some of which are endemic to the Pacific islands. --Jayron32.talk.contribs 03:46, 14 May 2009 (UTC)[reply]

Faffing about in space

I've been watching NASA TV to see the crew of STS-125 capture the Hubble Space Telescope and start servicing it. One thing puzzled me about the entire process: Why does it take so long? After Atlantis crept up on Hubble over several hours, it stopped for some 40 minutes about 50 feet away. The robotic arm then took an age to creep up to the grapple point on the Hubble. After the Hubble was captured by the arm, it was just held there for at least another 30 minutes (an orbital sunrise and an orbital "midday" came and went), until the Hubble was eventually drawn into the cargo bay and docked to the waiting servicing pallet. In all, most of the second day was spent getting to and capturing the Hubble. So what was going on with the long waits and slow pace of things? Astronaut (talk) 05:38, 14 May 2009 (UTC)[reply]

It's probably something to do with trying to synchronise your own speed with that of the object you are trying to capture. After all, they are both travelling at 16,000 mph. They're not just floating. And after all, they don't want to break Hubble, otherwise that would make the mission pointless, so they have to be careful and do a lot of preparation and checks inside the spacecraft before they start to do anything.--KageTora - (영호 (影虎)) (talk) 06:35, 14 May 2009 (UTC)[reply]
They are being exceptionally cautious - both the Shuttle and the Hubble Space Telescope are stupidly expensive pieces of machinery (in Hubble's case, nearly irreplaceable). So they pause after every step to make sure everything is still working right, that everything is as it should be, before proceeding to the next step. Also, keep in mind that the Hubble weighs eleven tons - that's not the kind of mass you want to have crashing into your spacecraft at high speed! Hence, all the manoeuvring is done very slowly, again, to avoid breaking anything (the same way as you walking more slowly when carrying a tray of glasses filled with juice). — QuantumEleven 11:06, 14 May 2009 (UTC)[reply]
Or to put it bluntly, when YOU have an eleven-billion dollar ride, you can drive it however fast you want. Nimur (talk) 13:35, 14 May 2009 (UTC)[reply]

global positioning system

what iz global positioning system —Preceding unsigned comment added by 117.198.240.128 (talk) 10:08, 14 May 2009 (UTC)[reply]

Please see our article on Global Positioning System and come back if you have any further questions. --antilivedT | C | G 10:10, 14 May 2009 (UTC)[reply]
GPS = you can haz your location... 79.122.112.53 (talk) 11:16, 14 May 2009 (UTC)[reply]
Wouldn't cats prefer to csn haz Cheeseburger Positionig System? -- Ferkelparade π 11:25, 14 May 2009 (UTC) [reply]
Cats can haz needz both CPS and GPS. It's useless to know only where cheezburger is - if you don't know your relative position to it, you can't haz it. Nimur (talk) 13:33, 14 May 2009 (UTC)[reply]
Need lotz and lotz of lolSats. SteveBaker (talk) 19:09, 14 May 2009 (UTC)[reply]
This article may help. Cuddlyable3 (talk) 13:35, 14 May 2009 (UTC)[reply]
Wow! Thanks, I was not aware of a Simple Wikipedia. Jay (talk) 08:45, 15 May 2009 (UTC)[reply]

SI to cgs

hi, i've got this question's numerical answer but i'm confused about the units.

the question is : convert 7350 N/m2 into cgs units

what i did: 7350 * 105 gcm-2s-2m-2

7350 * 105 gcm-2s-2(100 cm)-2

7350 * 105 gcm-2s-2 (10000 cm)-1

7350 * 10 gcm-2s-2cm-1

this is where i'm confused. the cm-1 cancels the cm and the answer becomes g/cm2, but i (and you) know that it should be gcm/s2, or should the cm-1 not come at all from the 4th step to the 5th step??

(i could have done it the easy way. N/m2 is pressure. so i could have written it as pascal and converted it into barye. but, unfortunately, barye hasn't been taught in our school. so, i can't possibly use that method) —Preceding unsigned comment added by 122.50.131.71 (talk) 14:16, 14 May 2009 (UTC)[reply]

Search google for unit conversion and you can get all kinds of cool unit converter calculators. 65.121.141.34 (talk) 14:25, 14 May 2009 (UTC)[reply]
How about using dynes, which are the CGS unit of force? Google can do that conversion. Nimur (talk) 14:34, 14 May 2009 (UTC)[reply]
Your mistake is in your first line, you did not correctly convert Newtons to correct units in CGS system. Verify your units. Even in CGS, a unit of force should still be a mass times an acceleration. It looks like you mixed up the conversion of Newtons and the conversion of meters to centimeters in to the same line, and did it incorrectly. There should be no cm^-2 on that line, it should just be cm. Nimur (talk) 14:40, 14 May 2009 (UTC)[reply]
Nimur caught one mistake. The other one is that you converted m-2 incorrectly. the correct convertion would be (100 cm)-2=(10000 cm2)-1. Dauto (talk) 22:52, 14 May 2009 (UTC)[reply]

meiosis

Hi I'm a bit confused by the events of Metaphase I compared to metaphase II. In metaphase I I believe the bivalents line up randomly at the metaphase plate. This is a source of variation in meiosis. But when it comes to metaphase 2 - do the chromosomes line up randomly again before they split? Is this is another source of variation? I have read the meiosis article but am still a little confused. Please help! —Preceding unsigned comment added by 139.222.240.42 (talk) 15:30, 14 May 2009 (UTC)[reply]

As described in our Meiosis article:
  • Anaphase I separates homologous chromosomes
  • Anaphase II separates sister chromatids
If there were no recombination and no mutation, then anaphase I would generate variation (by randomly segregating homologous but not identical chromosomes) whereas anaphase II would separate identical sister chromatids. Because of mutation and recombination, sister chromatids can differ, so differences arise at both steps. Does that help? --Scray (talk) 22:30, 14 May 2009 (UTC)[reply]
Meiosis is definitely complicated, and you're not alone in being confused. When the chromosomes are copied they form identical sister chromatids that join up with the other homologous pair. The task of metaphase I is to randomly segregate the pairs of sister chromatids from each other so that the resulting cells are basically a random combination of the chromosomes that were originally inherited from your father and mother. This is clearly a source of variation and it basically describes independent assortment of alleles that are carried on different chromosomes. The task of metaphase II is to separate the sister chromatids so that there's only 1 copy of each chromosome. There isn't quite as much randomization at this point since the sister chromatids that are being separated were initially identical to each other. However, due to random crossing over that happens during the first part of meiosis, at the time of metaphase II, the sister chromatids are themselves different patchworks of the maternal and paternal chromosomes and this leads to an additional level of randomization in the final gametes. Does that help any? --- Medical geneticist (talk) 23:03, 14 May 2009 (UTC)[reply]

Yes thanks a lot for your help!

enhanced oil recovery

Why do they inject nitrogen into oil fields instead of regular air? 65.121.141.34 (talk) 16:13, 14 May 2009 (UTC)[reply]

Because air contains oxygen, which may chemically react with components of the oil. Nitrogen is chemically inert and will not produce any undesirable reactions. --Jayron32.talk.contribs 18:20, 14 May 2009 (UTC)[reply]
Oxygen is also a key component of combustion. Nitrogen is safer. cheers, 10draftsdeep (talk) 18:28, 14 May 2009 (UTC)[reply]
Particularly under the high pressures gas is injected at under EOR. TastyCakes (talk) 18:47, 14 May 2009 (UTC)[reply]
I have been under the impression that unrefined crude oil is not very easy to ignite, though I suppose that a lot of safety measures are needed since it would be a very expensive and dangerous fire. 65.121.141.34 (talk) 18:49, 14 May 2009 (UTC)[reply]
Well yes and no... At high pressures and somewhat high temperatures, as experienced in most oil and gas reservoirs, crude is probably somewhat combustible, at least that's my understanding since fire flooding is used in places (no wikipedia article on it, but here is a patent on the subject). That coal seams can burn for as long as they do seems to support that. But the real danger is that "light ends" (short hydrocarbon chains) will come off of even the heaviest crude oil to some degree, and these are very combustible. Add to this that oil wells will frequently encounter gas and gas condensates (in varying quantities) and I think it is clear that there is enough flammable stuff in an oil well to justify avoiding the use of oxygen. TastyCakes (talk) 20:15, 14 May 2009 (UTC)[reply]
You'll find nitrogen used in other applications as well. For example, most beer kegs use Carbon dioxide as a propellant, since it is a natural product of the fermentation process, so adding extra CO2 to the system seems like a fine idea. However, Guinness and a few other high-end specialty beers will use nitrogen instead. CO2 will dissolve in water to form carbonic acid which will alter the flavor of the beer. For really malty beers like stout, the additional sour flavor from the acid can change the flavor profile in ways the brewer does not want. Nitrogen will not produce a detectable change in flavor, plus being less soluble in water, nitrogen tends to produce smaller bubbles than CO2 does, which produces the characteristic "waterfall" effect of a well-poured pint of stout. --Jayron32.talk.contribs 18:56, 14 May 2009 (UTC)[reply]
They also offer it as a substitute for air in tires, claiming it leaks out less (or more slowly) due to the nitrogen molecules' larger size. TastyCakes (talk) 20:51, 14 May 2009 (UTC)[reply]
Well, that doesn't make any sense... Air is Nitrogen, Oxygen, and a Carbon Dioxide (and negligibly small quantities of anything else) (on checking, Argon is actually more common that CO2... but I think we're already in the "negligible" domain). Nitrogen is the lightest molecule of those, with a higher drift-velocity. It would leak out faster than air. It could be argued that less inert nitrogen reacts less with the tire rubber/synthetic, but that's also flimsy logic compared to the rate of mechanical, frictional wear against the road surface. Sounds like a scam... Nimur (talk) 21:31, 14 May 2009 (UTC)[reply]
It may be lighter, but it has a larger radius than Oxygen (although I can't find an exact number right now). I don't think you could really use CO2 because of acidity issues, and any other "inert gas" is likely to be too expensive and no better than nitrogen, size-wise. I don't think pure Oxygen would really be an option either, due to corrosion and explosion dangers. They also claim that having pure nitrogen without any moisture has benefits, including reduced corrosion (not a big issue as far as I know). I tend to side with you, however, without a proper study showing otherwise I am skeptical as to the benefits of it. Here is an article on the subject. TastyCakes (talk) 21:47, 14 May 2009 (UTC)[reply]
I'm pretty sure drift velocity will affect diffusion (leaking) a lot more than atomic radius. The pore sizes are less of an important factor than the statistical likelihood that an individual gas molecule will get to a pore. Thus the lighter, faster gas diffuses faster. See Graham's law for more on differential rates of gas flow. N2 will diffuse about 7% faster than O2. If diffusion leaking were actually occurring on relevant timescales, a 7% worse performance would be noticeable. I conclude that "leak reduction" is not at all the motivation for Nitrogen in tires OR enhanced oil recovery. It's much more likely that chemical reactivity is the issue in both cases (although of dubious benefit for at least the tire case). Nimur (talk) 22:34, 14 May 2009 (UTC)[reply]
Surely it is more a matter of the average pore size/hole size of vulcanized rubber than anything else? I would also think the viscosity of the two gasses at tire temperature would play a bigger part than their drift velocity for long term leak purposes, although I don't know the values for nitrogen or air. In any case, you don't have to convince me, I'm not going to buy it any time soon ;) TastyCakes (talk) 22:41, 14 May 2009 (UTC)[reply]
(undent) Just to correct an inaccuracy above, if gas is leaking out of pores, the pores are likely millions of molecular radii across, and so the size of the molecule has NO bearing on the issue. All that matters is the molecular mass of the molecule, because at any given temperature, heavier molecules move slower, which means they "hit" the hole less often than do lighter gases. That is the only relevant property when considering this. Thus, regardless of the volume or linear size of the nitrogen molecule vis-a-vis the oxygen molecule, its only the mass that affects the leakage. See Graham's law for more info. The third paragraph of the "history" section explains it quite well. --Jayron32.talk.contribs 22:53, 14 May 2009 (UTC)[reply]
Hmm ok I guess that makes sense. Do you know how big the "holes" are in tire rubber? Are they really that much larger than nitrogen molecules? TastyCakes (talk) 14:10, 15 May 2009 (UTC)[reply]
Removing oxygen also avoids the development of bacteria (bugs) that can foul a well. Besides avoiding other chemical reactivity I think this is the major reason for oxygen and carbon dioxide removal. They even have to degas any surface water they inject.--OMCV (talk) 13:07, 15 May 2009 (UTC)[reply]

what is nitrostyrene?

wat is it made up of..ect. anybody know? its not on wikki —Preceding unsigned comment added by 66.237.50.35 (talk) 16:17, 14 May 2009 (UTC)[reply]

Assuming the nitro group is not directly bound to the benzene ring, then this is what it is: http://www.orgsyn.org/orgsyn/orgsyn/prepContent.asp?prep=cv1p0413 --Russoc4 (talk) 17:10, 14 May 2009 (UTC)[reply]
To answer the poster, a chemical name tells you exactly what it's made up of: it's a nitro attached to a styrene molecule. You can see that styrene has several different shapes/parts, so what we can't answer is exactly where the nitro is attached. Russoc mentions one possibility (that compound has some interesting chemical reactions possible!). Also, meta-nitrostyrene and para-nitrostyrene (with the nitro attached to the benzene ring in various places) have some uses, especially in their analogous nitropolystyrene polymer form. Apparently you can either polymerize the nitrostyrene directly or else polymerize styrene (or some other derivative) and then nitrate it. DMacks (talk) 20:08, 15 May 2009 (UTC)[reply]

KCN

What is the saturation point of KCN? Where would I find this information? 128.193.170.120 (talk) 17:50, 14 May 2009 (UTC)[reply]

Is there any kind of technology on the internet that lets you take a climate chart of a particular location, provided by the site, and do a search to come up with all the climate charts around the world that correspond most closely with the initial chart? This climate chart would include average monthly highs and lows and average monthly precipitation. —Preceding unsigned comment added by 189.4.53.90 (talk) 19:18, 14 May 2009 (UTC)[reply]

Rockets

We currently almost always launch rockets east when going to orbit (to benefit from the rotation of the earth). My question is if there is any consequence to always launching east, like possibly slowing the length of the day since you're pushing the earth back?

I'm sure the effect is tiny, but what if it's 500 years from now and we have billions of people launching megaton spaceships eastward to save on their fuel costs? Would the effect then become noticeable? TheFutureAwaits (talk) 19:31, 14 May 2009 (UTC)[reply]

Actually, some rockets are lunched north into a polar orbit, but you are right, most rockets are to the east. The earth weight is so high, and its spin rate so fast, that in order to slow the length of the day an amount that will be perceptible to people without clock accurate to 10 decimal places, rockets will have no effect until such point that we are launching in the trillions of tons per year range. At that point, there might be an effect, but it will be a very polluted planet with all the rocket exhaust. 65.121.141.34 (talk) 19:55, 14 May 2009 (UTC)[reply]
I think, in the US at least, the only reason rockets are launched east is so they travel over uninhabited water, lessening the adverse consequences of an aborted/failed launch. I don't know about launches outside of Cape Canaveral, though. I am not aware of any physical benefit of always launching east, other than impact speeds of space junk from previous launches would be lessened. As for affecting the rotation of the earth, the wind has a much greater effect than our various items in orbit, by several orders of magnitude. The tidal forces from the moon have an even greater effect. Unless we eventually have a space station a significant percentage of the mass of the moon, there should be no discernable effect. -RunningOnBrains 20:03, 14 May 2009 (UTC)[reply]
Actually ROB, if you launch east, you already are traveling at 700-1000 mph, so your rocket does not need quite so much fuel to get into a speed high enough for orbit. If you launched west, you would have the opposite effect and your rocket would start at -1000 mph instead of +1000 mph and would need to increase your speed 2000 more mph then an east launch. 65.121.141.34 (talk) 20:14, 14 May 2009 (UTC)[reply]
What goes up comes (usually) down. Launching a rocket to the east is not done by tilting the rocket on the launch pad. Cuddlyable3 (talk) 20:22, 14 May 2009 (UTC)[reply]
No, he's right...that was dumb of me. Launching east does give an angular boost.-RunningOnBrains 20:27, 14 May 2009 (UTC)[reply]
The angular momentum of the a closed system is conserved, so system consisting of the earth plus the artificial satellites would have a constant angular momentum, which mens the earth wourl slow down, IF the system were closed. However, the system is not closed. the eastward launch requires the expulsion of rocket exhaust in the westward direction.IF the exhaust exceeds escape velocity, then the eastward angular momentum of the system increases. Note that this effect and the transfer effect you originally asked about are trivial by comparson to the effect of tidal drag, which transfers angular momentum from earth rotation to lunar orbital distance. -Arch dude (talk) 02:07, 15 May 2009 (UTC)[reply]
If and when we reach the technological level of launching trillions of tons into space, i am sure we'll have methods to counter this effect. --131.188.3.21 (talk) 11:15, 15 May 2009 (UTC)[reply]
I'd hope we'd have a working space elevator by then - or at least, that we'd start building our huge space stations in space. Presumably the heaviest parts of a space station are minerals, which can be mined from asteroids - we'd only need to ship organic compounds from planetside. Dcoetzee 03:12, 19 May 2009 (UTC)[reply]
It might be tricky to develop a 0g metal refining operation. Your melted metal would go everywhere wouldn't it? 65.121.141.34 (talk) 18:11, 21 May 2009 (UTC)[reply]


May 15

Can tame, captive-bred animals be used to breed offspring that can be returned to the wild?

I notice that my captive-bred pet sun conure is a member of a species that is endangered in the wild. As far as I know, there are loads of sun conures in captivity. There might even be more kept as companion parrots in people's homes than there are in the wild. So, would it be possible, if the need ever arose, to breed pet sun conures and release the young ones (that weren't familiar with humans) back into the wild? Or would they be unable to survive? Thanks. --90.241.160.124 (talk) 00:03, 15 May 2009 (UTC)[reply]

Does Captive breeding answer your questions? Tempshill (talk) 01:02, 15 May 2009 (UTC)[reply]
Not really. I'm interested in knowing if animals that behave exactly like they would in the wild can ever be bred in captivity from parents that are pets? We can talk about sun conures specifically, if that makes it any easier. Thanks. --90.241.160.124 (talk) 01:39, 15 May 2009 (UTC)[reply]
Do you mean to ask if is possible to intentionally create a feral population? Dauto (talk) 02:05, 15 May 2009 (UTC)[reply]
I suppose I am, yeah. But only in the areas where the bird is or was found naturally. --90.241.160.124 (talk) 02:45, 15 May 2009 (UTC)[reply]
Amazingly we don't have an article about the parrots of Telegraph Hill. If escaped parrots can adapt to such an unnatural environment, one would think they should be able to readapt to their native environment. Looie496 (talk) 03:11, 15 May 2009 (UTC)[reply]
Reintroduction may have some information for you. There is a difference between having the same skills as a member of a wild population, being able to integrate into and survive within a wild population and having sufficient skills to survive in the wild. Just to illustrate: A captive bred conure may not know the warning call a wild one would use for a panther. If it doesn't get eaten by one, it or it's descendants may be able to learn that. The captive conure may not have the proper social skills and knowledge about nesting to make it within a wild flock. If they are bred in sufficient numbers and with sufficient diversity to form their own flock they may develop skills they can pass on to their next generation, even if those are different from wild conure behavior. For example the parrots of Telegraph Hill might have a call for "hawk" that their rainforest relatives may not have or need. This form of reintroduction usually involves a high attrition rate when scientists have tried it. Successful reintroduction programs usually involve lots of steps, close monitoring and possibly several generations. An animals survival strategy also plays a role here. Some animals just breed in such large numbers that they can stand losing a lot of individuals. Animals with low reproductive rates usually invest in longer training periods and parental care to ensure their offspring make it. Hope this helps.71.236.24.129 (talk) 04:46, 15 May 2009 (UTC)[reply]
Domestic cats can product feral offspring. If a domestic stray cat has kittens, and they are not socialised (introduced to humans) within a certain time period, they will difficult or impossible to tame or introduce successfully into a domestic environment. I'm not sure we'll ever need to breed cats for release into the wild, though. --Kateshortforbob 11:22, 15 May 2009 (UTC)[reply]
Cats are a whole different ball game. I have successfully reintroduced several feral cats into domestic environments. It just takes an awful lot of effort and a certain type of home. (A townhouse apartment won't work. A barn would be ideal.) Since our shelters are overflowing with domesticated cats shopping for a home it's usually not worth the trouble. I even used to have a semi-tame wild cat that I could pet and feed (while the door to the outside remained open.) Most experts will tell you they're impossible to domesticate. Domestic cats do not need to be reintroduced into their natural habitat - which would be people's homes. Other Felinae like Felis silvestris, Prionailurus viverrinus, Felis margarita and Felis manul are in decline and captive breeding and reintroduction programs are under way. They face many different challenges like diseases, difficulty breeding, loss of prey and loss of habitat or human encroachment. The target here is not to domesticate them, to make them pets, and then reintroduce their progeny into the wild, but to keep breeding pairs in human controlled environments or zoos and then prepare their offspring for release into the wild. Those would still not be feral cats though. With zoo raised cats you have the same issues with reintroduction as described above, which is why they usually try to keep them in reserves or sanctuaries. With both cats an conures the question becomes where the "wild" that you want to reintroduce them into can still be found. 71.236.24.129 (talk) 14:35, 15 May 2009 (UTC)[reply]

Yes I believe that individual home reared Conures can successfully survive in what is left of thier native habitat. My largest concern would be with the diversity and size of the gene pool. Geneticists will tell you that without a "certain" amount of diversity and a certain number of indivivuals (there is a formula for such)a reintroduced population is doomed to failure. Wikipedia probably has articles on genetic bottlenecking, gene pools and other relevant topics but i don't know how to link them here. 67.193.179.241 (talk) 12:03, 15 May 2009 (UTC) Rana sylvatica[reply]

Huntington's disease in Scotland

I read in one source that Huntington's disease has an extraordinarily high prevalence in "some areas of Scotland". Which ones? 99.245.16.164 (talk) 00:20, 15 May 2009 (UTC)[reply]

The Moray Firth apparently. SpinningSpark 06:28, 15 May 2009 (UTC)[reply]

Making the Rage Virus

Is it possible for scientists to create a virus that could make humans rage with anger and cause them to kill other humans and animals? 174.114.236.41 (talk) 01:20, 15 May 2009 (UTC)[reply]

It already exists -- it's called intolerance. Wikiant (talk) 01:25, 15 May 2009 (UTC)[reply]
... and it's already mutated into a virulent strain, increasingly common in urbanized areas especially where bitumen or asphalt takes up a large proportion of the land surface area, and is known colloquially as Road rage. Fortunately, there are treatments available. BoundaryRider (talk) 02:01, 15 May 2009 (UTC)[reply]
Well how about an actual useful answer? A virus like this somewhat already exists. The rabies virus turns dogs and foxes into crazed killers - but fortunately, not so in humans (although it's nasty enough without that particular symptom) - but if such a virus is possible in other mammals, I could imagine something similar happening in humans. Our article explains that "In many cases the infected animal is exceptionally aggressive, may attack without provocation, and exhibits otherwise uncharacteristic behavior."...and..."The first stage is a one- to three-day period characterized by behavioral changes and is known as the prodromal stage. The second stage is the excitative stage, which lasts three to four days. It is this stage that is often known as furious rabies due to the tendency of the affected dog to be hyperreactive to external stimuli and bite at anything near."...and also..."a new symptom of rabies has been observed in foxes. Probably at the beginning of the prodromal stage, foxes, who are extremely cautious by nature, seem to lose this instinct."
That's a virus that's transmitted in saliva and has therefore evolved to cause its victims to go nuts and start attacking and biting.
SteveBaker (talk) 02:46, 15 May 2009 (UTC)[reply]
There are lots of infectious agents which do induce delirium and brain damage in their hosts. Depending on what parts of the brain are affected, it is not unreasonable to find an infectious disease which would cause the result described by the OP. --Jayron32.talk.contribs 04:21, 15 May 2009 (UTC)[reply]
The challenge is finding a virus that consistently targets brain areas in just the right way to induce a behavior like the "furious rabies" that Steve's (useful) answer cites. I don't think scientists currently have the knowledge or tools to design such an agent. --Scray (talk) 04:57, 15 May 2009 (UTC)[reply]
It would probably be quite easy to do this, one doesn't need to know the mechanism to select for things like this. After all somebody with this level of morals wouldn't mind human testing. It is totally unnecessary though, it seems altogether too easy to get whole nations to take up arms against others and bomb and torture and otherwise behave vilely. You want a way of doing this even less controllably? 08:07, 15 May 2009 (UTC)
In case anyone has missed the reference, the "Rage virus" is the premise of the film 28 Days Later. --Tango (talk) 13:59, 15 May 2009 (UTC)[reply]
It is definitely possible, though how hard it is I don't know. What you need is a virus that can get into the bloodstream to have DNA that will code for enzymes that will synthesize a psychotomimetic. The DNA/RNA for enzymes is easy to extract from plants/fungi (ergot, atropa belladonna), but I dont think these drugs (lysergamides, atropine, respectively) would cause people to start killing each other, although people under the influence of deliriants do sometimes accidently kill themselves (I can't find the ref right now, but some english guy flew to new york and took too many nytols to overcome jet lag. He was found in his hotel room bleeding from the head, he had smashed his head in with a lamp or something along those lines. Google isn't finding it unfortunately). --Mark PEA (talk) 17:06, 15 May 2009 (UTC)[reply]
Though not a virus, if you watch the film Jacob's Ladder (film), you will see that BZ, a derivative (or ingredient) of LSD is purported to do this.--KageTora - (영호 (影虎)) (talk) 22:34, 16 May 2009 (UTC)[reply]
We have an article on BZ, which is an anticholinergic like atropine that I mentioned earlier (except it lasts longer, alot longer). Just a note, BZ is not an ingredient of LSD, nor is it anything like it in fact. LSD can be classed as a psychedelic or less specifically an hallucinogen, where as BZ/atropine are deliriants and have completely different mechanism of action. I'll might have to check out that film. --Mark PEA (talk) 22:26, 18 May 2009 (UTC)[reply]

Partial Pressure

Hello. There is an equal amount of nitrogen gas and hydrogen gas in a balloon that has a pinhole leak. There is no word of where the leak is exactly on the balloon. The temperature stays constant. Why will the partial pressure of nitrogen gas exceed that of hydrogen gas? Thanks in advance. --Mayfare (talk) 03:53, 15 May 2009 (UTC)[reply]

Sounds like homework, but anyway, you can read about Graham's law here just as well as in a science textbook. DMacks (talk) 04:12, 15 May 2009 (UTC)[reply]
Which molecules are moving faster? So which molecules will hit the pinhole more often? So which molecules will leave the balloon faster? So what happens to the ratio of the gases in the balloon? Answer these questions, or just read the Graham's law article cited above. --Jayron32.talk.contribs 04:20, 15 May 2009 (UTC)[reply]
Two! Two Graham's Law answers on WP:RD/S! Ah ah ah ah ah. Count von Count (talk) 04:49, 15 May 2009 (UTC)
I'm surprised that no one picked up on the fact that there are two answers to this question: The rate of effusion may be greater, but additionally, in air Nitrogen is much more plentiful, so even if Hydrogen were heavier, it would still eventually have a lower partial pressure in the balloon. -RunningOnBrains(talk page) 18:25, 19 May 2009 (UTC)[reply]

Yet another perpetual motion machine

I know it wouldn't work. Tell me why. An electric transformer generates a voltage on one side proportional to the change in voltage on the other. If you put an exponentially increasing voltage one one side, that should generate an exponentially increasing voltage on the other, which would generate an exponentially increasing voltage on the first. This would make a self-sustaining increase in voltage. Another way to do the same basic thing would be by moving a charged particle along an exponential path. The Abraham–Lorentz force will create a force on it proportional to its jerk. — DanielLC 05:10, 15 May 2009 (UTC)[reply]

energy is lost as heat. —Preceding unsigned comment added by 82.44.54.169 (talk) 05:38, 15 May 2009 (UTC)[reply]
OP: memorize those five words, because it is (probably) the answer to most any perpetual motion machine you could think of (except the fancy stuff you could try to do with magnets, which might end up depeleting their own magnetic charges over time, and slow down for that reason). As to why, thermodynamics should give you the answer to that... —Preceding unsigned comment added by 79.122.38.222 (talk) 10:57, 15 May 2009 (UTC)[reply]
Daniel, by the Lenz's law the induced electromotive force back on the first side should opose the one you had to begin with, and not reinforce it. Read the article I linked. Dauto (talk) 12:56, 15 May 2009 (UTC)[reply]
1)The induced waveform would only be proportional to the input exponentially increasing waveform until the magnetic core material saturated. 2)There is a frequency response limitation in a transformer; it does not have infinite bandwidth, so the output waveform would not be able to reproduce an input waveform with too high of a rise time. The induced voltage from a cable thumper or lightning may not be able to travel through a power transformer, for instance, because the impulse is too brief. (Safety note: Don't experiment with this at home.).Edison (talk) 18:11, 15 May 2009 (UTC)[reply]
This would disobey the first law of thermodynamics, not just the second. As such, heat escaping wouldn't make it not a perpetual motion machine. You can't create any energy, useful or not. In any case, you could theoretically change the curve to account for any loss in voltage. The second law of thermodynamics tells me that it won't work, but not why. I still don't understand that whole thing with flux. Is the problem that there is only one flux field, and as such I can't treat the two inductions separately? That still doesn't answer the Abraham-Loretz force version. Is that one similar? I get the feeling it is, but I'm not sure. — DanielLC 19:32, 16 May 2009 (UTC)[reply]
Yes, the problem is that you can't treat the magnetic fields or voltages (whether applied or induced) separately. They are coupled together, so you must treat them together, otherwise you'll get a wrong answer. This is similar to how a differential equation cannot (in general) be solved by isolating each term and then summing over the termwise answers. As for the bit about the Abraham-Lorentz force, the article says it nicely: "the motion of the particle must be periodic (an assumption that is explicitly made in the derivation of the force)". Someone42 (talk) 05:45, 17 May 2009 (UTC)[reply]

molarity

here's the question: what volume of 36M and 1M sulphuric acid must me mixed to get 1L of 6M sulphuric acid?

i need some explanation. don't just give me hints... explain those hints too, please... i'm new to this concept. —Preceding unsigned comment added by 122.50.136.97 (talk) 05:14, 15 May 2009 (UTC)[reply]

First you need to figure out the proportions needed to get a 6M concentration. Then you can figure out the amounts to give you the total volume you need. If you add X liters of 36M (36 moles/liter) solution to Y liters of 1M (1 moles/liter) solution, figure out the total number of moles that gives you and the total volume in terms of X and Y. Now the total number of moles divided by the total volume gives you the new molarity, which you want to be 6. That gives you enough information to form an equation that you can reduce down to a ratio between X and Y. From there, it should be a simple matter of calculating the amounts of each solution needed to give you the desired 1 liter result. -- Tcncv (talk) 05:41, 15 May 2009 (UTC)[reply]
Example: Suppose you had 1M and 5M solutions and needed 1 liter of 2M solution. Using the above approach, X liters of 1M solution added to Y liters of 5M solution gives you 1X + 5Y moles in X + Y liters. To get a 2M solution, set (1X + 5Y)/(X + Y) = 2. This can be rewritten as X + 5Y = 2(X + Y) or 3Y = X. So you need three parts 1M solution to 1 part 4M solution, this works out to 3/4 liters and 1/4 liters respectively to give you the desired amount. -- Tcncv (talk) 05:51, 15 May 2009 (UTC)[reply]

Gene Accession to Protein ID conversion

I am attempting to convert a large (1000s) list of gene accession numbers into their corresponding protein IDs (SwissProt/UniProt accession). The site [[10]] performed the task, but only returned protein accession numbers for ~80% of the supplied gene accession numbers, seemingly lacking the SwissProt IDs to the more recently characterized proteins. I looked at Pubmed, and found it to be more complete; but, as far as I know, the IDs can only be retrieved individually there rather than by a large list, which is obviously inefficient. I was wondering if there is a similar program to these with (ideally) the completeness of pubmed and the speed of idconverter. (NB: in addition to the gene accession numbers, I also have, or can acquire, the gene names and common names.) Any help will be greatly appreciated!Lashyn (talk) 06:03, 15 May 2009 (UTC)[reply]

Lashyn, I don't have an answer but I suggest you ask User:AndrewGNF who runs the Protein Box Bot. The bot seeds gene articles with the type of information you are trying to get. I'm not sure if Andrew has run across the same problem or not, but its worth asking. David D. (Talk) 06:56, 15 May 2009 (UTC)[reply]
I believe that SwissProt requires a pretty high level of evidence in order to get an accession number and there simply may be no SwissProt ID for every gene ID. The NCBI RefSeq genes are highly curated but other gene IDs may be predicted coding sequences, etc. The fact that you found SwissProt accession numbers for only 80% of the gene accession numbers is probably about right. Another way to look at it would be to take all SwissProt accession numbers and then find the corresponding Gene IDs. Then you'll have a complete list of the GeneIDs that CAN be linked to a SwissProt ID. --- Medical geneticist (talk) 12:24, 15 May 2009 (UTC)[reply]

Wet, Soap, Wash, Rinse, Dry

Since Swine Flu became the new thing to worry about i've seen a marked increase in how to wash your hands posters in toilets. Fair does, it's cheap to roll out and I suspect does make people more likely to wash their hands (reminders often do - even for seemingly insanely normal things). Anyhoo my question was this...I prefer to Soap, wet, wash, rinse, dry - does that (changed) ordering make any difference? Will my hands be less clean because I didn't wet before soaping? I guess it's nothing but what are Friday afternoons for if not trying to get answers to (largely) pointless questions? 194.221.133.226 (talk) 13:25, 15 May 2009 (UTC)[reply]

I'm sorry, but your order of doing things is wrong, and you now probably have Swine Flu. Please see a doctor. (Just kidding, of course. I don't think it matters a whole lot if you put the soap on before you've wetted your hands, as long as you take the time to wash and rinse.) --98.217.14.211 (talk) 13:42, 15 May 2009 (UTC)[reply]
I agree - your imminent death soon after you fall over into a muddy puddle and roll around snorting contentedly - is now virtually certain. The soap lowers the surface tension of the water and acts as a surfactant to encapsulate whatever comes off of the surface so it can rinse away more easily. You need the soap and the water to mix completely because neither soap or water alone can perform this magic trick. Whether you add soap to water or water to soap isn't so critical...although I guess if you first cover your hands with soap and then hold them under running water, you may send more of it down the sink than if you'd first wetted your hands, then shut off the water and then added soap. However, since most people use VASTLY more soap than they need - it's probably just as effective your way. The new advice for me was the amount of time to wash for. They are telling kids to sing the alphabet song while they wash - since that's timed to be about 15 seconds. On NPR the other day - they recommended that adults sing "Bohemian Rhapsody" but since that was (at the time it was released) the longest 'single' record ever made (5 minutes, 55 seconds) - I think that NPR may have been kidding! SteveBaker (talk) 14:59, 15 May 2009 (UTC)[reply]
Damn, I've been washing my hands while humming In-A-Gadda-Da-Vida. I don't have much skin left, but I laugh in the face of swine flu! Matt Deres (talk) 16:26, 15 May 2009 (UTC) [reply]
Hate to disabuse you, Steve, but the single of "Hey Jude" lasted 7:11, and that's not the only pre-1975 single longer than "Bohemian Rhapsody." Deor (talk) 21:49, 15 May 2009 (UTC)[reply]
More released singles longer than Bohemian Rhapsody, and also released prior to Bohemian Rhapsody include: 1971's American Pie at 8:33, the 1972 single version of Papa Was a Rollin' Stone at 6:54 (the album version was 11:54), 1972's Jessica at 7:30, the 1972 7:02 single verion of Layla (more successful as a single than the 1971 2:43 version). 5:55 is long, but when released it was certainly not even close to the longest single released to that point. Sorry Steve. For the record, I sing the full album cut of the Rare Earth version of Get Ready. With lava soap and a steel brush. My hands are a bit raw, but there ain't nothing on them after that. --Jayron32.talk.contribs 01:25, 16 May 2009 (UTC)[reply]
Now you're just making fun. If you children can't play nicely together I'll turn this ref desk around! --Tango (talk) 16:50, 16 May 2009 (UTC)[reply]
My sister is a medical student and she was telling me about their lessons in washing your hands (they have a very strict way to do it, washing each part in a very specific order, but I think that's just to make sure they don't forget a bit rather than because there is anything better about that order) and the interesting thing she said is that the key thing for getting rid of viruses is the mechanical action of scrubbing/rubbing/whatever your hands. The soap doesn't do all that much and, obviously, antibacterial agents don't help against viruses. So, I think the advice to wash your hands for a significant amount of time is probably good. --Tango (talk) 17:47, 15 May 2009 (UTC)[reply]
My wife used to be an operating department nurse - and that whole scrubbing-up ritual really does get burned into the soul. She hasn't worked at that job for 5 years - but you can still catch her standing there with her hands held up like a kangaroo when she's deep in thought because she spent so long being told to keep her hands firmly above waist height after scrubbing. SteveBaker (talk) 17:57, 15 May 2009 (UTC)[reply]

What is reality ?

I am a teenager, and i have read a bit about Quantum Mechanics, its interpretations, wave function collapses, and paradoxes. My question is, Where does this all leave us ?
Can we still think that the world is deterministic, that it is written on my forehead what i will do? Is there a place for free will ? Is there reality out there, or when i close my eyes does everything turn into imaginary wave functions with respect to me ? Is physics really complete, at low velocities, ie, is QM infallible in non-relativistic situations? I understand that there is a bit of a debate over these topics, but i do want to know where the scientific community stands at this point. In short, can someone please explain, what is reality ?Rkr1991 (talk) 13:41, 15 May 2009 (UTC)[reply]

Reality is anything that does not goes away after you are utterly sick of it. For example: being unemployed is a reality for many people. 122.107.207.98 (talk) 14:12, 15 May 2009 (UTC)[reply]

"Reality is that which, when you stop believing in it, doesn't go away." - Phillip K. Dick. — The Hand That Feeds You:Bite 21:30, 18 May 2009 (UTC)[reply]
The uncertainties of the quantum world tend to average out at the macro scale that humans live on. So for most of the time, it can be ignored and the world treated as a more or less deterministic place. However, we humans are cunning and we can magnify quantum uncertainty to the point where it becomes visible and important (eg in the Schrodinger's Cat thought experiment). But Quantum theory isn't the only source of indeterminacy. There are many systems out there which are 'chaotic' in the mathematical sense that the outcome of some experiment on these systems depends on the initial conditions with extreme (and possibly, infinite) sensitivity. You've heard of "The Butterfly Effect" where (it is said) the flapping of a butterfly's wings can change the course of a hurricane on the opposite side of the planet a year later. Well, that may be an underestimate. It's possible that a quantum fluctuation of a single electron would be enough to change the weather patterns. So determinism exists - but only strictly within non-chaotic systems - or over shorter time/distance scales - and not at very small scales. However, we can still use basic victorian-style physics to build bridges that don't fall down and airplanes the size of large buildings that seem to defy gravity by exploiting classical airflow laws. We construct amazing things that work pretty much as we expect them to - and that's only possible because determinism is reliable in most cases.
I don't believe there is free will. What goes on inside the brain is a matter of cellular biology - which is basically chemistry - which is basically physics. There is no magical 'outside' will that changes what happens in the physics - and either with or without quantum or chaotic determinism, what is the thing that is making the choice for that chemistry to either go out for Chinese take-out or stay home and cook left-overs? Ultimately, chemistry is what drives that. But it doesn't matter because it seems to us like we have free will - so we might as well act as if we do.
Whether there is a "reality" out there or not is a non-falsifiable proposition - there is no conceivable experiment you can do that would show that it's not all a figment of your imagination. You've seen "The Matrix" - right? So science takes the view that you can assume that nothing really exists and that's OK - but it's a fairly pointless thing to do. It doesn't advance your knowledge, further your career or make you happier - so you might as well assume that reality exists and carry on playing with it.
Physics certainly isn't complete. We don't know about the Higgs Boson for example - we've built this gargantuan machine to test for it - and (sadly) the machine is currently broken. But even in the realms of the "normal" world - there are unknown things. We're still having trouble figuring out WTF happens when you take a "Wint-O-Green" Life Saver into a very dark place and crush it with a pair of pliers causing it to emit a brief spark of light.
Quantum mechanics is more than just a hypothesis. It's extremely well tested. If quantum mechanics were just a theoretical possibility, the computer you are sitting at wouldn't work because the flash memory inside relies on quantum effects to do what it does. If quantum theory didn't work, we'd never have figured out how to make blue LED's.
SteveBaker (talk) 14:46, 15 May 2009 (UTC)[reply]

An illuminating answer, i must say, but still more questions. You talked about the chaos theory. Even though the effects are very hard to predict, it remains that the situation is , much like the tossing of a coin,in principle, intrinsically deterministic. So no problem with there. Reality. My question remains unanswered. Will the world be reduced to complex wave functions when i blink ? Does there or doesn't there exist things that exist whether or not i see it ? Is the electron already there, or does it choose to appear there as i have forced it to make a choice by observing it ? As Einstein said, does the moon exist only if i look at it ? And you said no free will. Which pretty much means it is already decided what choices we will take. Which pretty much means determinism. But doesn't the uncertainty principle forbid this ? If something can be anywhere now, how can it be at some particular point in the future ? I'm sorry if i sound aggressive, buut these debates have been raging in my mind for quite some time now, and i had to ask an expert.Rkr1991 (talk) 15:02, 15 May 2009 (UTC)[reply]

I think you're confused by the often-quoted point of quantum mechanics where "observing" a system changes it. I don't like this word "observed" because it implies that sentient beings have some sort of power over the laws of physics: this is not the case. "Observation" simply implies interacting with another wave/particle/wavicle, so that its position and momentum at one point in time is known (whether or not a human happens to be watching to record the evidence of that interaction).
The answer to your question is yes, as far as we know, QM is infallible. But this is easy to say, because we have defined that QM only "applies" for slow and light (think non-heavy, not photonic) objects. Through predictions, experiments, and results, scientists have narrowed down the cases where QM makes good predictions, and determined that when certain conditions are present QM spews nonsense. It's easy to say that a theory is "infallible" when you only apply it to special cases.
As far as your latest post, you ask "If something can be anywhere now, how can it be at some particular point in the future?" It isn't fair to say that just because a particle's wavefunction is non-zero everywhere that it can be "anywhere". The probability of me tunneling (think quantum, not creepy) through the earth to appear next to your computer is, technically, non-zero, but that is not going to happen.
Take all of this with a grain of salt: I am, at best, an advanced beginner at quantum physics. I'm sure SteveBaker will correct me if I'm wrong :-D -RunningOnBrains 15:50, 15 May 2009 (UTC)[reply]
Regarding the original question, the Science Desk is really not very well suited to resolve these sorts of doubts. You have to go read some books. Looie496 (talk) 16:04, 15 May 2009 (UTC)[reply]

I don't think Rkr1991 should be so quick to brush away chaos theory. The important point is that chaos theory acts as an uncertainty multiplier. The teeny-tiny uncertainty of whether the butterfly flaps it's wings now...or...(wait for it)...NOW! can make the difference between that hurricane happening or not. The chaotic nature of (for example) the weather magnifies the degree of uncertainty from something that really doesn't matter at all - to something really major. What that means is that chaos theory can magnify the literally, fundamentally, non-deterministic things that happen in the quantum world up to the macro scale where humans are affected by the results. We cannot - even in principle, with infinite computing power and perfect knowledge - usefully predict the weather more than maybe a month into the future. Taken together, quantum theory and chaos theory mean that even at the large scales where we like to think that the 'real world' consequences of quantum uncertainty is negligable - it's not. SteveBaker (talk) 17:49, 15 May 2009 (UTC)[reply]

I would like to emphasize the fact here that i am not talking about what is going to happen in practice , merely the principle. Practicality notwithstanding, since the wavefunction is non zero everywhere, the electron can indeed in principle be found everywhere. Coming to practical terms, ok, my electrons can never be found in the sun, but that doesn't mean you know where it is now. The fact lies that there is a finite measurable indeterminacy in position of the electron, and now i can say that "If something can be anywhere now, how can it be at some particular point in the future?" But you are right, i am confused a bit about that observing part... the problem is that most books are too abstract or complex for me to plough through... I just want to get the picture, the principle, the idea, not everything in its mathematical beast-like form...Rkr1991 (talk) 16:34, 15 May 2009 (UTC)[reply]

And if the universe is infinite (or even very VERY large) - then somewhere on some far distant planet - a half ton rock has indeed just jumped four feet to the left and turned into a mauve grand piano without any provocation. In principle, that could happen to you, right now. (Please let me know if it does...that would be fun to know!) It is only that the probability is so amazingly low that we may ignore it for all practical purposes...and even theoretical ones for that matter! However, it is only the quantum uncertainty of that electron "tunnelling" that enables your computer's flash memory to work...so at the level of the very small, this effect dominates the way the universe works. SteveBaker (talk) 17:49, 15 May 2009 (UTC)[reply]

Ok, I'm just going to take a shot at this, but what I think the world would look like when you close you're eyes is just atoms reacting with each other. So things that you can see when you have you're eyes open, like light, are just the photons of light hitting electrons and reflecting off into space, to be reflected off into whatever. But when you're eyes are open, you're brain and interpret these waves into the color spectrum and creates shapes and sizes. But then I think even there is still kind of low resolution, if you consider string theory. I don't know a lot about it, and correct me if I'm wrong, but tiny little vibrating strings make up everything, and some are looped into themselves while others are just wiggly lines. So if you could see the true "reality," then you'd see all these little tiny strings are that are vibrating and interacting with each other. 129.21.109.153 (talk) 17:12, 15 May 2009 (UTC)[reply]

I think the whole "blinking" or "not looking" thing is a bad interpretation. If you don't look at the Sun, is it still there? Well the tan/sunburn that you will get implies that it is. When your eyes are closed, you are just blocking light waves from reaching your retina, but I know you aren't looking at things from this perspective. You are asking whether "reality" exists if you are in a coma, for example. I guess that depends on what you define as reality, and this is more of a philosophical question than a scientific one. As for free will, I personally have no problem with accepting it doesn't exist, in fact, I feel like I have no control over my life (I realise this may just be a symptom of depersonalization disorder, but I don't fit other criteria), but I always notice how my biology affects me. If I'm in the shower and it starts to feel cold, I turn the heater up. If I'm cold I go and put my jacket on, If I see a question on Wikipedia which I believe I'm capable of producing a decent answer to, I reply to it. Etc. In terms of philosophy, I would be classified as a physicalist determinist. --Mark PEA (talk) 17:29, 15 May 2009 (UTC)[reply]

There was some recent discussion about a related topic on this very page. Make sure you read my coments as well as BenRG's. To avoid repetition I will only coment about free will here. Can anybody explain me why people equate determinism with lack of free will? That makes no sense to me. I think it is exactly the oposite. In order to have free will, we need a deterministic world otherwise people would go around doing random things instead of doing things they want to do. Dauto (talk) 19:44, 15 May 2009 (UTC)[reply]

Assuming free will refers to a conscious ability to make decisions, in a deterministic universe, all our decisions would have been determined (or determinable) before we were born. There's no space for free will there. Zain Ebrahim (talk) 08:00, 16 May 2009 (UTC)[reply]
I disagree. I think you confuse being predictable with having been determined. As I see it, determinism is essential in order to preserve true free will. Dauto (talk) 02:28, 19 May 2009 (UTC)[reply]
What are your definitions of free will and true free will? If free will is a conscious ability to excercise control over our decisions in spite of (or independently of) past events then a deterministic universe rules out free will. Btw, a random universe would rule out that kind of free will too because random decisions are not consciously made. Zain Ebrahim (talk) 07:22, 19 May 2009 (UTC)[reply]
What you're saying doesn't make sense to me. Living organisms do what they 'want'. What an organism wants to do is a calculation based upon the information they are receiving from their senses, along with their genetic code and upbringing (and diet, drugs, ...). As I posted in the previous example (with the shower and the jacket), decisions are made to try and maintain homeostasis, although some genetic code is better at maintaining homeostasis than others (e.g. those with the short allele of the 5-HTTLPR are more susceptible to commit suicide (Caspi et al. (2003). Science. 301(5631):386-389.)). This is confirmed with the needs of shelter, water, nutrition, warmth... but doesn't really explain reproduction. The explanation for the urge to reproduce is presumably due to DNA that makes us 'want' to reproduce, because without it, we wouldn't be having this conversation. --Mark PEA (talk) 22:20, 15 May 2009 (UTC)[reply]

You're a teenager and you've hit a mine of interesting thoughts. As people have said, the way you're thinking about this isn't necessarily very scientifically useful. For one thing, you seem to be mixing 'reality' up with 'your perception of reality'. But it is fun and interesting, particularly when all these ideas are new to you. Get hold of some Philip K. Dick short stories, read the Principia Discordia (although it will probably just seem silly to you), enjoy playing with these ideas. But remember that for all practical purposes, reality exists independently of you and you exercise free will. All the rest is mental entertainment (or seeking after ultimate truth. One or the other). 80.41.104.220 (talk) 20:01, 15 May 2009 (UTC)[reply]


Quantum theory provides us with a striking illustration of the fact that we can fully understand a connection though we can only speak of it in images and parables.
However the development proceeds in detail, the path so far traced by the quantum theory indicates that an understanding of those still unclarified features of atomic physics can only be acquired by foregoing visualization and objectification to an extent greater than that customary hitherto.
We have to remember that what we observe is not nature herself, but nature exposed to our method of questioning.
The existing scientific concepts cover always only a very limited part of reality, and the other part that has not yet been understood is infinite.
Whenever we proceed from the known into the unknown we may hope to understand, but we may have to learn at the same time a new meaning of the word "understanding."
Any concepts or words which have been formed in the past through the interplay between the world and ourselves are not really sharply defined with respect to their meaning: that is to say, we do not know exactly how far they will help us in finding our way in the world. We often know that they can be applied to a wide range of inner or outer experience, but we practically never know precisely the limits of their applicability. This is true even of the simplest and most general concepts like "existence" and "space and time". Therefore, it will never be possible by pure reason to arrive at some absolute truth.

The above are selected quotes from Werner Heisenberg, one of the founders of quantum mechanics. Cuddlyable3 (talk) 20:36, 15 May 2009 (UTC)[reply]

If you've managed to wrap your head around the above, here's another bummer: Have you noticed just how much "nothing" is involved in solid matter? It still amazes me how it all works together so well that you can actually have solid objects and people instead of just random particle soup :-)71.236.24.129 (talk) 22:11, 15 May 2009 (UTC)[reply]
That is because physical science is learning more and more about what turns out to be less and less. Cuddlyable3 (talk) 10:26, 16 May 2009 (UTC)[reply]

Well, i am not a professional in this, so correct me if i am wrong. Dauto had asked why i equate determinism with lack of free will. From what i understand, Free will is taking decisions ourselves, when we have a choice, even if it is obvious, it is we who will choose. Sometimes they maybe be obvious, sometimes they may not, like coming to a fork in the road and not knowing which way to go. But the fact remains that we make the decision. Now if everything is already predetermined, then it is already known, perhaps to god, what decision we would take, say the left road in the fork (see : i made a decision here ). So if god already knew it, i was just destined to type lift road here and not anything else... Spooky : since i was destined to choose the left road, did i really have a choice ? Did i really exercise my free will ? Or in other words, in a deterministic world, does free will exist ? This is something like not punishing a criminal because he was destined to commit the crime... he couldn't have done otherwise... Note that this problem wouldn't arise in an indeterministic situation  : If even GOD didn't know what choice i would take, then it was entirely upto my mind, and me, of my own free will, chose left. I may have made a mistake somewhere, if so please excuse me... Rkr1991 (talk) 04:35, 16 May 2009 (UTC)[reply]

In terms of the punishing criminals thing: If the criminal is predetermined to commit crime, society is predetermined to lock them away. If someone punched me in the face and said "Sorry, I have no free will", then I will just punch them back and say "Me neither". What it boils down to in the end is a question of fairness. Is it fair that a child gets abused and then grows up to be a criminal, and then locked away, etc. when if they had grown up in a caring family with adequate wealth they would be going on to earn good money in a good job and live outside of prison? That is just nature I guess. --Mark PEA (talk) 09:36, 16 May 2009 (UTC)[reply]
Determinism contra free will is a contradiction that is impossible to resolve by logic. However the consequence of each choice is not experienced until after the choice is made. The resulting subjective development may be surprise, guilt, pleasure. learning or something else. Whether you regard that as predetermined or not, you cannot know it in advance. Wondering about whether something unknown might matter prompted Donald Rumsfeld's dictum Now what is the message there? The message is that there are known "knowns." There are things we know that we know. There are known unknowns. That is to say there are things that we now know we don't know. But there are also unknown unknowns. There are things we do not know we don't know. So when we do the best we can and we pull all this information together, and we then say well that's basically what we see as the situation, that is really only the known knowns and the known unknowns. And each year, we discover a few more of those unknown unknowns.Cuddlyable3 (talk) 10:22, 16 May 2009 (UTC)[reply]
There is another theory about determinism that doesn't really involve god that no one has mentioned yet. If you look at everything at the quantum level then it's all random. There is no way to predict what an electron will do, and this has been going on since the big bang, and probably before that. So what if our "free will" is just the random playing out of quantum reactions? This theory includes not just our actions and choices but thoughts as well (even thoughts about god). Interesting, right?129.21.109.153 (talk) 19:13, 16 May 2009 (UTC)[reply]


Ya, i agree with the above answer. That does present an interesting possibility. In fact that is what i had in mind while asking this questionRkr1991 (talk) 06:02, 17 May 2009 (UTC)[reply]

I think watching (most of) these movies could help your quest somewhat. They'll make you think, but also will entertain (except Signs and Unbreakable which aren't so good, and Walkabout which I haven't seen. The Tarantino films don't answer much either... --Jackofclubs (talk) 20:06, 16 May 2009 (UTC)[reply]



Mark PEA's answer may explain why criminals are punished, but it still doesn't explain if there is free will or not, which is what the question is all about. Rkr1991 (talk) 06:03, 17 May 2009 (UTC)[reply]

I need your definition of free will. If it is absolutely free, it would imply that it is random, unpredictable and unconstrained. If this were the case, (nearly) every field of science with the prefix "neuro" and "psycho" would be obsolete. Studying the behaviour of people would be as about as productive as studying the behaviour of a (non-fixed) roulette wheel. I think practically everyone accepts that free will isn't "free", rather "constrained". What I'm proposing is that it is so constrained that there isn't any choice at all. I'm going to pose a quick (probably not well thought out) thought experiment:

In one condition of the experiment, you are in a room sat on a chair, wearing a blindfold and earmuffs, blocking your vision and hearing. You have also had local anaesthetic injected into your wrists 5 minutes prior to sitting, preventing pain (but not touch) from being received from your hands. You are asked to extend your hands out as far as you can, and they will touch some flat-feeling objects (you haven't seen these objects). What is your reaction after touching the objects? In the second condition, everything is the same to you, except this time no local anaesthetic is injected. When you extend your hands and touch the objects, you realise they are extremely hot (probably irons). What is your reaction?

The point of this thought experiment is to show you how your behaviour is completely controlled by the stimuli you receive, and there is no choice involved. I realise I'm going to be picked apart by many because this situation involves a reflex arc, but in my view it is just wishful thinking to believe that when a signal is calculated in certain areas of the brain (composed of neurons) "free will" is involved as opposed to other areas of the brain or other areas of the central nervous system (both also composed of neurons). What is so different about the neurons in the brain stem (heart rate/etc.)/hypothalamus (hunger/sleep/etc.) compared to those in the prefrontal cortex (personality)? If you believe in some sort of spirit, how does a lobotomy change this spirit? If you believe in a spirit or some quantum indeterminacy hypothesis for free will, how do psychoactive drugs change this free will? As I mentioned earlier (with refs), how does having the short allele of the 5-HTTLPR make one's free will more susceptible to neuropsychiatric disorders? Am I just being a blind confirmistic determinist? Because I can't see how all these changes in molecular chemistry are just very large coincidences and that subatomic indeterminism is the actual cause (not including lobotomy, which doesn't affect subatomic hypotheses for free will, only spiritual).
On another note, an interesting study involving conscious intention was published in Science just over a week ago [11] (and discussed on NewScientist [12]). I fully recommend getting the entire article (your school/college should have access to Science, IMO anyway). I can't really sum it all up, but basically, stimulation of one area of the brain caused movement of a limb that the participant was completely unaware of, where as stimulation of another area of the brain in small amplitude caused an urge to move a limb/lips/etc, and higher amplitude stimulation caused the participants to believe they had actually moved a limb/talked/etc, when they hadn't. This implies that one area of the brain is involved in conscious intention, and if activity is high enough, believes that those intentions were carried out, and these areas of the brain are actually naturally innervated by subconscious areas of the brain which cause the movement to occur. But when stimulated individually artificially, there is some mis-innervation which - depending on the area of the brain - either causes the conscious perception of having done something but actually not done it, or the actual doing something but no conscious perception of it. Sorry that isn't eloquently put, but I can't copy and paste the conclusions of the article due to copyright reasons.
I've just seen that in the same issue there was another piece of research about rhesus monkeys' decision making based upon their confidence in the correctness of that decision (which was also due to neurons in the parietal cortex - [13]), I've only read the abstract though so I'll post on here if there is anything very relevant to this discussion after I've read the full article. --Mark PEA (talk) 14:42, 17 May 2009 (UTC)[reply]

Left/right and relativity

Does the left and right side undermine the relativity theory in a sense that there is an absolute benchmark - the possessor of these extremities? If the arms for example are considered left and right from the benchmark of one's own body, does it turn non-relativist, absolute? 91.135.250.34 (talk) 13:44, 15 May 2009 (UTC)[reply]

I don't think your understanding of relativity is quite correct. In general relativity, there is still a coordinate system, which means that relative positions can and do exist. The difference in relative position (how far apart your arms are) depends on who is observing and how fast they are moving and accelerating. But, under no circumstances would they "flip sides", because they are separated by a space-like interval. Maybe you heard some discussion about chirality and mixed up some terminology? Nimur (talk) 14:01, 15 May 2009 (UTC)[reply]

Identify this military equipment

Can anyone identify this thing? This image was posted on DefenseLink with the caption,

I'd like to put this image in Wikipedia on the appropriate article, but I'm not sure what it is. (Maybe a reflecting telescope?) Nimur (talk) 13:45, 15 May 2009 (UTC)[reply]

I think it's more likely a BGM-71 TOW launcher along with associated aiming mechanism. See this HMMWV-based launcher for comparison. — Lomn 14:04, 15 May 2009 (UTC)[reply]
There is a "71" on the tube, so I'd call that a solid ID. Further research indicates it's probably the M220 variant of the BGM-71. Nimur (talk) 14:18, 15 May 2009 (UTC)[reply]
Since you mention checking markings on the tube, the stenciling below the eyepiece suggests it's the TOW 2B top-down attack warhead, so the BGM-71F designation looks like the most specific for the missile itself. — Lomn 15:13, 15 May 2009 (UTC)[reply]

calculation

although the title suggests a mathematical problem, it's acutally chemistry

what is the molecular mass of a substance each molecule of which contains 9 atoms of carbon, 13 atoms of hydrogen and 2.33 * 10-23 grams of other component?

that's 108 grams of carbon and 13 grams of hydrogen. and adding them up, i get 121 grams. but how am i gonna add 2.33 * 10-23 grams?????!!! —Preceding unsigned comment added by 122.50.132.126 (talk) 13:51, 15 May 2009 (UTC)[reply]

Well, you have made a silly mistake... See, an atom of carbon doesn't weigh 1 gram. 1 mole, or 6.023X10^23 atoms weigh 12 grams. Now i think you'll do the problem...Rkr1991 (talk) 13:55, 15 May 2009 (UTC)[reply]

I expect you want the molecular mass in Daltons or "atomic mass units", not grammes. So its 108 Da from carbon and 13 Da from hydrogen. Grammes is related to Da through Avagadro's constant, so you don't need to know what element it is, nor do anything with moles in this case.YobMod 13:56, 15 May 2009 (UTC)[reply]

Biomass

Which mammal species has the largest mass (number of individuals x average weight) on earth? Is it people? 65.121.141.34 (talk) 14:11, 15 May 2009 (UTC)[reply]

A table at Biomass (ecology) says cattle outweigh humans, for one thing. Tempshill (talk) 15:38, 15 May 2009 (UTC)[reply]
Very helpful, thanks. 65.121.141.34 (talk) 16:08, 15 May 2009 (UTC)[reply]
At one extreme, you might think of the largest animals on earth - the Blue Whale. At 172,000kg each - and with only 12,000 of them left - they total about 2 billion kg. There are about 7 billion humans and at perhaps 80kg each, we reach about 560 billion kg. Even before we started on the mass extinction of the blue whales, there were only about 300,000 of them - so 53 billion kg is the best they ever managed. So perhaps we should be looking at things that are tiny but much more numerous than us. It is estimated that there are 1,000,000,000,000,000 ants in the world - but at 0.003 grams each - that only gets us up to 3 billion kg...more than the blue whales - but nowhere near as much as all the whale species taken together (and I did cheat and take all of the ant species together). So if anything is going to beat out humans, it's got to be something in the middle range - much more numerous than whales but much bigger than ants. And that's why cattle win. SteveBaker (talk) 17:23, 15 May 2009 (UTC)[reply]
Minor detail - last time I checked, ants didn't feed their young on milk... --Tango (talk) 17:41, 15 May 2009 (UTC)[reply]
I read in The Book of General Ignorance that if you take a field of cows and weigh all the worms in the field, the worms will outweigh the cows considerably. 90.193.232.41 (talk) 17:45, 15 May 2009 (UTC)[reply]
Our article Earthworm says "Darwin estimated that arable land contains up to 53,000 worms per acre (13/m²), but more recent research from Rothamsted Experimental Station has produced figures suggesting that even poor soil may support 250,000/acre (62/m²), whilst rich fertile farmland may have up to 1,750,000/acre (432/m²), meaning that the weight of earthworms beneath the farmer's soil could be greater than that of his livestock upon its surface." - but a lot of cattle are not kept on rich, fertile farmland. Here in the western world, they are mostly cooped up in gigantic factories - and out in the 3rd world, they are on barely-grazeable wasteland. In either case, the number of worms will be dramatically lower. Weighing in at perhaps a gram or two each, 1.7 million of them might constitute 2 or 3 metric tons of biomass. A cow weighs around 800kg - so if there are less than 3 or 4 cows to the acre then maybe this claim is true. Worldwide - there are about 1.7 billon cattle - each one weighs about 10 times more than a human - so even though there are around 7 billion of us - the cattle win the biomass contest by a factor of two. The worms though...well, if we go with the 250,000 to the acre for 'poor soil' and 4x1010 acres of 'arable' land in the world - then we have 1016 worms - which (at 1 to 2 grams each) is 1 to 2 x 1013 kg. Which beats out the weight of cattle at 1x1012...by a factor of 10.
So I don't know - worms or cows? Sadly, our OP asked about SPECIES - and while there is only really one species of domesticated cattle (they can all interbreed AFAIK) - there are dozens and dozens (or maybe hundreds) of species of earthworm. So while the total weight of all worms beats out the cattle - I think the cattle still win on a species-by-species contest.
SteveBaker (talk) 18:18, 15 May 2009 (UTC)[reply]
Well that and worms are not mammals. Otherwise some termite species would probably be a major possibility 65.121.141.34 (talk) 18:40, 15 May 2009 (UTC)[reply]

So small mammals perhaps? Are there any estimates of the world population of rats, or mice? 65.121.141.34 (talk) 18:07, 15 May 2009 (UTC)[reply]

Hmmm - good point. Well, rats weigh about half a kilo - so for every cow, you'd need to have more than 1600 rats...about three trillion of them world-wide...but again, to meet the OP's requirements, they have to be all of one species - and there are about 50 species of rat...so while I can't find a number for their population size - I'd be surprised if they won. http://www.snopes.com/critters/wild/rats.asp says that there are far fewer than one rat per person in the UK - and about one per 36 people in even the most rat-infested cities. Since we're looking for more like 500 rats per person...all of one species - it seems very unlikely. Mice, even less likely still because they are so much lighter. SteveBaker (talk) 18:29, 15 May 2009 (UTC)[reply]
If you want to be picky you'd have to sort out what cells are actually "human" or "cow" and what are independent organisms. I lost the figures in my files, but a significant portion of you walking around are actually bacteria, fungi and other separate entities. It may even out in the end, though, because you'd have to do that for all the qualifying mammals. 71.236.24.129 (talk) 22:31, 15 May 2009 (UTC)[reply]
That's a point worth considering. I remember a statistic that only 10% of the cells in a typical human body are human, although I'm not sure how reliable that is and I'm not it would be as drastic if you considered it by mass rather than by number of cells. If I had to guess, though, I'd say all mammals will have roughly the same proportion, so it shouldn't matter. --Tango (talk) 23:49, 15 May 2009 (UTC)[reply]

Rabbits breed like, well, rabbits - they're mammals and they're can be found pretty widely across the globe. Contender perhaps? ny156uk (talk) 23:15, 15 May 2009 (UTC)[reply]

Or Sheep? From the article it suggest the global sheep stock is 1.059.8bn. If we say 45kg (again from the article as low point for Ewes) that would be well about 45bn kg... oh well - don't you just hate it when you realise what you've gone and estimated ends up proving yourself 'wrong'? Seems that Sheep ain't nearly enough to compare with the human estimates noted above :-( ny156uk (talk) 23:21, 15 May 2009 (UTC)[reply]

http://faostat.fao.org/site/339/default.aspx might be worth a look around. Sorry should have done all this as one answer rather than the 3 half-assed answers! ny156uk (talk) 23:23, 15 May 2009 (UTC)[reply]

EARTH

Is Earth definitely symmetrical about the axis passing through through the Northern and southern most points??? Or Is it deformed ??? —Preceding unsigned comment added by 59.165.84.9 (talk) 15:21, 15 May 2009 (UTC)[reply]

Shape of the Earth. This article is a bit technical, but it's easy to answer your own question when you think about this question: how many Mount Everests are there?-RunningOnBrains 15:56, 15 May 2009 (UTC)[reply]
See also Land hemisphere. —Tamfang (talk) 05:32, 16 May 2009 (UTC)[reply]
It is very close to be symmetrical about that axis, but not perfectly so (due to oceans, continents, hills, valleys, etc.). Around other axes, there isn't rotational symmetry because of an equatorial bulge caused by the rotation. --Tango (talk) 17:38, 15 May 2009 (UTC)[reply]
I was surprised to read about the following asymmetry today: if and when the West Antarctic Ice Sheet melts into the ocean, it will have a more pronounced sea-level-rise effect on the northern hemisphere because it's so massive that water is bunched up near it due to its own gravitational pull! [14] --Sean 18:05, 15 May 2009 (UTC)[reply]
Actually, it's pretty darned close to spherical. There was a discussion last week or so (too lazy to dig it out of the archives myself) which brought up the fact that the variations in heights on the earth surface are smaller than the tolerances allowed for regulation billiard balls, and the "equatorial bulge" which makes the earth's equatorial diameter larger than its axial diameter is also smaller than allowable variations in regulation billiard balls. Basically, the earth is actually smoother and rounder than a billiard ball. The variations look huge when placed on human scales, but when looking at the earth as a whole, they are quite tiny. Take Everest for example. It is 8.848 kilometers tall. The earth is about 6,371 km in diameter. Thus, the biggest bump on the earth represents 0.139% variation from the mean diameter. I would call that pretty darned smooth. Also, the variation between the polar and equatorial diameters is 21.3 kilometers, or 0.334% variation of the mean diameter. Again, pretty darned close to spherical. --Jayron32.talk.contribs 01:09, 16 May 2009 (UTC)[reply]
That's Quite Interesting! And I was thinking it all went pear-shaped about 6 billion years ago.--80.3.133.3 (talk) 08:25, 16 May 2009 (UTC)[reply]
I may be missing a joke here, but the Earth is only about 4.5 billion years old. --Tango (talk) 16:47, 16 May 2009 (UTC)[reply]
Our UK brethern use Pear-shaped to mean "gone awry." -Arch dude (talk) 02:32, 17 May 2009 (UTC)[reply]
What would our friends across the water say, then? 'All gone eggplant shaped'?--KageTora - (영호 (影虎)) (talk) 04:43, 17 May 2009 (UTC)[reply]
I am British, so I got that part. It was the 6 billion years part I couldn't understand. --Tango (talk) 14:31, 17 May 2009 (UTC)[reply]

units

i'm very confused about this question, not only about the procedure but about the units also.

if the velocity of light is taken as the unit of velocity and an year is taken as the unit of time, what's the unit of length? what's it called?

should i do: length = velocity of light * 1 year

and then subsitute them with their values in m/s and days, or something else....

please help me

thanx —Preceding unsigned comment added by 122.50.132.126 (talk) 15:40, 15 May 2009 (UTC)[reply]

Take a look the article Light-year and come back if you have additional questions. --Zerozal (talk) 15:57, 15 May 2009 (UTC)[reply]
... and for extra credit, ask your teacher whether their "year" is a Julian year, a sidereal year or a tropical year (but only once you understand the difference between those terms yourself). Gandalf61 (talk) 16:05, 15 May 2009 (UTC)[reply]
Dimensional analysis and the related factor-label method are concepts which are important to be familiar with (at least for science-inclined types). -- 128.104.112.117 (talk) 16:09, 15 May 2009 (UTC)[reply]

May 16

Dryer sheets

So I have some uniforms for work. On the laundry label it says, in big bold letters, "DO NOT USE DRYER SHEETS." I'm just curious, what's the big deal? Can dryer sheets ruin the clothes somehow? They seem so harmless. TravisAF (talk) 03:28, 16 May 2009 (UTC)[reply]

What line of business are you in? Maybe there is some special reason? If (for example) you were in the medical field - perhaps some patients are allergic to things on dryer sheets...I have no idea whether that's true - but it's a thought at least. SteveBaker (talk) 03:31, 16 May 2009 (UTC)[reply]
Hmmm TravisAF's user page says he's in the airforce. Airforce uniforms and dryer sheets? SteveBaker (talk) 03:33, 16 May 2009 (UTC)[reply]
From what I've been told, dryer sheets affect the absorption of cloth like towels. I doubt that applies here though... Dismas|(talk) 03:43, 16 May 2009 (UTC)[reply]
Google says "..not using fabric softeners or dryer sheets since chemicals from these could clog the pores of the uniform's polyester material and adversely affect performance."[15] and "Evidently the PTU is a special moisture resistant fabric with evaporative properties that is ruined by both liquid fabric softener and dryer sheets."[16] in the specific case of US armed forces PT uniforms. Nanonic (talk) 04:01, 16 May 2009 (UTC)[reply]
Microfiber clothes and cleaning cloths sold in Germany bear a warning label not to use fabric softener to avoid ruining them. I've never seen that on any microfiber materials sold in the States. I was wondering whether the European fabric softener was different or whether they just didn't expect Americans to follow such labels anyway, so why bother? 71.236.24.129 (talk) 11:18, 16 May 2009 (UTC)[reply]

Digestion

Some people claim that mans digestive system is unsuitable for digestion of flesh of the other animals, but neither can we digest uncooked plant products!!Like we cook meat before consumption we cook even vegetables..We cook most of the natural products.Is meat which is cooked also hard to digest??? —Preceding unsigned comment added by 59.165.84.9 (talk) 05:29, 16 May 2009 (UTC)[reply]

The enzymes in your stomach primarily break down meat while other biomolecules are broken down in the rest of the gastrointestinal tract. Humans can indeed digest uncooked plant products, such as bananas. JameKelly (talk) 07:44, 16 May 2009 (UTC)[reply]
I'm also intensiely curious about this subject. Many vegetarians - the ones who take the whole thing really seriously - says that the human digestive system doesn't digest meat properly, and that it basically sits in your gut and ... rots. Is there any truth to any claim that eating meat is really that bad for you? The only real facts I know about it (which may or may not be right =p) are that you can't get vitamin B12 from any natural source other than meat, and that the unhealthiest natural fats are also found only in meat... 90.193.232.41 (talk) 08:41, 16 May 2009 (UTC)[reply]
Partly they're playing word games. Digestion and rotting both describe the Decomposition of orgainc material. When the Gut flora breaks down long chained fats some toxic substances can get produced. I have no data on whether that only happens if the digestive enzymes haven't broken down the chains sufficiently or whether that always happens. The argument loses a lot of momentum if you look at food chemistry. There are plenty of toxic chemicals in vegetarian foods (Cyanide came up quite often, glycoalkaloids are in potatoes and cinnamon contains coumarin, to name just a few.) Problems arise when petri dish results are transferred to complex real life systems. Evidence based medicine often fails to come to the same conclusions as theoretical models based on studying individual components. Dietary supplements are often not taken up as efficiently as the real nutrients. (google a study for calcium from cheese vs, pills for an example. Don't know of a B12 study.) I have severe doubts that a diet that has been proven to be deficient in at least one essential nutrient can be healthier than a good mixed diet. The thing is we're not eating that either. Some meat is consumed dried rather than cooked. See e.g. Pemmican Jerky (food) For raw meat see Steak tartare. Countless fruits, nuts and vegetables are consumed raw. (Think of all the things you could put in a salad bowl)71.236.24.129 (talk) 11:02, 16 May 2009 (UTC)[reply]
Cooking meat does assist digestion, but it's not essential. A much more important concern is infectious disease. The modern processing, storage, refrigeration and handling of meat may lead to harmful bacteria growing in the meat. If the meat isn't cooked properly, this can lead to food poisoning. Axl ¤ [Talk] 11:24, 16 May 2009 (UTC)[reply]
Although improper handling of vegetables has also caused a lot of sickness as well, both bacterial (E. Coli for example) and viruses (Hepatitis A, from recent memory). Improper handling of food is bad no matter what it is. -- JSBillings 13:03, 16 May 2009 (UTC)[reply]


I'll just come out and say it: it is scientifically incorrect to say we are not able to digest meat well. Actually, it's totally wrong and out of touch with reality. We are actually very well designed for meat consumption: our stomach is full of gastric acid specifically tuned to the right pH to catalyze protease and break down protein found in meat. In fact, we are more specifically tuned for digesting meat than we are for cellulose, which must pass out as roughage. Although we are omnivores, we have more structure and metabolism in common with carnivores than with herbivores. Take a look at human canine teeth and incisors. These are not designed for chewing leaves and roots. (We have molars for that, but unlike an herbivore, we do not replace our molars every year or so). Anybody who wants to make the claim that we are biologically "designed" for vegetarian diets should reconsider their facts. Humans are omnivores. In a modern industrial society, we are able to provide sufficient nutrition with a wide variety of alternative diets (including vegetarianism, which may have some health benefits), but we did not evolve "naturally" to be herbivores. Nimur (talk) 16:14, 16 May 2009 (UTC)[reply]

Bird identification

Garden Warbler?

I asked on the Commons help desk if there was somewhere I could check a bird identification (to avoid misclassifying a photo) and was referred here. If this is the right place to ask, could someone tell me if the identification is correct? If not, is there somewhere else I can ask? N p holmes (talk) 09:00, 16 May 2009 (UTC)[reply]

I agree with your ID as a Garden Warbler - I looked at it in the Collins Bird Guide. It's the grey patches at the side of the head that clinch the ID. Wood and Willow Warblers are yellower, as is the Chiffchaff. The only real way to absolutely clinch the ID is to hear it, though: the Garden Warbler has the most wonderful song of the summer. --TammyMoet (talk) 09:22, 16 May 2009 (UTC)[reply]
Thanks. I did hear it (a fast song with interspersed slightly longer notes): not a Willow Warbler or Chiffchaff, but I've never heard a Garden Warbler. N p holmes (talk) 11:01, 16 May 2009 (UTC)[reply]

Suicide Bridge

Isn't it possible to survive a jump into the water from a suicide bridge? Assuming you don't bellyflop.68.148.149.184 (talk) 09:14, 16 May 2009 (UTC)[reply]

If you look at Drag (physics) The force depends on the density of the medium. Compare 1.204 for air to 1000 for water from our article. A person Diving off a board that is 10 m (about 11 yd) high will hit the water at almost 50 km/h (31 mph). They will try to achieve a mostly aerodynamic form when they enter the water and try to minimize the area of impact. They will also most likely not choose their head as the first point of impact, but try to create flow around their hands and arms first (also see La Quebrada Cliff Divers). A person jumping off one of those bridges would hit the water at a higher speed (maybe not quite terminal velocity) and will likely present a lot of vulnerable surface area to the impact. Even if they don't break their neck, their skin might break on impact and they'd bleed to death because blood doesn't clot effectively in water. 71.236.24.129 (talk) 10:10, 16 May 2009 (UTC)[reply]
The world record for the highest dive stands at over 50m (see Dana Kunze) whereas the mid-span clearance below the Sydney Harbour Bridge, for exampe, is 49m - so, yes, such a jump into water is theoretically survivable, but only with a high level of fitness and training. Gandalf61 (talk) 10:59, 16 May 2009 (UTC)[reply]
Presumably that record breaking diver survived the dive with no, or insignificant, injuries. If you land feet first from a pretty high bridge you can easily break your legs but otherwise survive as long as someone drags you out quickly enough and you get medical attention. I'm not sure what the highest survivable dive would be, but it has to be more than 50m. --Tango (talk) 13:32, 16 May 2009 (UTC)[reply]
There are MANY people who have survived falls from airplanes without a parachute ([17] for example) - landing in bushes or whatever. I'm sure some of those ended up in water. (See also Free-fall#Surviving_falls) So it's definitely survivable - even at terminal velocity - but it's definitely not certain - or even likely - so it's not recommended. There are more interesting cases in Category:Fall survivors - and some useful data on fall survivability in Alcides Moreno. SteveBaker (talk) 16:03, 16 May 2009 (UTC)[reply]
No need to consider hypotheticals or falls from airplanes, there are plenty of documented cases of people surviving actual bridge suicide attempts. Golden Gate Bridge says "As of 2006 only 26 people are known to have survived the jump", but also "Some of those who survive the impact, drown or die of hypothermia in the cold water". So depending on how they counted, at least 26 and maybe more survived the fall (out of 1200 or more). -- BenRG (talk) 21:02, 16 May 2009 (UTC)[reply]
I read an article once, sorry I have no idea where, claiming that of all the survivors of Golden Gate Bridge jumps, 100% of them had landed feet-first, and, it's claimed, leaning slightly backwards. Tempshill (talk) 03:27, 17 May 2009 (UTC)[reply]
On Mythbusters there was an episode where they tested what it would be like to fall from the Golden Gate Bridge, but having a hammer or something fall below you to break the surface of the water just before you hit. They said there was a myth that this could save your life. The crash test dummy, Buster, was damaged in just the same way as when they did a control without the hammer. In fact, one of its legs came off. It's unlikely to survive such a fall, but, as posters have said above, it is possible. On a side note, There is a terminal velocity, so after a certain point it really doesn't matter how high you fall from, because you'll still be going the same speed (slightly simplified explanation).--KageTora - (영호 (影虎)) (talk) 02:29, 18 May 2009 (UTC)[reply]
I read (in one of those worst-case scenario books, I think; don't know the reliability) that a common source of injury when going in feet-first is a jet of water shooting up your bung hole and rupturing your colon. "Puckering saved my life". --Sean 13:34, 18 May 2009 (UTC)[reply]
It's interesting to note that there is a "worst height" from which to drop a cat. If you drop them from higher, they will react during the fall and spread out their bodies to slow their terminal velocity. There are wacky cases of cats falling from tall buildings with minimal injuries. Dcoetzee 03:04, 19 May 2009 (UTC)[reply]

Age and energy

Why do people above 16 seem to have less energy than those below 16? Thank you. Clover345 (talk) 10:24, 16 May 2009 (UTC)[reply]

After 16 we save our energy for things that under-16s in the old days weren't supposed to know about. Cuddlyable3 (talk) 10:37, 16 May 2009 (UTC)[reply]
Er no, I don't think that's a very good answer. —Preceding unsigned comment added by 82.44.54.169 (talk) 11:11, 16 May 2009 (UTC)[reply]
What do you mean, energy? Olympic athletes are usually not under sixteen. Similarly for Artic explorers who pull sledges to the poles. Perhaps you are thinking about under 16s being less inhibited with moving their bodies around than older people are, such as break-dancers. 78.146.190.197 (talk) 13:32, 16 May 2009 (UTC)[reply]
I think simple physics explains this. Assume your "under 16" kid is half your adult height - that means that they have something like one-eighth of your weight and their muscles have one quarter the cross-sectional area. The power produced by a muscle is proportional to it's cross-sectional area - and the amount of work it has to do to (say) have the kid run around yelling something annoying over and over again - is proportional to the kid's mass. So their power-to-weight ratio is twice that of an adult that is twice their size. So I'd conclude that while they might SEEM to have twice the "energy" you have - in reality, they are really able to produce twice the acceleration of one eighth the mass. Obviously as your size approaches that of an adult - this capability gets less and less pronounced. Once you are 90% of the size of an adult (perhaps) at age 16, you have pretty much reached that all-time-low. SteveBaker (talk) 15:54, 16 May 2009 (UTC)[reply]
It won't be 1/8 the mass. Humans don't remain in the same proportions are they grow, that's why Body Mass Index is calculated with the square of height, not the cube. Imagine we have a median BMI 4 year old that is 90cm tall and a median BMI adult that is 180cm tall. According to the graphs on our BMI article, the child should have a mass of 13kg and adult should have a mass of 71kg. That's about 1/5.5, quite a bit more than 1/8. I don't know how cross-sectional area of muscles actually varies with height and age, but I doubt it is as simple as for the spherical cow you are assuming. --Tango (talk) 18:50, 16 May 2009 (UTC)[reply]
Well, duh - obviously it's not that simple - but whatever is? The point is that small animals have more 'bounce' than big ones...it's a good explanation - despite all of the obvious approximation. SteveBaker (talk) 03:13, 17 May 2009 (UTC)[reply]
I'm not convinced that you get an at all meaningful answer after all that approximation. While there may be an increase in apparent energy due to this I suspect that it is negligible. Humans are nothing like spherical cows, not even approximately. --Tango (talk) 14:30, 17 May 2009 (UTC)[reply]
The relevent Wikipedia article here is Senescence, which is the study of the process of aging. There's lots of good links from that article to take you where you want to go. --Jayron32.talk.contribs 19:58, 16 May 2009 (UTC)[reply]
It doesn't apply very well to the difference in 'energy level' of (say) a 12 year old compared to (say) a 20 year old - which is the kind of thing we're being asked for here. SteveBaker (talk) 03:13, 17 May 2009 (UTC)[reply]
For a biological answer, there is evidence that "energy" correlates with the level of dopamine released in the brain -- this is how amphetamine and cocaine exert their energy-enhancing effects. There is also, I have read, evidence that dopamine release is highest in children, and drops steadily with age. Very low levels of dopamine produce Parkinson's disease, which is characterized by torpor. Looie496 (talk) 19:41, 17 May 2009 (UTC)[reply]
What is known about the causes of Parkinson's disease. I struck the irresponsible claim above. Cuddlyable3 (talk) 21:03, 18 May 2009 (UTC)[reply]
The sentence you struck was correct, see Parkinson's disease#Pathophysiology. Parkinson's disease is the best understood of brain diseases in the sense that we know exactly what the damage is: loss of dopamine cells. Unfortunately, we don't have a very good understanding of what causes the dopamine cells to be lost. Looie496 (talk) 21:47, 18 May 2009 (UTC)[reply]

Swans - can they break your arm?

I have often heard that a swan can break your arm. There isn't anything on the swan article about this - is this true or is it an urban myth. Also should there be something in the swan article about this? Catoutofthebag (talk) 11:09, 16 May 2009 (UTC) Additional question Also are there any recorded incidents of people having their arms broken by swans? Catoutofthebag (talk) 11:42, 16 May 2009 (UTC)[reply]

Whooper swan says they can be up to 15 kgs. That means their wings can produce enough lift (force) to move those 15 kgs. Think of hitting something with a 15 kg Mallet. People who handle swans learn how to disable their wings when grabbing them, from experienced swan handlers. (Only example I know is from Hamburg, Germany de:Eppendorfer Mühlenteich.) —Preceding unsigned comment added by 71.236.24.129 (talk) 11:38, 16 May 2009 (UTC)[reply]
The "mallet" would have a large wing attached to it, which would slow it down. And being kicked by a 200lb person is probably not going to break a limb either. It is unlikely that Swans move their wings fast enough to provide 15kg of lift when threatening people, as they would either take-off or fall over if the wings were flapped horizontally. The swan handlers would be worried about the swans breaking their wings, as with handling other birds. 78.146.190.197 (talk) 14:37, 16 May 2009 (UTC)[reply]
It's a myth.--80.3.133.3 (talk) 12:56, 16 May 2009 (UTC)[reply]
Swans in the UK are in theory owned by the Queen. For hundreds of years they have been owned by royalty, and I believe were eaten by them. I understand that the sawns-can-break-your-leg myth was just put about to stop the peasants from eating them. 78.146.190.197 (talk) 13:26, 16 May 2009 (UTC)[reply]
On that site it says "From Swanuk.org: "Yes, but only in exceptional cases. If a wing in full span and velocity were to hit a weak-boned person (such as a child or an elderly person) then it is theoretically possible. In reality it is almost unheard of and is never used as a form of attack as swans are a defensive bird. The only time they become aggressive is when they are protecting their nesting ground or cygnets when they will chase off intruders, be they other swans, geese or humans who get too close." So is the answer a yes or a no? Or a theoretical yes. Catoutofthebag (talk) 13:42, 16 May 2009 (UTC)[reply]
Parsing the quoted text above indicates that the answer is "no" in almost all circumstances. The answer could only be "yes" in theory for a child or elderly person. 78.146.190.197 (talk) 13:48, 16 May 2009 (UTC)[reply]
Anecdote time. I used to share a house with a guy who slaughtered domestic geese for a living (several years before). He told me that a goose had once broken his wrist with a strike from its wing as he was trying to restrain it. He did have some sort of bone condition though... --Kurt Shaped Box (talk) 15:14, 16 May 2009 (UTC)[reply]
OR: My Thanksgiving turkey from last year was a fell beast, at least as strong as a swan, and it gave me some pretty good whacks, but nowhere near hard enough to break anything. --Sean 14:53, 16 May 2009 (UTC)[reply]
Oh dear. It's also a "myth" (read "not true") that swans in the UK are owned by the Queen. See - er - swan.--86.25.195.52 (talk) 06:32, 17 May 2009 (UTC)[reply]
The Swan article does not mention (at the time I read it) the Queen as far as I can see, but the Swan Upping article says that swans are owned by the British monarch - currently the Queen. 78.146.103.200 (talk) 16:10, 17 May 2009 (UTC)[reply]
"Today, the Crown retains the right to ownership of all unmarked mute swans in open water, but The Queen only exercises her ownership on certain stretches of the Thames and its surrounding tributaries.
This ownership is shared with the Worshipful Company of Vintners and the Worshipful Company of Dyers, who were granted rights of ownership by the Crown in the fifteenth century. Nowadays, of course, the swans are no longer eaten." From the official website of the British monarchy. 80.41.111.135 (talk) 19:37, 17 May 2009 (UTC)[reply]
There's an urban legend doing the rounds that says that killing a swan in Britain is considered an act of high treason and that theoretically, they're still allowed hang you for it. I've had several people quote that little factoid to me over the years... --Kurt Shaped Box (talk) 23:12, 17 May 2009 (UTC)[reply]

is there any evidence that the NSA uses Papal infallibility?

Is there any evidence (a leak, etc) of cooperation between the NSA and the vatican for using Papal infallibility to break encryption, for example, by factoring large composite numbers. Once the factors are produced, they are very easy to verify... --94.27.244.146 (talk) —Preceding undated comment added 11:25, 16 May 2009 (UTC).[reply]

I am very confused as to what you are asking. Are you saying that the Pope could theoretically use infallibility to have God factor the numbers for him? If so, this is really a question for the humanities desk. Although, if this is what you're asking, it is ridiculous; Papal infallibility only applies to matters of faith. The Pope can't decree who's going to win the Super Bowl, after all :-D -RunningOnBrains 11:41, 16 May 2009 (UTC)[reply]
Papal infallibility means the Pope can't be wrong, though you're right in that it applies to matters of faith. However, maybe there is a way to spin a large number with mysterious, unknown factors into a question of faith? If so, is there any evidence that the NSA has done this in cooperation with the Vatican? (The reason it's not "ridiculous" -- I mentioned this before -- is that the NSA doesn't have to believe in papal infallibility for it to work: the numbers are very easy to verify once the Pope produces them.) Any evidence of this ever being done? --94.27.244.146 (talk) 11:59, 16 May 2009 (UTC)[reply]
Believing that God would be swayed by any kind of spin is about as dumb as believing a good lawyer could get you out of Hell. SpinningSpark 12:04, 16 May 2009 (UTC)[reply]
(edit conflict) Again, you misunderstand the concept of Papal infallibility (I suggest you read the article). It does not suggest that the Pope can not be wrong, it is a provision where a Pope, if given a revelation by the Holy Spirit about church doctrine, can speak ex cathedra, and can not be accused of incorrect reasoning (since the "reasoning" came from God himself). -RunningOnBrains 12:06, 16 May 2009 (UTC)[reply]
I'm sorry, but there is no reason church doctrine cannot be massaged into including a factorization. The Vatican could very easily come up with questions of doctrine for the Pope the answering of which would be equivaelnt to factorizing a number. No reason it couldnt be done, and you have not given any. But I've just realized something: you haven't answered my actual question, about any evidence of cooperation between the NSA and the Vatican, at all! So if there is some evidence of that, then your whole answer could be considered a deterrent... --94.27.244.146 (talk) 15:41, 16 May 2009 (UTC)[reply]
Sorry, merely trying to clear up an apparent misconception. (But it is true...I am in favor of censorship in all its forms...especially when people get too close to the truth.) -RunningOnBrains 19:17, 17 May 2009 (UTC)[reply]
There is a popular misconception that papal infallibility means that the pope is never wrong. That is not the case. It's not even the case that it means that the pope is never wrong when talking about religion/faith. What it does mean is that under certain conditions the pope can proclaim that he has been divinely inspired as to the truth of a certain matter. The Catholic philosophy is that God (the Holy Spirit) would not allow the pope to be wrong when making such ex cathedra proclamations. (Again, while ex cathedra means "from the chair", it doesn't actually mean that there is a special chair he sits on which makes him infallible. The chair is a metaphor referring to his official position as pope.) There are a number of rather strict conditions that must be met before papal infallibility is considered to hold - it is not used on a routine basis. Out article on papal infallibility in fact only lists seven instances in the past 2000(ish) years, the most recent one being back in 1950. Pope John Paul II did not make any ex cathedra proclamations, despite having a relatively long tenure of 27 years -- 76.201.155.7 (talk) 12:15, 16 May 2009 (UTC)[reply]
I understand there are very special conditions which must be met for such a procalamtion. Is there any evidence that shows the NSA and Vatican cooperate secretly to make, but not disseminate, such proclamations? Also, your implication that there are only seven instances in the last 2000 years is misleading...the article only lists seven that were recounted in a 1985 study, and the Wikipedia article you just linked continues "The Vatican itself has given no complete list of papal statements considered to be infallible.". As far as I can see the conditions, he would have to speak on a matter of faith that must apply to all followers of the church. I don't see why such a proclamation could not at once answer a question important to the NSA, and I also dont see why it should not be possible not to disseminate this proclamation (keep it secret). If there is no evidence of NSA cooperation with the vatican, why don't you, and the other poster above, just say this? —Preceding unsigned comment added by 94.27.244.146 (talk) 15:51, 16 May 2009 (UTC)[reply]
Is this question based upon some sort of conspiracy theory, as a matter of interest? --Kurt Shaped Box (talk) 15:59, 16 May 2009 (UTC)[reply]
Even if we accept Christianity and Papal Infallibility as fact, This isn't something the pope can do on demand. (Too bad. If it worked like that it'd be extremely testable. We could have a scientific proof of God in about ten minutes.) The only reason he's infallible is because he's directly relaying a message from God. If God sent you a message personally it'd be just as infallible, except no one would ever believe you. Presumably if God wanted to help the NSA crack some codes, He wouldn't even have to bother the Pope. He could send the decryption keys to the NSA directly. 16:27, 16 May 2009 (UTC)
Of course there's no evidence. It's completely absurd. Nobody can provide any links for you (besides the obvious ones of papal infallibility and NSA) because the idea is so bizarre that nobody has ever thought it worthwhile to refute it. If you had read those articles, you would realize that your question makes as much sense as asking whether the Luftwaffe had secretly been involved in the formulation of Avogadro's number. Matt Deres (talk) 16:39, 16 May 2009 (UTC)[reply]
Nobody on here has yet said categorically, "no, there is no such evidence" because nobody has ever researched such a topic. There is no reference we can cite that says "there is no evidence". But even so, I am going to go out on a limb here, and say, no, there is no reliable evidence that the NSA and the Pope have been conspiring together to use whatever misinterpretation of papal infallibility you are working under to break encryption techniques. (I estimate a very low probability that I will be proven wrong in this claim). The very notion is ludicrous, but since you won't give up until someone tells you "no", here it is. No. Maelin (Talk | Contribs) 16:36, 16 May 2009 (UTC)[reply]
If such evidence did exist, we would all be furiously investigating it to determine if it was valid evidence. I don't think there's any evidence at all - not even flimsy evidence. If the original questioner believes he/she has found some evidence, then by all means, present it to us so we can tear it to shreds with the merciless scientific method. Nimur (talk) 16:52, 16 May 2009 (UTC)[reply]
Well, here was "pope", "nsa", and "crytpography" mentioned on the same page, and at IBM, no less. Nimur (talk) 16:55, 16 May 2009 (UTC)[reply]
No. There is no evidence whatsoever. (This is a summary version of the text I wrote that was lost by an edit conflict). The Pope is a normal human being. He has no supernatural powers, because there is no such thing as the supernatural, although it often makes a good storyline for various types of fiction. To stay sane, base your beliefs on true evidence, particularly scientific evidence, otherwise you'll end up like David Icke. You might as well say that the CIA is covering up the evidence that pigs can fly. I find it a matter of concern that in my experience, many North Americans barely discriminate between fact and fiction. 78.146.17.231 (talk) 17:15, 16 May 2009 (UTC)[reply]
you might as well say that the CIA is covering up the evidence that pigs can fly -- I guess you haven't been keeping up on the biggest news item of the past few weeks: swine flu. 94.27.244.146 (talk) —Preceding undated comment added 18:45, 16 May 2009 (UTC).[reply]
There is no rational link between the CIA's flying pigs conspiracy, and swine flu. Try to avoid jumping to conclusions. 78.146.17.231 (talk) 22:36, 16 May 2009 (UTC)[reply]
The swine flu conspiracy? Nimur (talk) 23:37, 16 May 2009 (UTC)[reply]
Anyway, the CIA have more important things to cover up. Like how the government puts fluoride in the water to turn the poor homosexual and thus reduce the birth rate... ;) --Kurt Shaped Box (talk) 22:49, 16 May 2009 (UTC)[reply]
The trolls are out this weekend. Tempshill (talk) 03:30, 17 May 2009 (UTC)[reply]

How to identify flowering plants - both wild and garden?

There are several flowering plants I would like to identify, both wild flowers (here in the UK) and garden flowers. Is there any online database that helps in the identification of such flowers? And are there any usefull clues to narrowing the identification down, such as the number of petals? I have a copy of a flora - the New Flora Of The British Isles by Clive Stace - but you need to be a trained latin-speaking botanist to use it, and it has very few illustrations. 78.146.190.197 (talk) 13:16, 16 May 2009 (UTC)[reply]

Here's [18] somewhere to start. You'll need to submit info, but it also serves to familiarise you with some technical words which might assist you with your book. Richard Avery (talk) 14:21, 16 May 2009 (UTC)[reply]

You need a botanical key. By the way this is just a redirect and we could do with a more specific article! These can be in several steps, firstly to identify the family, and then down to finer levels of classification. I have an old one in book form to identify genera. Graeme Bartlett (talk) 08:20, 17 May 2009 (UTC)[reply]

I mentioned above that I already have got a "botanical key" - I am more familar with it being called a flora. I eventually, after several goes, identified the plant I was most curious about partly using the link above thanks - it is commonly called Green alkanet. 78.146.103.200 (talk) 16:27, 17 May 2009 (UTC)[reply]

Adaptation or death effect?

I was chatting with a friend and my thoughts sort of got off on a tangent about the human body. I know that, for example, if you eat too much of X you might end up actually needing to eat excess amounts of X because your body is "dependant" on the higher amount. The way people talk about it you'd think the body is aware of itself and makes conscious decisions. Obviously untrue. But is there a specific mechanism in the body that makes it absorb less of something if excessive amounts are present, as a failsafe? Or do the cells that absorb it naturally end up dying from the overdose, leaving only cells that absorb it inefficiently to survive and replicate themselves (a bit like how "supervirii" come about)? 97.104.210.67 (talk) 15:54, 16 May 2009 (UTC)[reply]

The effect you are describing sounds like Physiological tolerance which is a redirect to Drug tolerance right now, and contains links to other articles. You may want to follow some of those links and see where it takes you. --Jayron32.talk.contribs 18:49, 16 May 2009 (UTC)[reply]
@97.104.210.67 -- Can you justify your assertion that "if you eat too much of X you might end up actually needing to eat excess amounts of X because your body is "dependant" on the higher amount"? While this may be true for drugs as a special case (see Jayron's comment) I don't generally think this has much to do with typical dietary intake. Perhaps you should start by reading about how nutrients are absorbed into the body. Try the articles gastrointestinal physiology and anatomy. In answer to your question, one mechanism by which the body adjusts nutrient uptake is by regulating cell surface proteins on the enterocyte that participate in transport of small molecules across the brush border into the blood stream. --- Medical geneticist (talk) 19:11, 16 May 2009 (UTC)[reply]

Minimum force to break an arm (inspired by swan question above)

The above swan discussion (not sure how to link to it) got me thinking - is there are a minimum amount of force required to break an average (healthy) human arm? People seemed to suggest it wouldn't be possible for a swan to, but then i've seen footballers break bones with seemingly minimal force (I guess it may be lots of force just it's not apparent from the camera's perspective). Anyhoo I would've thought that a 'lucky' swan and an 'unlucky' human could result in a swan breaking their arm. ny156uk (talk) 16:59, 16 May 2009 (UTC)[reply]

oh come on, "minimal" force? We've all run into door frames, table edges, etc a few times in our lives, not to mention falling on ice etc, none of this breaks an arm! (except perhaps among the elderly) —Preceding unsigned comment added by 94.27.244.146 (talk) 18:31, 16 May 2009 (UTC)[reply]
It depends entirely on how the force is applied. The angle and concentration of the force in certain areas will have very different results. Are we striking the arm against the edge, or "jamming" the arm at the wrist or elbow? Are we affecting the humerus or the ulna or the radius bone? There's just far to many variables to predict what will happen. You can break an arm after tripping and falling, and you can be hit by a car and not break an arm. You aren't going to get a reliable number here... --Jayron32.talk.contribs 18:43, 16 May 2009 (UTC)[reply]
My friend broke his arm falling off a low sofa. I think it depends how you land. Catoutofthebag (talk) 20:43, 16 May 2009 (UTC)[reply]
I think people often underestimate the forces involved. Typically, when footballers break bones, the impacts aren't anywhere near as light as they may seem. First of all, these guys tend to be running at the time, so they're packing quite a bit of energy there. Momentum and mass -- typically, their own body (and your average footballer isn't a short skinny guy) -- can result in pretty impressive forces. I mean, it often looks like they're not really doing all that much (still photographs and slow motion can really fool the eye), but if you've got a guy who weighs, say, 80 or 90kg (about 175 to 200 pounds) running just about as fast as he can, he represents a lot of energy. If he suddenly comes to a stop so that all of that energy is concentrated on a single limb, that's a lot of stress right there.
This video (which, I should add, is work safe but may not be something you want to see, because it does involve a pretty nasty-looking leg break), for example, shows a guy who's just running. Nobody touches him, he doesn't really stumble or anything, but he is trying to slow down and change his direction of movement very quickly to take control of the ball, and unfortunately the way he puts down his foot concentrates the forces involved on that his leg, and the bone just can't take it. Yet his movements seem kind of light, almost effortless, until he falls down.
By comparison, let's take high-speed motorcycle races: crashes tend to look far more dramatic, but the riders often walk away without any serious injury. The reason for this is that not only are they wearing protective gear, but the tracks tend to be flat and surrounded by wide areas of clear grass. That means that when they crash, they can just slide and expend the energies involved until they come to a stop. They typically aren't badly injured unless their initial landing right after the fall is bad or they get hit by another bike -- all they get is bruises and maybe some burns from the friction. The point is, even though the crashes may look very violent, the actual forces the rider's body is subjected to often aren't that great. It's all in how you expend the energy. -- Captain Disdain (talk) 02:20, 17 May 2009 (UTC)[reply]
    • In an even more dramatic example, I recently saw footage of a racecar crash in which the car launched into the air, bounced off the guard fence in front of the audience, slid to a stop and burst into flames. The driver walked away from the car apparently uninjured - a woman in the crowd broke her arm. Dcoetzee 02:51, 19 May 2009 (UTC)[reply]

HIV antibody testing

Is it possible to have a negative result from HIV antibody test after use of antiretroviral drugs?¬¬¬¬ —Preceding unsigned comment added by Tunmisadej (talkcontribs) 17:55, 16 May 2009 (UTC)[reply]

You mean for someone that is HIV positive, presumably? --Tango (talk) 18:35, 16 May 2009 (UTC)[reply]
I don't get why a person would have such a test taken if they are already HIV positive -- that status doesn't change -- as indicated by being on antiretroviral drugs... I think we're going to need more background on this one... 94.27.244.146 (talk) 18:49, 16 May 2009 (UTC)[reply]
I can think of a cynical reason for taking such a test — with the intention of demonstrating a negative result. Antiretroviral drugs do not remove the antibodies, so the HIV (antibody) test remains positive. However the test can occasionally give false results — see "HIV test#Accuracy of HIV testing". Axl ¤ [Talk] 22:35, 16 May 2009 (UTC)[reply]
It's not uncommon for someone to receive care in a location (e.g. country) far from home. Sometimes they are unable to speak due to an acute illness, and they might not have their medication list with them. --Scray (talk) 23:44, 16 May 2009 (UTC)[reply]
You can be HIV+ without knowing it, of course. The OP asked if it was possible to get a negative result - interpreted literally, the answer is clearly "yes". You can get a negative result by being HIV-. --Tango (talk) 00:41, 17 May 2009 (UTC)[reply]
The standard tests using ELISA screening followed by Western Blot confirmation remain reliable, but false-negative tests in persons receiving long-term antiretroviral therapy have been reported with some of the "quick" tests, which must be interpreted with care (some are no longer in use). --Scray (talk) 23:44, 16 May 2009 (UTC)[reply]
I read the OP's question as: Suppose a patient is HIV-positive and takes the drug cocktails. Is it possible for the virus to be hammered down so much in quantity that he'll test HIV-negative? Tempshill (talk) 03:38, 17 May 2009 (UTC)[reply]
That's a nice find, Scray. Of note, "These data suggest that these subjects experienced anti-gp41 seroreversion as a consequence of early and prolonged suppression of viremia by HAART." So the answer to the original question is: yes, depending on which test is used. Axl ¤ [Talk] 09:28, 17 May 2009 (UTC)[reply]
Thanks and agreed. This highlights the value of our standard test for HIV, and the importance of being very careful before offering alternatives as equivalent. --Scray (talk) 11:00, 18 May 2009 (UTC)[reply]

Metal Salts

If I have elemental copper and aluminum, how can I turn them into metal salts? Alyosha (talk) 18:00, 16 May 2009 (UTC)[reply]

Copper is fairly inert stuff, but will react with water, oxygen, and carbon dioxide (slowly) in the air to form patina or verdigris. If you are looking for a more controlled reaction, it will react with concentrated nitric acid to form copper(II) nitrate salt and noxious brown nitrogen dioxide gas. Aluminum presents an interesting problem. Chemically, it is a very active metal; however aluminum forms a thin coating of Aluminum Oxide on its surface, which is fairly impervious stuff. Powdered aluminum will react MUCH faster than solid aluminum, owing to the higher surface area. However, even aluminum foil will react with any decently strong acid, say hydrochloric acid, to form aluminum chloride salts. You could also use nitric acid as above, but the production of NO2 gas is to be avoided unless absolutely necessary. Since Copper is not normally oxidized by acid (the nitrate ion in the nitric acid is the oxidizing agent here) HCl won't do much to copper. Aluminum, however, is easily oxidized by any strong enough acid, so I would go with the easier to work with HCl. --Jayron32.talk.contribs 18:39, 16 May 2009 (UTC)[reply]
I don't really have either acid, and there's no chance that I'll be able to get them. But thanks anyway! :) Alyosha (talk) 19:01, 16 May 2009 (UTC)[reply]
Then let's back up a sec here. Since you're asking (we now know) for practical advice, you'll need to tell us what your level of expertise is and (if you're just a household experimenter) what you do have available. What's your larger goal here? Is the whole point of the experiment to do this conversion? Are you wanting some specific salt? Are you wanting to use the salt for something? Are you just trying to dissolve metal? With some context and constraints, it will be much easier to provide advice that is actually useful to you. DMacks (talk) 19:07, 16 May 2009 (UTC)[reply]
I am try to create a Galvanic cell. My plan was to create the metal salts for the electrolyte solution. Alyosha (talk) 19:15, 16 May 2009 (UTC)[reply]
this is kinda complicated
this works just as well
No need to. You just need some electrolyte solution that will have material that will complete both the oxidation half and the reduction half of the reactions, and while the "classic" galvanic cell uses a set up like is in this picture, this is really only needed for quantitative studies where you want to carefully control the the concentrations of the solution to study the effect on the voltage, or something like that. If all you want is to make a simple galvanic cell, just about any acidic medium will work. If you take ANY two dissimilar metals and place them into a sufficiently resistive but acidic medium, you will generate a usable voltage. Take an iron nail and a copper nail, jam them into opposite ends of a lemon or an orange or a potato or something like that, and you will get a measurable voltage. It is usually enough to power a small liquid-crystal display, such as a watch. Google "lemon clock" or "potato clock" to get examples. Basically, the acid in the lemon and/or potato acts as the electrolyte solution for both half-cells, and the solid parts of the lemon/potato/whatever are resistive to electricity, but also semi-permiable to ions, so it also acts as the salt bridge. Its really a simple set up. So try it. Take a lemon, jam your metal parts into opposite ends, and hook it up to the leads in your watch in place of the battery. It will probably work. It will at least generate enough current to register on a simple voltmeter. --Jayron32.talk.contribs 19:56, 16 May 2009 (UTC)[reply]
Thanks.Alyosha (talk) 20:21, 16 May 2009 (UTC)[reply]
Heck yeah, we even have a lemon battery article:) DMacks (talk) 21:51, 16 May 2009 (UTC)[reply]
Everyone should build a lemon battery at some point in the lives! (It works well with any citrus fruit, but it also works with other fruits and vegetables - potatoes, even!) --Tango (talk) 22:30, 16 May 2009 (UTC)[reply]

Question for you about quantum mechanics. At Wiktionary we have a definition of: {{physics}} The branch of [[physics]] which studies [[matter]] and [[energy]] at the level of [[atom]]s and other [[elementary]] [[particle]]s, and substitutes [[probabilistic]] mechanisms for [[classical]] [[Newtonian]] ones.

I reckon we could improve the last part which says "and substitutes probabilistic mechanisms for classical Newtonian ones" into something better. Please see wikt:quantum mechanics. --Jackofclubs (talk) 18:43, 16 May 2009 (UTC)[reply]
Maybe quantum theory too - I'm not so sure on this definition. --Jackofclubs (talk) 19:54, 16 May 2009 (UTC)[reply]

Is it physically possible to rip a man's arm off with your bare hands?

Question as topic. Partially inspired by the swan question above, partially inspired by my playing of Mortal Kombat II again last night. Is this within the limits of human strength (I'm including trained musclemen/strongmen here too)? --Kurt Shaped Box (talk) 18:55, 16 May 2009 (UTC)[reply]

Sure, take a weak man or leper, it is easy. --Jackofclubs (talk) 19:13, 16 May 2009 (UTC)[reply]
Leprosy would have no effect on the ease of pulling off an arm. Such notions, and pejorative terms like "leper", have no place on the science desk. --Scray (talk) 23:24, 16 May 2009 (UTC)]][reply]
"Leper" primarily means a person afflicted with leprosy and that useage is proper English. I see no reason to suppose Jackofclubs used "leper" with a secondary perjorative intent nor why WP:AGF is not applied here. Cuddlyable3 (talk) 14:52, 17 May 2009 (UTC)[reply]
Fair enough - thanks for pointing out that I worded that too strongly. Leprosy#Etymology does point out that this term is considered pejorative (it's got an ugly past). --Scray (talk) 20:51, 18 May 2009 (UTC)[reply]
I don't think so. Regardless of the victim's muscle strength, the ligaments around the shoulder are strong. Dislocation or fracture might occur with a lot of effort, but not avulsion. --Scray (talk) 23:28, 16 May 2009 (UTC)[reply]
I'd be surprised if it was impossible! It'd most likely take quite a bit of work and leverage, if the subject was a healthy adult, though, but if you keep yanking on that arm, sooner or later something's going to give. That's a far cry from Mortal Kombat action, of course, but still. -- Captain Disdain (talk) 01:43, 17 May 2009 (UTC)[reply]
I read a story once of a chimp ripping the arm off a person who had been tormenting it. Dmcq (talk) 13:01, 17 May 2009 (UTC)[reply]
Oh, sure, I have no trouble believing that. Chimps are in a league of their own compared to us puny humans, as this Straight Dope column entertainingly tells us. They pack serious muscle. -- Captain Disdain (talk) 14:07, 17 May 2009 (UTC)[reply]
I suppose that if you kept twisting, pulling and bracing against the torso with your leg, then it might come off eventually. Does anyone know offhand (or is able to quickly calculate) the force required to directly rip off a human arm? --Kurt Shaped Box (talk) 01:28, 19 May 2009 (UTC)[reply]


Well, if it was on CSI - it must be true. :-P SteveBaker (talk) 01:25, 19 May 2009 (UTC)[reply]
If you cooked the person first, it would be a lot easier. Think Thanksgiving turkey. 65.121.141.34 (talk) 13:59, 19 May 2009 (UTC)[reply]
True. Also, if you used a meat cleaver, it would be a lot easier. Or tied the torso to concrete wall and attached a rope to the arm, and the other end of the rope to a car, and then drove away. But that wasn't the question... -- Captain Disdain (talk) 11:20, 20 May 2009 (UTC)[reply]
My suggestion follows the letter of the law, question, whatever (though probably not the intent). 65.121.141.34 (talk) 16:43, 20 May 2009 (UTC)[reply]

Infinite Improbability

Do we live in a universe that there is is no probability of something happening (it will never happen, probability = zero), or is anything and everything is possible but the probability of it happening is infinitely small (probability = 1/infinity)? 129.21.109.153 (talk) 19:28, 16 May 2009 (UTC)[reply]

First of all you have to realize that probability = 0 doesn't mean that it can't happen. For example when you throw a dart at a dartboard, if there are infinite points then the chances of hitting exactly the center is 0, since if you 'zoom in' enough you can always see that it is not exactly the center. Still, it can happen, although the probability is 0 that it will. (1 point is the bullseye / there are infinite points total = 0)
As for your question, it is more relevant to things like location: it is true that everything can be everywhere, but that the probability for example of you turning up whole and unharmed in China in the next second is pretty close to 0 -- however it could happen, because particles don't have exact locations but probability distributions instead... —Preceding unsigned comment added by 94.27.244.146 (talk) 20:00, 16 May 2009 (UTC)[reply]
I disagree that the probability of hitting the centre of a dartboard is zero - you are assuming the dart board can by modelled as a portion of a Euclidean plane to arbitrary precision, but that isn't the case. There is a limit to how precisely you can define the centre (see Heisenberg's uncertainty principle among other things), so either you have to say that the concept of a centre of a dartboard is meaningless, so there is no probability, or the centre is a region with finite area, so there is finite (ie. non-zero) probability. --Tango (talk) 22:26, 16 May 2009 (UTC)[reply]
Or better yet, Planck length. --131.188.3.20 (talk) 00:14, 17 May 2009 (UTC)[reply]

OK that makses sense. So anything is possible then right, even if the chances are really really small? Also as a quick aside, does the probability distribution of a particle cover everwhere, or does it end with in a certain distance from the "center", or where it's most likely to be?129.21.109.153 (talk) 21:08, 16 May 2009 (UTC)[reply]

It depends on what you mean by "anything". Anything not specifically ruled out by the laws of physics is possible. It is not possible (unless our understanding of physics is incorrect, which can't be ruled out), for example, for something inside the event horizon of a black hole to leave it. It is not possible for an electron to spontaneously turn into a positron without any other particles being involved (due to conservation laws). etc. etc. --Tango (talk) 22:26, 16 May 2009 (UTC)[reply]
What I once read was that in infinite time, everything that is not absolutely impossible will happen. 78.146.17.231 (talk) 22:47, 16 May 2009 (UTC)[reply]
Will almost surely happen. That's not quite the same as "will happen", but the difference is rather subtle (and probably only really understandable to mathematicians). --Tango (talk) 00:23, 17 May 2009 (UTC)[reply]
According to one interpretation (which I personally think is ricidulous,) everything will eventually happen. See Omega Point (Tipler). -Arch dude (talk) 00:36, 17 May 2009 (UTC)[reply]

For any ergodic mesoscopoic or macroscopic system that is not "frozen out" or locked in an infinitely small part of its phase space, it is certain that the vast majority of accessible phase space will never be visited. Consider a system of N = 100 000 non-interacting distinguishable particles in a box (say, a few grams of sand in a jar aboard the International Space Station). The grains of sand are flying around at about 10 cm/s, and the jar is about 10 cm high. Probability for all the grains to momentarily find themselves on their respective sides of the midline of the jar is 2-N. Even if we assume that the sand changes its state (i.e. any grain crosses the midline) every microsecond, which is over-optimistic to say the least, it will still take approximately 103x104 seconds for the sand to visit an non-negligible fraction of the possible states, and very much longer to visit all states (as nothing prevents the same state from occurring more than once). The lifetime of the universe is much, much shorter than 103x104 seconds. It is therefore quite meaningless to speak about everything possible happening. In fact, an infinitely small part of possibilities are ever realized, at least in statistical mechanics. --Dr Dima (talk) 02:06, 17 May 2009 (UTC)[reply]

Of course - IF the universe is finite in both space and time - then the probability of any of these crazy things happening is extremely close to zero - so close that we can be quite confident in asserting that they won't happen. However, if the universe turns out to last forever - or to be infinite in spatial extent - then no matter how improbable, all of these things are happening (or are going to happen) - not just once - but an infinite number of times. However, the probability of any human ever being able to detect one of these crazy things is still so close to zero that it might as well be. It's worth noting that we don't know for sure whether the universe is either spatially or temporally finite - so we can't be definite about the answer to this question. But infinities and one-over-infinities are tricky things to deal with and talk about. SteveBaker (talk) 02:59, 17 May 2009 (UTC)[reply]

As SteveBaker, says "infinities and one-over-infinities are tricky things to deal with and talk about" but in this case we can be pretty definite that not everything that can happen will happen, even in an eternal universe. Consider for example a machine that picks up a random real number between 0 and 1 every second for eternity. Even if run for eternity, the machine will not end up picking all of the numbers in its range; in fact, by some measure, the machine will run through almost none of the numbers between 0 and 1. ( Yes, this mathematical example can be converted to a physical thought experiment, but I won't bother to do so here.)
Of course, the basic problem is that are talking about concepts like "everything" "eternity", "infinite" without actually defining any of those terms. And it's hard impossible to answer the original question, without agreeing upon a common terminology. Abecedare (talk) 03:31, 17 May 2009 (UTC)[reply]

Werner Heisenberg, one of the founders of quantum mechanics stated The existing scientific concepts cover always only a very limited part of reality, and the other part that has not yet been understood is infinite. It is presumptuous to claim that anything or everything is not possible in that infinite part of reality. Cuddlyable3 (talk) 14:23, 17 May 2009 (UTC)[reply]

I understand all of the points that have been made, but doesn't it make sense that there is infinite time (there is...right?), then everything will happen, exist, occur, eventually. And it already has and will infinitely more times. Even when and however this universe ends then something "new" will come out of it and then that will have an entire time of existing and then it will end and something else will arise, etc. So I think that given enough time, anything and everything will occur, even if it is infinitely improbable. 129.21.109.153 (talk) 16:26, 17 May 2009 (UTC)[reply]

The universe almost certainly hasn't existed for an infinite amount of time already (current estimates put it at about 14 billion years old) and almost certainly won't exist in its current form for an infinite amount of time. There are various theories about what could happen, I think the ones that a currently thought to be most likely are heat death and Big Rip. While the universe continues to exist forever in both of those cases, it doesn't exist in a form in which much can happen (at least, it can't happen very fast - I'm not sure if activity in a heat death tends to zero fast enough to prevent infinitely many things happening). --Tango (talk) 16:48, 17 May 2009 (UTC)[reply]

Science Desk Question

This is not at all a science question, but a question about the behaviour of this desk. Has anyone noticed that when you ask or answer a question on this desk then publish it, the page appears then immediately begins to scroll up? It only happens on this desk and none of the others. What is happening and can it be fixed by someone?--KageTora - (영호 (影虎)) (talk) 22:40, 16 May 2009 (UTC)[reply]

Would be better asked at Wikipedia talk:Reference desk —Preceding unsigned comment added by 82.44.54.169 (talk) 22:44, 16 May 2009 (UTC)[reply]
I don't know if this is anything to do with what you're describing - but I've noticed that the presence of mathematical formulas on this page causes the page to scroll up or down (depending on whether they're above or below what you're reading) if you try to navigate the page while the graphics are still loading... --Kurt Shaped Box (talk) 22:53, 16 May 2009 (UTC)[reply]
When one returns to a previously viewed page, a browser tries to bring one back to the same position one last scrolled to. I suppose it confuses browsers when a page has changed due to ongoing edits at different places while one is away. Cuddlyable3 (talk) 14:04, 17 May 2009 (UTC)[reply]
I think it has something to do with images. When you edit a section on Wikipedia, you are automatically returned to that section when it reloads the page with your changes. If this section is at the bottom of a long page (such as here after you ask a new question in a new section) with images, the text may load far before the images, depending on your browser speed. As the images load, the text will be "pushed down" a certain amount. It may happen here only since we tend to have a lot of images (animal identification, graphs, diagrams, etc.)-RunningOnBrains 19:26, 17 May 2009 (UTC)[reply]
Is there any way of telling my browser not to do that? I find it mildly irritating. --Kurt Shaped Box (talk) 23:18, 17 May 2009 (UTC)[reply]
You could refuse to look at pages in which the image tags don't carry size info. —Tamfang (talk) 16:43, 18 May 2009 (UTC)[reply]
Is there any way of telling my browser to warn me before I do that, then? ;) --Kurt Shaped Box (talk) 23:46, 18 May 2009 (UTC)[reply]

TB treatment that grants immunity to lung cancer?

My grandmother says she underwent an aggressive drug treatment for tuberculosis, and that doctors told her she can never get lung cancer as a result of it, despite that she smokes daily. If drugs used to treat TB can totally prevent lung cancer, why aren't they offered to all smokers? NeonMerlin 22:51, 16 May 2009 (UTC)[reply]

I don't know the medical side of that statement. But from an ethical point of view, doctors do not want to encourage smoking. Smoking can cause all sorts of other diseases and other cancers (cancer of the mouth being one). --KageTora - (영호 (影虎)) (talk) 23:05, 16 May 2009 (UTC)[reply]
Certainly sounds like misinformation, miscommunication, misunderstanding, misremembering, or denial. My vote is on the last one. Smokers are always looking for excuses. --Scray (talk) 23:12, 16 May 2009 (UTC)[reply]
Yes, we are. And I will admit to being guilty many a time for that.--KageTora - (영호 (影虎)) (talk) 23:28, 16 May 2009 (UTC)[reply]
I don't buy it either. The treatment for TB is antibiotics. If antibiotics acted as a vaccine against lung cancer I'm pretty sure I would have heard of it. --Tango (talk) 00:38, 17 May 2009 (UTC)[reply]
As the other posters have mentioned, this is almost certainly a "no way", but it may be worthwhile to ask your grandmother exactly what the treatment was. You could then research that treatment (at an online encyclopedia perhaps!) and that might give you insight into how she got confused by what the doctor said. Quitting smoking at any age can be a boon; this kind of misinformation is helping her make an unhealthy choice. Matt Deres (talk) 14:21, 17 May 2009 (UTC)[reply]
Quitting smoking at that age (I assume she is fairly old) is rare, but my father did it at 65 - everyone was shocked because he had been a smoker for exactly 50 years. However, when quitting smoking it is best to do so gradually, as it can be a dangerous shock to the system for some people (this is not medical advice - wait for it!) so my father went on to having a cigar or two a day (max) instead of a full packet of cigarettes. He's done well, and still enjoys his cigars. It's always best to make an effort, at least. I went down from 20/day to 10/day a few years ago, and managed to even get that down to 5/day when I cam back to the UK (because it's so bloody expensive and having to go outside all the time is a PITA!). Good luck in your research!--KageTora - (영호 (影虎)) (talk) 02:19, 18 May 2009 (UTC)[reply]
There is a treatment for cancer that uses active TB. Maybe her doctor was referring to that [19]. I would be surprised if it provided immunity. [20] This study found both TB and cancer in a small number of patients. [21] 71.236.24.129 (talk) 17:32, 18 May 2009 (UTC)[reply]

Time Zones

Inspired by a question over on Misc. Desk about planes, jurisdiction, and British Licensing laws. In some ways I consider this a science question, but don't ask me what the relevance to the question on the Misc. Desk is, because it's very convoluted. How fast (or slow) would a plane have to fly through time zones (west, of course, and at the equator) for the time to remain the same, say, at 6pm? Basically, I suppose the question is asking how fast does the Earth rotate.--KageTora - (영호 (影虎)) (talk) 23:03, 16 May 2009 (UTC)[reply]

1670 km / hr (1040 mph). Dragons flight (talk) 23:55, 16 May 2009 (UTC)[reply]
That's 40,000km in 24 hours. The circumference is also 360*60=21,600 nautical miles, so the speed would be 21,600/24=900 Knots. Yo cannot do this in a commercial airliner at the equator since it's faster than the speed of sound. I find it easier to remember that 40,000km=25,000 miles=21,600nautical miles, than it is to remember the actual conversion ratios. (For miles and km, you can also remember that C=186,000 miles/sec=300,000km/sec.) -Arch dude (talk) 00:30, 17 May 2009 (UTC)[reply]
Of course, if you simply stand sufficiently close to the North Pole, you could do it yourself just by walking in a circle over the course of a day, although the time of day in the polar circles is less meaningful than elsewhere due to the strange observed behaviour of the sun. Note that since geographic time zones boundaries are set based not only on the sun's behaviour but also on national borders and other geographical features, the intervals around the equator are not perfectly evenly spaced. Whilst you would see the sun remain in the same relative position, if you kept a clock at local time and updated it each time you crossed a time zone boundary, you would not get a nice, perfectly regular update. Maelin (Talk | Contribs) 02:04, 17 May 2009 (UTC)[reply]
If it is done at a latitude where a jet plane could do it, don't forget that as it crosses the International Date Line, traveling west, the calendar date jumps forward one day. However, you must put your watch back one hour each time you cross a time zone demarcation line. You put your watch back 24 times when going around the earth. That negates the calendar jump forward. In between each putting back the watch moves forward normally, and that adds up to the actual flying time. The plane would get back the day after it started because somewhere on the trip local time passed from 11:59:59 p.m. to 00:00:00 a.m. (not necessarily at a time zone demarcation line). That would move the date forward. This would apply even if you are close to the North Pole and walking around it. – GlowWorm. —Preceding unsigned comment added by 98.21.104.80 (talk) 03:41, 17 May 2009 (UTC)[reply]
As always, Wikipedia has an article on this topic. Take a look at sun-synchronous orbit. The article geared towards satellites, because those are the most likely realistic scenario. There are a lot of variations, based on particular orbit inclinations and trajectories. You can work out the necessary velocity for a plane to do this (substitute orbital radius for Earth's radius plus altitude above sea level). Specifically, you want an orbital period to be one sidereal day as viewed from the surface of the earth; this is called a semi-synchronous orbit, and it's discussed in several articles. Nimur (talk) 04:14, 17 May 2009 (UTC)[reply]
If there were continuous land mass at the equator and no national boundaries then time zones could match the Earth's rotation which is 15 degrees of longitude per hour. Flying at the speeds calculated above could keep one at the same time in each zone. However the actual equatorial route passes over various national borders and coastlines. These are the reason for displacements of some time zone borders to east or west[22] and therefore the plane would have to vary speed correspondingly, while the average speed is as given above. AFAIK countries on the equator do not add the complication of Daylight saving time. Cuddlyable3 (talk) 13:50, 17 May 2009 (UTC)[reply]

So it would actually be quite hard, then? Oh, and thank you all for understanding that my question did not mean 'at exactly 6pm', as that would be impossible, but meant 'within a certain hour, such as 6pm-6:59pm'. Thanks for the answers.--KageTora - (영호 (影虎)) (talk) 14:47, 18 May 2009 (UTC)[reply]

If you did not need to fuel, Concorde, and many military aircraft could do it. Practically speaking however, midair refueling is not possible on the Concorde, and takes place at relatively low speed in any case. 65.121.141.34 (talk) 13:56, 19 May 2009 (UTC)[reply]

May 17

what spider is this?

what species? i saw it in my back yard the picture is [23] i posted it earlier but a picture of spiderman showed up since it had the same name as the commons picture, spidey.jpg. anyways any help identifying it would be awesome. —Preceding unsigned comment added by Spideygonnagetu99999 (talkcontribs) 03:47, 17 May 2009 (UTC)[reply]

I have never seen that before (I am in the UK), but I am assuming by the time stamp that you are more West from me, and therefore probably North America. Try this database. They have photos. Good luck.--KageTora - (영호 (影虎)) (talk) 04:37, 17 May 2009 (UTC)[reply]

My intuition tells me it's in Linyphiidae family, but I can't be sure. Sorry. --Dr Dima (talk) 07:30, 17 May 2009 (UTC)[reply]

I'd go for an Orb-web spider, possibly a Barn spider, it would help to have some idea of size. Mikenorton (talk) 12:02, 17 May 2009 (UTC)[reply]
Can you please describe the web of the spider you are asking to identify? That will help a lot. --Dr Dima (talk) 22:24, 17 May 2009 (UTC)[reply]
This spider is in my yard in Richmond, California its about 2-3 inches from front feet to back feet, the main part of its body being 1.5-2 inches, and its about 1-1.5 inches wide. the web is big and roughly hexagonical but more than six, like 18 to 20 ended shaped. —Preceding unsigned comment added by Spideygonnagetu99999 (talkcontribs) 23:38, 17 May 2009 (UTC)[reply]
This sounds a lot like an orb-weaver indeed. Orb-weaver webs are rather large (usually 1-3 feet across), near-vertical, and look something like this. Individual linyphiid webs, on the other hand, are small and sheet- or tent-like. Also, 1.5" size is about right for Araneus sp. (a typical orb-weaver), but huge for a linyphiid. --Dr Dima (talk) 04:40, 18 May 2009 (UTC)[reply]

Variable Frequency Sine Wave Generator

I need to create a sine wave generator circuit, whose frequency output f will be proportional to the input voltage/current. In a Wein-bridge oscillator, if in place of the resistance, a transistor is used, operating as an amplifier, with the gate connected to the frequency control terminal, will it suffice? —Preceding unsigned comment added by 218.248.80.114 (talk) 06:30, 17 May 2009 (UTC)[reply]

You are wanting a voltage controlled oscillator or VCO. You can buy ICs with this built in, or as you suggest a MOSFET or field effect transistor could be a resistance controlled by a voltage. If you want to you can use a capacitor controlled by a voltage, such as a reversed biased diode (varicap), this can very quickly change the frequency. Graeme Bartlett (talk) 08:08, 17 May 2009 (UTC)[reply]
You may consider designing an Arbitrary waveform generator to produce a sine wave output. The only active analog circuit this requires is a Digital-to-analog converter which is available as a single fully-specified component. It would receive from memory (RAM or ROM) at a relatively high clock rate samples of the wave to be generated. The memory would be loaded, or the clock rate controlled, by a digital controller that includes an ADC for the input voltage/current. Cuddlyable3 (talk) 13:16, 17 May 2009 (UTC)[reply]
I've used a varac diode to tune the resonance of a voltage-controlled oscillator. This is conceptually the simplest VCO, because your applied voltage directly changes the resonant frequency of the LC-tank, but it is complicated by the nonlinear and uncooperative behavior of a varac. Nimur (talk) 19:59, 17 May 2009 (UTC)[reply]

Human digestive system - top speed? ;-)

So I was curious how fast the human digestive system works, and read the article - apparently 40~50 hours? A lot longer than I'd have guessed. What I'd really like to know, though, is how fast something can run through you... I'm clearly thinking about really bad diarrhea here...

Surely I'm not the only one who has ever had really bad diarrhea and began to think back over what all they'd eaten trying to pinpoint the source?

I did an archive search for "diarrhea speed" but only got lots of discussion about traveling at the speed of light, you sickos! ;-) —Preceding unsigned comment added by 61.189.63.198 (talk) 07:28, 17 May 2009 (UTC)[reply]

A patient preparing for a Colonoscopy must stay on a clear liquid diet for a day, followed by a day of purging using laxitives and lots more clear liquids. -Arch dude (talk) 09:04, 17 May 2009 (UTC)[reply]
Ever had a barium meal? It takes less than two hours for that stuff to come out and you need to make sure you are near a toilet when it starts!--KageTora - (영호 (影虎)) (talk) 09:36, 17 May 2009 (UTC)[reply]
I think some gastric infections (e.g. Norovirus) may affect other parts of the digestive system, in that something might stimulate strong peristalsis to get whatever is causing it out of the system. Digestive Transit (for which we don't have an article!) is the name given to the time taken for food to pass out of the system. It varies between people: if you don't drink enough and/or are constipated, things will take longer to leave you (if at all!). --TammyMoet (talk) 10:55, 17 May 2009 (UTC)[reply]
For estimating normal speed, eat some sweetcorn and time thje result. 78.146.103.200 (talk) 16:48, 17 May 2009 (UTC)[reply]
Also, the GI tract is not a FIFO queue. Different types of food have different characteristic time-scales. It's possible for a faster item (like soup) to pass faster than a slow item, like steak. By the time it's in the intestine, everything's pretty soupy anyway. Nimur (talk) 17:04, 17 May 2009 (UTC)[reply]
A search for "diarrhea speed" gets stuff about the speed of light? Sounds like a joke from Spaceballs (Sure glad they called it "ludicrous speed" instead. :-) )Somebody or his brother (talk) 00:23, 18 May 2009 (UTC)[reply]

3 PHASE IN ELECTRIC POWER SYSTEM

I want to know why the electric powe generator is designed to be 3 phase not more than this? in other way why is the angle between Phase and the other must be 120 degree?~ thank u —Preceding unsigned comment added by 3ateka (talkcontribs) 10:13, 17 May 2009 (UTC)[reply]

The article for this is Three-phase electric power. This is an engineering decision. It isn't impossible to use fewer, or more, phases, but using three phases is most cost-effective. It is the solution that needs the least conductor material, and if you're going to build a power line across hundreds of miles, conductor material (copper) will be the decisive cost factor. --dab (𒁳) 10:34, 17 May 2009 (UTC)[reply]

Nikola Tesla used two phase power pretty effectively to run his induction motors. But when you add the third phase, starting the motor is easier and the flow of energy to the motor is steadier. There is little benefit to adding more phases, considering the higher cost of conductors, insulators, and structures, not to mention circuit breakers, transformers, etc. Edison (talk) 20:52, 17 May 2009 (UTC)[reply]
In the US, most residences only have one 60hz phase. We use both poles of one phase to get to "pushes" per cycle, so flourscent bulbs flicker at 120hz. the high-voltage is delivered to the transformers in the neighborhood via a single conductor, and each home is connected to teh transformer with two power conductore (the phase nad the anti-phase) plus a neutral. In industrial areas, the powe company delivers all three phases using three high-voltage wires that are 120deg out of phase with each other. I think motors can use both poles of each phase, for a total of six "pushes" per Hz. So in industrial areas, more capital is spent on the wires but less is needed for the big motors. for residential-type applications (motors below about 2hp), the cost differnce between the more expensive single-phase motors and the cheaper three-phase motors is small-to-nonexistent, so the whole thing makes economic sense. -Arch dude (talk) 23:18, 17 May 2009 (UTC)[reply]
With three phase, the torque is continuous. With single phase, (and maybe 2 phase, like Tesla's early installations) the torque pulses. Edison (talk) 19:02, 18 May 2009 (UTC)[reply]

Egg crystals?

I left a little bit of whisked egg in a bowl overnight in a warm, dry room, and it has formed what appear to be crystals, although they are a little cloudy (photo at http://www.benjamin-mills.com/photos/2009/egg-crystals.jpg). Is this really a crystalline solid, or just a kind of flaky amorphous solid?

Ben (talk) 16:39, 17 May 2009 (UTC)[reply]

Looks like a beautiful set of radial cracks caused by the drying out of the egg, like dessication cracks in mud. The radial symmetry comes from the shape of the bowl and the thickness of the layer. On a completely flat plate the cracks would have been roughly hexagonal, like in the basalt of the Giant's Causeway (though of course they're a result of cooling rather than drying). Mikenorton (talk) 17:27, 17 May 2009 (UTC)[reply]
I agree - not all geometric patterns are due to crystal lattice structures. This appears to be uniform fracturing due to dessication. Nimur (talk) 19:56, 17 May 2009 (UTC)[reply]

OK, thanks guys.

Ben (talk) 22:58, 17 May 2009 (UTC)[reply]
This also looks like the basis for a really good science-fair project for elementary school kids to demonstrate dessication cracks. -Arch dude (talk) 23:24, 17 May 2009 (UTC)[reply]

Little gas poker

What do you call the hand-held wand used for lighting candles and gas stoves? It has a liquid gas container and a piezo ignition. I thought it was a gas poker, but Googling suggests that that is a more heavy-duty device for starting fires. I want to find one on line but I don't know what they're called (in the UK.)--86.25.195.233 (talk) 17:55, 17 May 2009 (UTC)[reply]

Piezoelectric gas lighter or static electricity generator. Axl ¤ [Talk] 19:03, 17 May 2009 (UTC)[reply]
(e/c) Could it simply be known as a lighter? Lakeland (that nirvana of kitchen goods) currently has on sale something called a Handy Lighter which performs that function, although it doesn't mention what ignition method is used. Kitchen lighter also produces some relevant results on Google. --Kateshortforbob 19:09, 17 May 2009 (UTC)[reply]

What is the name of this bird?

Does anyone know the name of this bird, photographed in Austria? Thanks a lot... --Edcolins (talk) 18:48, 12 April 2009 (UTC)[reply]

It looks like a Common Blackbird. Axl ¤ [Talk] 19:17, 17 May 2009 (UTC)[reply]
From the size of the eye, its posture and the rather fluffy plumage at the base of the breast I'd go for an Alpine Chough. Mikenorton (talk) 19:29, 17 May 2009 (UTC)[reply]
Yes, you're right, Mike. The colour of the legs also fits. Axl ¤ [Talk] 19:32, 17 May 2009 (UTC)[reply]
Many thanks! Great! --Edcolins (talk) 19:44, 17 May 2009 (UTC)[reply]

HIV and preeclampsia

I have been trying to get a dissertation topic for Part II FMCOG relating to HIV and preeclampsia since 80% of antenatal patients I attend to are HIV positive. Please, could you suggest some topics.41.219.251.119 (talk) 20:51, 17 May 2009 (UTC)[reply]

LOL

I've just been watching a comedy show on TV. I notice that at the funny bits all the audience laughs at the same time, and rarely at other points. Are funny things just funny, or do we accept cues from other people that encourage us to laugh at the same time out of some kind of evolutionary advantage. When I watch comedies alone I often find that I never laugh througout the whole programme, but appreciate which situations are humourous. Also, I have heard that laughing benefits us in bearing our teeth, which is an aggressive response to a situation as seen in many primates. Your thoughts, please, russ (talk) 20:58, 17 May 2009 (UTC)[reply]

Most TV shows have a "laugh track" with recorded laughter, or a "live studio audience" including the writers and friends of the cast who laugh predictably at the appropriate places. In 1939s when NBC in the U.S. started their experimental broadcasting, there was a concern that theatrical movies would not play well on TV if comedies, because in the theater, as in live theater, there is a range of latencies for people to figure out the joke and laugh, and the movie actors would pause after a gag (as on stage) to allow for that before the action moved on. Laughing out loud may be something of a group phenomenon. The concern was that the solo home viewer would be put off by the resulting slow pace, but it would be less of a problem with group viewing. The "laugh track" can act as a filler to avoid the latency problem. It also helps the clueless understand that something funny happened. The humorist Mark Twain had a standard joke he used at the beginning of an appearance to measure an audience. Their latency in getting it and laughing told him whether to dumb down his material and use broad humor for a dullard audience, or to use his more demanding material. Edison (talk) 21:25, 17 May 2009 (UTC)[reply]
Most shows with live audiences employ applause signs to tell the audience when to applaud and when to laugh. The audience also never goes in "cold" - before the action starts the stage manager or someone similar will always stand in front of the live studio audience and give them directions on when it's appropriate to laugh, etc. Frequently for comedy shows, (especially the late night shows like the Tonight Show) they will also have a warm-up comedian* to get the studio audience in a good mood. (* Amazing - Wikipedia doesn't seem to have an article on warm-up act)-- 128.104.112.117 (talk) 21:52, 17 May 2009 (UTC)[reply]

We do laugh more openly when there are others to hear it. And as for the way primates bare their teeth, it is apparently more to do with a reassurance that you mean no harm. A fear grin.Popcorn II (talk) 22:40, 17 May 2009 (UTC)[reply]

I think we take cues from others, but I have some admittedly personal observation that shows that there are always those who are slower and faster, even if not warmed up. The warm-up guys are there mostly, I think, to get the slower ones up to speed. Otherwise, you'll have a situation like what happened with a friend. He's the stoic kind who always tries to keep his cool and usually only grins or chuckles. Well, we were all laughing at one of those silly B movies and making fun of it and he sits there reall quiet for about 20 minutes, then suddenly bursts into a laughing fit that's like all his laughter came out at once. He kept getting the cues, but something in his personality was holding him back, till all those cues caught up with him.Somebody or his brother (talk) 00:34, 18 May 2009 (UTC)[reply]
yes, there are laugh tracks and prepared audiences, but also laughter is strongly contagious. Looie496 (talk) 02:28, 18 May 2009 (UTC)[reply]
After a performance has been recorded with a live audience the sound track can be "sweetened" by adding laughter or applause to the genuine reactions. It is hard to detect when that is done skilfully but on occasions audience reactions seem suspiciously mistimed. Cuddlyable3 (talk) 13:11, 18 May 2009 (UTC)[reply]
You know that this laughter is totally faked in at least some shows because it's added to shows where no live audience is present - and it sounds exactly the same. I recall, for example, that when we watched M*A*S*H (TV series) in the UK, it was broadcast without the canned laughter. I couldn't believe it when I first came to the US and watched the exact same shows with canned laughter...it totally changed the show from something rather pointedly anti-war to just another sit-com. Weird. SteveBaker (talk) 13:33, 18 May 2009 (UTC)[reply]

Magic and illusion

I have several questions about magic: 1- What's the difference between magic and illusion?

2- Is their a true magic? I mean is it true what the media says and what is sometimes posted on wikipedia about magicians such as David Copperfield and the Liberty Statue? If so how can we explain it physically?

Thanks awfully..--Email4mobile (talk) 21:00, 17 May 2009 (UTC)[reply]

1. "Magic" generally refers to one of two things: illusionism, which is essentially the art of fooling people into thinking that they're seeing something that appears to be impossible or supernatural, whereas in reality it's all done with trickery and clever misdirection, or paranormal magic, which actually is supernatural -- that is to say, it actually violates the known laws of reality. The latter, it should be stressed, is something that a lot of superstitious people certainly believe in, but of which there exists no actual evidence. The former is illusion and the latter is delusion, you could say. (It's particularly important to understand that there's no evidence of someone actually performing supernatural acts of magic, but there are many, many instances of people claiming to have magical powers and being exposed as liars and frauds. That in itself doesn't prove that there's no supernatural magic, of course, but when there's no evidence for something and a lot of the people who claim that there is are exposed as liars, it doesn't exactly make an argument for its existence.)
2. I don't know what you mean when you refer to what the media says and what is sometimes posted on Wikipedia. I can tell you, however, that David Copperfield did not actually make the Statue of Liberty disappear, he just made it look like it did. That's an illusion. We don't know exactly how he did it, though, because like most magicians, Copperfield likes to keep his tricks to himself, but William Poundstone's explanation for how it was done (explained in general terms in the article I just linked to) seems fairly sensible. If that's how he did it, it's a classic example of misdirection, an illusionist's most important tool: the idea that Copperfield would actually move the audience instead of the statue without anyone noticing is just so audacious that most people would never even consider it! But it could certainly be done.
So no, I don't believe that there's any magic in the supernatural sense. There certainly isn't any hard evidence of magic -- just stories people like to tell, but as the Copperfield example above illustrates, even if people really believe in something, that doesn't mean it actually happened. It's easy to fool people when you know how, and in many cases, the more outrageous the trick, the more willing people are to believe it. -- Captain Disdain (talk) 21:38, 17 May 2009 (UTC)[reply]
Some think there's a difference, some don't. -RunningOnBrains 00:41, 18 May 2009 (UTC)[reply]
I think that one of the most convincing reason to disbelieve these 'magic' acts is that if the magician could truly perform them, he'd be able to do so in much clearer, testable situations. The industrial value of being able to make objects weighing hundreds of tons simply 'vanish' - or to have people who could fly or create coins out of thin air would be extensive and revealing the techniques involved would earn the performer billions of dollars. The fact that they restrain themselves to a simple stage presentation is very telling. All of this quite aside from the violations of scientific laws of all kinds that would be required. So we can be VERY confident in saying that (without exception) magic tricks are tricks - and most magicians are only too happy to admit that. SteveBaker (talk) 00:43, 18 May 2009 (UTC)[reply]
The method David Copperfield used to make Lady Liberty disapear is a known, if rather elaborate conquerors' trick. This Article explains it, if you really want to know. APL (talk) 02:26, 18 May 2009 (UTC)[reply]
I refer to the same method above (except in our own article), but our article says that it's speculation, not actual proof of how he did it. I agree that it's probably how it was done, but in the interests of the accuracy of our article -- is it actually sure that this is how it was done? -- Captain Disdain (talk) 07:41, 19 May 2009 (UTC)[reply]

Thank you very much, gentlemen for this valuable emphasizing. --Email4mobile (talk) 06:53, 18 May 2009 (UTC)[reply]

  • Magicians generally refer to large stage effects (tricks) using big boxes and large props as illusions, although some smaller tricks can also be called illusions. - Mgm|(talk) 11:30, 18 May 2009 (UTC)[reply]
As the aforementioned Gob puts it... "It's an illusion, Michael. A trick is something a whore does, for money." -- Captain Disdain (talk) 14:05, 18 May 2009 (UTC)[reply]

Lye in a swimming pool

(Caution: do not try this at home.) On CSI Miami, in Dissolved (CSI: Miami episode) someone added enough sodium hydroxide to a swimming pool to make the pH a bit above 12.3 (per the CSI Ph meter). How many kilograms of sodium hydroxide would have to be added to a home swimming pool of say 100,000 liter capacity to achieve that alkalinity? How many liters of supermarket vinegar (say 8% acetic acid) would it take to get the pH back down to 7.0? It did not look like they poured very many gallons of vinegar to neutralize the lyewater. Would the high alkalinity solution instantly break a glass beaker if poured into it as in the episode? Would concentrated lye water have a strong smell? Edison (talk) 21:16, 17 May 2009 (UTC)[reply]

NaOH is awfully strong, but dilution in 100,000 liters is no small effect. If my calculations are right, to make a pH of 12.3, you only need a molarity of 0.02, or about 0.8 grams of NaOH per liter of water. In a 100,000 liter swimming pool, that gives you about 80 kilograms of NaOH. More to the point, how did somebody add the NaOH? It's much more likely that it was not added as a pure solid, but added as a more concentrated liquid (like Lye). Since lye is only about a pH of 13, you would basically need to fill most of the swimming pool with it to reach a pH of 12.3. Nimur (talk) 21:51, 17 May 2009 (UTC)[reply]
Glass, especially chemical lab beakers made of pyrex, does not react with sodium hydroxide. Lye does have a distinctive smell, and if the pool were full of it, it ought to be obvious. Nimur (talk) 21:54, 17 May 2009 (UTC)[reply]
And, while we are debating the validity of the scientific events in a TV show whose extent of scientific reality falls somewhere between "just making shit up" and "faking it", droping that much NaOH into a swimming pool will likely be exothermic enough to boil the water, and spatter everyone in the area with a nice boiling, skin dissolving solution of sodium hydroxide. Yum, yum. --Jayron32.talk.contribs 23:04, 17 May 2009 (UTC)[reply]
Nimur, the paper-thin things I've accidentally left in base-bath overnight or flasks etched with melted NaOH would disagree.[original research?] Now of course that's not how the typical person would use such a material:) DMacks (talk) 02:33, 18 May 2009 (UTC)[reply]
That analysis only works if the pool water is not buffered. If the owner has added a pH buffer to the pool water, or simply lives in a region where the tap water naturally contains buffering agents, you'll need to add additional NaOH to overcome the buffering effect. How much extra is hard to say, as water buffering capacity is usually measured by alkalinity, which assumes that you're adding acid, not base. -- 128.104.112.117 (talk) 18:16, 18 May 2009 (UTC)[reply]
Why would they use such a weak acid as vinegar to neutralise it? You can get "Muriatic Acid" (weak Hydrochoric acid) at pool supply stores - it would be much better at the job than vinegar! SteveBaker (talk) 00:19, 18 May 2009 (UTC)[reply]
Red Devil is nearly pure crystalline sodium hydroxide, you can buy it in any grocery store, and it's cheap -- on the order of a dollar a pound to the best of my recollection. So based on the numbers above you could probably do it for around $500 or less. Looie496 (talk) 02:43, 18 May 2009 (UTC)[reply]

Saturated NaOH will etch glass over time, but does not react violently. And making a pH 13 solution of NaOH by dumping pure solid into water is nowhere near exothermic enough to cause boiling (unlike sulphuric acid!), possible just because it takes to long to dissolve. A higher concentration of a weak acid probably would be just as good for such a neutralisation, as the act of neutralising would pull the equilibrium to one side, so the lower dissociation constant in water is not a factor. Was the pool being used to kill people, or just dispose of bodies? Pure NaOH is corrosive, but can still be picked up without gloves (not recomended though!), so i'd guess wouldn't kill people quickly enough.YobMod 08:00, 18 May 2009 (UTC)[reply]

One cubic meter is 1000 liters, so a pool 10mx10mx1m would have 100,000l, as would one 10mx6mx1.5m (well, close enough). Large, but not impossible for a home pool.--Stephan Schulz (talk) 12:14, 18 May 2009 (UTC)[reply]
Yeah - our home pool is 26,000 gallons - that's 98,000 liters. It's not by any means a huge pool - if you had a part that was deep enough for diving (which many of my neighbours do) then it could easily double that. SteveBaker (talk) 13:21, 18 May 2009 (UTC)[reply]
On the show, they neutralized the lye with a few gallons of vinegar. But a lab beaker shattered when the lye solution was poured in it and a live human was "boiled" to death. Edison (talk) 18:59, 18 May 2009 (UTC)[reply]
Meh. I use 3 M NaOH on a daily basis, which is 15 times as concentrated as here. Spilling it on the skin has no effect, except the usual soapy feeling from bases (not even peeling the next day, like most acids), so i very much doubt it could kill someone unless they are held in it for a while. And the 3 M solution i use is poured from Schott bottles to measuring cylinders to beakers - none of them ever break or even crack slightly. Overall seems a poor murder method indeed!YobMod 07:38, 19 May 2009 (UTC)[reply]

May 18

Dry skin itch

Why, precisely, does dry skin cause itching? What physical or chemical factors are involved? 69.224.113.202 (talk) 02:22, 18 May 2009 (UTC)[reply]

OR: What causes itching is people asking about itching on the Ref Desk - damn you! --Tango (talk) 02:26, 18 May 2009 (UTC)[reply]
Desquamation. I would say that the itching is an allergic reaction to dead epidermal cells, but researchers put it like this. Cuddlyable3 (talk) 11:45, 18 May 2009 (UTC)[reply]

Evidence of a civilization a billion years ago

Just a thought exercise: Assume that a civilization existed a billion years ago (and continued for about the same length of time as our own achieving roughly the same level of technology that we now have). Then, for some mysterious reason, this civilization disappeared. My question is: would there be any evidence that this imaginary civilization ever existed? —Preceding unsigned comment added by 209.161.213.227 (talk) 04:18, 18 May 2009 (UTC)[reply]

Well, sure. There'd undoubtedly be some physical artifacts left, even though much of it would be destroyed by the ravages of time. They'd show up in the fossil record, one way or another. More importantly, the ecological impact would probably be detectable. (A similar civilization couldn't have existed back then, though, because at the time there was really no ecosystem that civilization could've had an impact on, so the parameters would just have to be different.) I don't know if it would be anywhere near enough for anyone to actually be able to tell what the civilization was like, but I think there'd be evidence of its existence if one knew where to look and how to interpret what they were seeing. Actually, come to think of it, perhaps the most obvious piece of evidence would be that life on Earth would be completely different from what it is now: whatever bacteria and whatnot those people would leave behind would be the basis for our own life. Our primordial soup could literally be someone's leftover soup. -- Captain Disdain (talk) 09:42, 18 May 2009 (UTC)[reply]
Anything 'they' might have had or made back then would not likely be visible on the surface of the planet. Everything would be way underground buried deep in rock, as would any fossils of the creatures which had this civilisation. Life was around at that time, and had been around for at least 1.5 billion years before that, so it is certainly not impossible that some creature or another developed some sort of civilisation, which then got wiped out in some way or another. Changing the time frame to just 500 million years ago, we could say the same. It would, however, be very hard to detect, unless you knew what you were looking for. On a related note, you may want to look at this and see what you make of it. --KageTora - (영호 (影虎)) (talk) 11:13, 18 May 2009 (UTC)[reply]
Suppose the ancient civilisation having technology similar to ours did the same things (why not?). Expect aliens to arrive about now, attracted by the Arecibo message and asking if we want our probes back, like this. Cuddlyable3 (talk) 11:21, 18 May 2009 (UTC)[reply]
As the oldest fossils are "several billion years old", it seems likely that some evidence of human life would certainly remain. As many man-made objects are less susceptible to decay than living organisms, this would present evidence for our technological achievments (even simple mass-produced trash is dependant on a huge infrastructure). This, along with the planet-wide climate effects would make deduction of a widely-spread technological society pretty conclusive, imo.
Off course, none of this necessarily applies to any past civilisation, as they could easily have been entirely arable/biodegradable. Civilisation does not require an industrial revolution.YobMod 11:28, 18 May 2009 (UTC)[reply]
Having said that, there are a number of "ooparts" which bear consideration. I'm not saying I am a believer, but it would be interesting if we could find an explanation for them which fits our current 'knowledge' of history, or maybe we would have to rethink what we 'know'.--KageTora - (영호 (影虎)) (talk) 11:41, 18 May 2009 (UTC) I fixed the oopart link. Cheers.Cuddlyable3 (talk) 12:32, 18 May 2009 (UTC)[reply]
Cheers. I wondered why it wasn't working.--KageTora - (영호 (影虎)) (talk) 12:36, 18 May 2009 (UTC)[reply]
The book The World Without Us and the Discovery channel documentary that comes from it: "Earth Without People" follows the unlikely scenario where all humans suddenly vanish. That's not a very likely one - but if there were some kind of mass extinction due to (say) a massive meteor strike - then the story it tells of what remains of us after thousands of years is applicable - and it's pretty chilling. Aside from artifacts left on the moon, mars and out in deep space - there doesn't seem to be much that would survive. If that's true - then it might very well be that a sufficiently ancient civilisation would be very hard for us to detect. The fact that we find fossilised bones from the time of the dinosaurs (for example) leaves us with the impression that we'd also find fossilised Internet Routers and other indicators of civilisation. However, in reality, fossils are extremely rare - if a civilisation like ours were only to last (say) 100,000 years - then the probability of finding something within that window would be pretty tiny...the fossil record simply isn't that reliable. We have actually discussed this question before - and my opinion remains that the best place to look for such evidence would be out in space at places called Lagrange points - which would be a natural place to leave things that you wanted people to find a hell of a long time into the future. 13:16, 18 May 2009 (UTC)
Are even L-points stable on scales of a billion years? --Tango (talk) 14:54, 18 May 2009 (UTC)[reply]
I'm reading a book that depicts a similar scenario where aliens come to visit Earth 200 million years from now...and the only evidence they find of a past civilization is on the Moon (but it is difficult to tell if the artifacts were left there from a civilization on Earth or from another intergalactic traveller). Also, it really isn't a mystery why past ancient civilizations collapsed, leaving behind their ruins as if they disappeared suddenly: the biggest factors are war, famine, environmental depletion (think Easter Island), and climate change. The Arecibo message was aimed at the globular cluster M13, but it would take about 50,000 years for any reply message to reach Earth, assuming that the message actually arrives at its target, which it probably won't, because the motion of the cluster around the galaxy would likely have carried it away from the path of the message by then. ~AH1(TCU) 16:06, 18 May 2009 (UTC)[reply]
A billion years ago all life was microscopic life, with stromatolites about the only macroscopic evidence of life. So, essentially any macroscopic fossil would stand out as evidence of the unexpected. If you are positing a civilization of people that spring up out of nothing (e.g. aliens landing on the Earth) and lasts for only 10000 years, then the duration of their existence and their spread might well be very hard to find in the fossil record. On the other hand, if you are imagining a civilization that arises organically, then it is hard to imagine doing that without developing a wider ecology of macroscopic life. For example there are far more trees on Earth than people. A naturally developed macroscopic ecology would be expected to have taken many millions of years. If you include all of those associated forms of life and their long history, then it would be shocking not to have some evidence of that in the fossil record. If we assume macroscopic life is a prerequisite for civilization, then I think we can rule out the development of civilization a billion years ago. Dragons flight (talk) 17:28, 18 May 2009 (UTC)[reply]

coefficient of volume elasticity

is there anything called coefficient of volume elasticity?? —Preceding unsigned comment added by 122.50.137.137 (talk) 05:10, 18 May 2009 (UTC)[reply]

I don't think there is anything called volume elasticity and I am sure there is nothing called coefficient of volume elasticity - DSachan (talk) 11:16, 18 May 2009 (UTC)[reply]
Possibly you are thinking of the elasticity tensor or one of the elastic moduli such as the bulk modulus. Gandalf61 (talk) 12:51, 18 May 2009 (UTC)[reply]
Here is a definition of the coefficient of volume elasticity. The term is also used in biophysics, as in this abstract. Isn't Google great? Looie496 (talk) 18:44, 18 May 2009 (UTC)[reply]

dimensional analysis

my teacher set me this question:

using dimensional analysis, check whether the relation is correct.

Snth = u + a/2(2n - 1)

where the symbols have their usual meanings

i know that u stands for initial velocity and a for acceleration, but what about n and Snth??? —Preceding unsigned comment added by 122.50.137.137 (talk) 05:16, 18 May 2009 (UTC)[reply]

This is a formula in Kinematics ( study of spped, distance, tima and acceleration ) for the distance traveled in the nth second. Say you are dropping a ball from a cliff. If it has fallen 5m after 1s, 20m after 2s, and 45m after the third second, the distance travelled in the first second is 5m, the distance travelled in the second second (ignore the pun) is 20 - 5 = 15m, and the distance travelled in the third second is 25m. So your Snth has the same dimension as S, distance. n is the number of seconds, so you should be able to tell its dimensions. Also don't forget to sign your posts by typing 4 '~'s in the end...Rkr1991 (talk) 05:29, 18 May 2009 (UTC)[reply]

I don't like this business of "where the symbols have their usual meanings" because 'n' isn't often used as a symbol for 'time' and Snth isn't a "usual" symbol at all (although 'S' is conventionally used for a distance travelled). That makes this question very hard to answer definitively.
Does the constant '1' have a dimension (like '1 second' or '1 hour')? No - because if 'n' is a time - then (2n-1) is only dimensionally correct if '1' is also a time. As Rkr1991 explains it, the equation is only valid if the units happen to be seconds. If you were to provide 'n' in hours - then it would tell you the the distance travelled in the nth hour...but only if that magic '1' constant is now assumed to be in hours.
But as Rkr1991 explains it, there is a 'get out' clause here. If the equation is truly stated to describe: the distance travelled "in the nth second" then that leaves open the possibility that 'n' is "the number of seconds elapsed" - not "the elapsed time" (a subtle distinction that leaves n dimensionless and makes (2n-1) work). Using 'n' as a dimensionless number is certainly reasonable here because 't' would be the more usual symbol to use for a 'time'...but then the dimensions of 'u' and 'a/2(2n-1)' don't match anymore because u is a velocity (distance.time-1) and a is an accelleration (distance.time-2) so (2n-1) MUST be in units of time or else you can't add u to a/2(2n-1) legally. So that says that the units of 'n' MUST be time - and that constant '1' has to be in the same units as 'n' - which is unspecified in the equation. Furthermore if Snth has units of distance - then the equation fails again because 'u' is a velocity and 'S' is a distance. If 'a' were zero then you'd have 'Snth = u' - which doesn't work if S has units of distance. But if we believe Rkr1991's description of what the equation means then "Snth is the distance travelled over a second" - which is a velocity - not a distance.
I think the equation is dimensionally incorrect - although the way Rkkr1991 explains it, there is some wiggle room. It would produce a meaningful answer providing that 'n' is defined as 'the time in seconds' and not 'the time' - and providing that '1' is really '1 second' and that Snth can truly be construed to be a velocity. I think the question could have been better phrased - but the answer should be "No". SteveBaker (talk) 13:00, 18 May 2009 (UTC)[reply]
I agree with you, but to play devils advocate - the 2 could have units of time, and the (2n-1) not. The presence of constants in things like this always creates room for confusion. --Tango (talk) 15:00, 18 May 2009 (UTC)[reply]
Is this equation meant to be a discrete time version of kinematics? I really really don't like this "symbols have their normal meanings." That's a very meaningless statement. In the papers I've been reading lately, s is "slowness" (reciprocal velocity). And a is the anisotropy vector. And n is the Normal Move Out operator, or it can be almost anything else. You should verify the meaning of all of these variables, or you're really lost. "Conventional" variable names are only relevant in very specific contexts. Nimur (talk) 15:55, 18 May 2009 (UTC)[reply]

Well let me tell you that this equation is perfectly correct, both dimensionally as well as physically. Snth is the distance traversed in the nth second, not the average velocity. Note the distinction. So it has dimension L. The only trick in the question is the first term on the right hand side. It is not velocity. It is Velocity seconds. Velocity multiplied by one second. This will have been given in the question, because it is not the usual meaning. n is a number, dimensionless. So now the rest should be obvious. Though it is not mentioned here, i'm quite confident the teacher would have mentioned that u is velocity seconds in class. This is a standard equation for distance traveled in the nth second, so it is obviously right. Rkr1991 (talk) 05:27, 19 May 2009 (UTC)[reply]

The notation is far from standard, so I'm guessing "usual meaning" means usual meaning in that class, so we can't really tell if the question is right because we don't know the notation. Sure, we can come up with interpretations that make it correct, but that doesn't mean those interpretations are the "usual" ones. --Tango (talk) 15:52, 19 May 2009 (UTC)[reply]
Well, Rkr1991 can hypothesise that our OP has miscommunicated the question to us - but for absolute 100% certain, 'u' is not "velocity seconds" in it's "usual meaning" in the context of dynamics/ballistics questions. The usual meaning is found in equations like:
S = ut + at2/2
...where the meaning of 'u' is "the initial velocity"...and our OP even tells us that explicitly. If the meanings of these terms are not "the usual ones" then all bets are off. 'u' could be the phase of the moon, 'a' the dissolution rate of resublimated Thiotimoline, 'n' the airspeed an unladen swallow and 'S' the amount of wood a woodchuck could chuck if a woodchuck would chuck wood. So either our OP has the question wrong - or the question was written incorrectly - or the answer is "No". Pick any one - but the answer isn't "Yes". SteveBaker (talk) 20:24, 19 May 2009 (UTC)[reply]



The reason i say the equation is dimensionally right because of two reasons : (i) Any equation which is right is also right dimensionally. You can derive the given equation yourself, writing the formula S = ut + at2/2 for t = n seconds and t = n+1 seconds and then subtracting. Note that the n seconds part cancels out in the first term, which is why u here represents velocity seconds, not velocity. This equation is pretty much standard, and it would be against my senses to call it wrong just because the teacher has told usual meanings. (ii) The OP has said usual meanings. For this particular formula, this is the usual meaning. Now, since the dimensions of each quantity has not been explicitly stated, there is a very high chance that the teacher has actually mentioned this in class. Ya, you guys are right, for the exact question the OP has asked, the answer is NO, but i am pretty confident that u here refers to velocity secons and NOT velocity. So if this question was asked to me in an exam, i would right YES as the answer. Having known a formula, my instincts bristle at having to write it as not correct. But i think this question really doesn't require so much debate. You got to ask the teacher what she meant by u. The other confusions, regarding n and 1, should be quite obvious by now. n is a number. So is 1. Rkr1991 (talk) 04:52, 20 May 2009 (UTC)[reply]

I disagree. If you derive the equation the way you suggest then 'n' has the dimension of time and the '1' in(2n+1) is really 1 second - I don't see a unit against that constant in that equation - which means that it's dimensionally incorrect...not because the equation is incorrect per-se but because the person who wrote it down omitted the 'time' dimensionality for the constant '1'. You certainly can't argue that the "usual meaning" of the symbol '1' is "1 second" - so the equation is quite simply wrong. If I take it as written and choose 'hour' as the unit of time for 'n' and whatever contorted units you then arrive at for 'S', 'u' and 'a' then it produces the wrong answer. It's a fundamental precept of dimensional analysis that you can't add a dimensionless constant to something that is not itself a scalar. This equation (as written) is therefore incorrect...and I'm quite sure that's what the teacher expects you to point out. SteveBaker (talk) 11:58, 20 May 2009 (UTC)[reply]

Can you please explain how n will refer to time here ? Remember the formula is for distance in the nth second. n here is clearly a number. When you substitute t in the equation, do not make the common mistake that t is n. t is n seconds. Catch the subtle difference. But to accept whether the equation is right dimensionally or not lies with each individual, but certainly it should be clear that n and 1 are numbers. Rkr1991 (talk) 12:35, 20 May 2009 (UTC)[reply]

'n' has the dimension of time because of the way you derived this equation. Look above - you said it yourself: "You can derive the given equation yourself, writing the formula S = ut + at2/2 for t = n seconds and t = n+1 seconds and then subtracting. - you can ONLY substitute t=n and t=(n+1) if 'n' (and '1') is in the same units as 't'. Hence if 't' is time - then so is 'n' - and so must be '1'. So your derivation of the equation is wrong unless both 'n' and '1' have time as their dimensions. You may be about to claim that you aren't setting t=n but t=nseconds - thereby making 'n' into a scalar - but that's just introduced a non-scalar constant so that 't=n.k' where k=1second (it has to be like that in order to make your 't=n' substitution be dimensionally correct) - but then the problem is that when you do the subsitution, you get "S = u.n.k + a(n.k)2/2" - which would leave the equation our OP gave us littered with k's which would be carrying these '1 second' constants and making it dimensionally correct...but it most certainly doesn't have those...which is why it's incorrect. SteveBaker (talk) 18:16, 20 May 2009 (UTC)[reply]

Well, i'm sorry, but there appears to be a lot of confusion over nothing. Lets forget for now whether the equation is dimensionally right or wrong. Lets just analyze n and 1. First question. What is this formula for ? Ans : It gives the displacement covered is the nth second of motion. That's it, I'm through. Just think about it, how can n possibly be in seconds ? The whole formula wouldn't make any sense. Second, think physically. As i had explained in my very first post, you substitute t as n seconds. Now you tell me this formula would be littered with ks. Unfortunately, while writing equations, we don't have the practice of writing units along with the numbers. You just write the numbers. So in the formula you just substitute k = 1, assuming it is in seconds. You might argue now that see it is not the usual convention. But now you're coming to analysis of the whole equation. Personally, i think this is a standard enough formula to be asked as a trick question, but its up to the teacher, there's really not much point in arguing about that. But remember, the whole physical meaning of the formula is lost if you put t = n. When the question is how many meters has the ball traveled in the 5th second, then you must substitute n as 5. There is no meaning in substituting n as 5 seconds. All the ks are left out because of conventional convenience. Another earlier confusion was whether Snth is distance or velocity. Its distance, as can be clearly seen from the physical meaning of the formulaRkr1991 (talk) 03:39, 21 May 2009 (UTC)[reply]
Let me just Give an example. Say there are a set of 5 vectors. I want to know the magnitude of the 5th vector. Is 5 a vector here ? Or is it just a number ? Think about it in relation to this question. Rkr1991 (talk) 04:59, 22 May 2009 (UTC)[reply]

Brain health

Is there any explanation for a condition that brings a headache when a person is studying or trying to focus on a taskBotosh (talk) 05:48, 18 May 2009 (UTC)[reply]

Eyestrain causes headaches, and can be caused by "tedious visual tasks". -RunningOnBrains 07:01, 18 May 2009 (UTC)[reply]
See Tension headache.71.236.24.129 (talk) 09:40, 22 May 2009 (UTC)[reply]

Houseplant identification please...

I've looked on the internet and on W, and the best I can do is narrow these down to Succulents. I understand that's not much help, but I'm an amateur ornithologist, not a botanist.

Just bought this pair today in China and they were completely unlabeled...

Thank you!

succulents maybe?

--61.189.63.170 (talk) 07:04, 18 May 2009 (UTC)[reply]

try Kalaonchoe (spelling?) Sorry, gotta run. 67.193.179.241 (talk) 07:51, 18 May 2009 (UTC)Rana sylvatica Spelling Kalanchoe[reply]

Aloe aristata? --Dr Dima (talk) 08:25, 18 May 2009 (UTC)[reply]
More probably a Haworthia species, pretty. To the right is an Agave, I think. --Ayacop (talk) 17:02, 18 May 2009 (UTC)[reply]

Earth Rotation Effects

I asked this question few weeks ago but couldn't get an answer, so I will rephrase my question in another way:

If a rocket is initially launched (or an object thrown at very high initial speed) normal to the Earth (Vertical) at the equator for example then what would be the path it flows with respect to its origin on Earth (recall that earth is spinning and the rocket was initially having the same spin)? I'd appreciate it if the governing equations are included.--Email4mobile (talk) 07:21, 18 May 2009 (UTC)[reply]

Sorry, doctor said no equations this late at night for me :-D
In all seriousness, a rocket which appears stationary to an observer in the rotating frame has an initial tangential velocity v=[omega]xr (I never did learn how to make wikiequations). Thus if you add thrust normal to the surface of the sphere, its velocity will be v=at^2+v_0, with the a being in the, say, x-direction and v_0 being that initial tangential velocity in the y-direction. A few considerations when considering Earth: add an atmosphere and you have drag accelerations as well, and don't forget that "normal to the surface" isn't always necessarily what you think of as "up": Depending on latitude, the sum of the gravitational force and the centripetal "force" will not point to the center of the earth, but rather a small, but significant deflection away from the center.-RunningOnBrains 07:46, 18 May 2009 (UTC)[reply]

Thanks -RunningOnBrains, but I'm only curious about this stage when the rocket or any trajectory is being thrown 90o normal to the Earth surface and assuming ideal conditions where no air effects or any other kind of friction. This is because my original question was how the Earth rotation or spin would affect the released bodies from its surface (i.e. how it would be different from doing the same experiment at the Arctic or Antarctica). Indeed this question arose when a friend of mine was reading an entertainments physics book and asked us the following question: "Assume you're ideally jumping up and down, continuously at the Equator, can you slip from the Earth surface due to its spin? " His answer according to that book was No, never! But to me it wasn't convincing because I analyzed it starting with the initial conditions (same spin velocity and same angular speed) and so I realized their must be a slight slip in every jump due to the increase in the new radius (Earth's radius + height). When I used simple equations and approximations I concluded a full circle slip every 58000 years if the jump is around 0.5 m height (I eventually reached this formula, :: , where Tr is the required time for full circle slip, T is the normal 1 day time, Re is the standard Earth's radius and hmax is the max height of the jump. For this reason I wanted some one to help me justify this answer. --Email4mobile (talk) 08:25, 18 May 2009 (UTC)[reply]

OK - so you have an equation - what's the question? Do you not trust your equation? If your math is right - then the amount of 'slip' is proportional to the radius of the earth divided by the height the object reaches - which certainly fits with the idea that the effect is essentially negligable for things like people jumping and rocket launches. When you add in the atmospheric drag effects, then with a perfectly calm atmosphere, it would act such as to reduce the effect you are describing - when you add in wind speed, then the randomness of the result will completely dominate the results. 12:28, 18 May 2009 (UTC)
I'm sure the OP is aware about those caveats. He wants to confirm his calculations. I got an extra factor of 1/2.
.
But I didn't double check it, so I may be the one that made a mistake. Dauto (talk) 14:55, 18 May 2009 (UTC)[reply]
Conserve angular momentum. Roughly speaking, the stationary rocket begins on the ground, "orbiting" with a circular orbit equal to the radius of the Earth. When you add an impulse to kick it to a higher orbit ("flying vertically from the surface of the earth"), you have not changed its angular momentum, but you have moved it to a new orbital radius. You will find that the rocket is now traveling in an elliptical orbit which intersects the surface of the earth (crash!). Eventually, the rocket will return to the Earth and crash, unless you give it sufficient velocity to escape orbit. If you wanted to get into a useful orbit, you would need to add some angular momentum, which can only be done with a non-vertical launch trajectory. (Also, you should account for atmospheric drag, which will have an important effect on the orbit anywhere near Earth). Nimur (talk) 16:04, 18 May 2009 (UTC)[reply]

name that snake

Near San Francisco Bay, my cat killed a little snake, about a foot long, brown back, light beige belly. Should we worry? —Tamfang (talk) 07:49, 18 May 2009 (UTC)[reply]

This site and this site can help you a great deal in the identification of the snakes of California. Hope this helps. I'm actually curious what species it is. --Dr Dima (talk) 08:32, 18 May 2009 (UTC)[reply]
I wouldn't worry. I'm sure the snake didn't feel a thing :) --KageTora - (영호 (影虎)) (talk) 11:00, 18 May 2009 (UTC)[reply]
Alas that I didn't photograph it. The leading candidate is rubber boa. —Tamfang (talk) 17:10, 18 May 2009 (UTC)[reply]
Are you worried about the snake (species) or the cat or your family/self? Nil Einne (talk) 12:56, 18 May 2009 (UTC)[reply]
Yes, maybe it's just a phase the cat's going through. She'll grow out of it. Just stop playing GTA IV in front of her.--KageTora - (영호 (影虎)) (talk) 13:39, 18 May 2009 (UTC)[reply]
He is four years old and I think this is the first prey he has brought home other than leaves. —Tamfang (talk) 17:10, 18 May 2009 (UTC)[reply]
Mostly about the cat. It's futile to worry about the safety of vermin in the city. —Tamfang (talk) 17:10, 18 May 2009 (UTC)[reply]
Well I would question whether a snake in what is potentially its native habitat and which from the discussion appears to be non-posoinous is unlikely to, for example, get into your house and chew your food etc and is not know as a carrier of disease can be called a vermin especially since the snake may help to kill rats and mice which may get into your house and chew your food and are known as carriers of disease. And definitely it seems to me it's worth worrying about endangered snake species (I don't know if this one is). As for the cat, well to be honest if you're asking on the RD by the time you get an answer your cat is likely to be either dead or out of danger. If your worried your cat may make a habit of catching snakes, well to be honest as with birds, there's not so much you can do. You could try putting a bell on your cat but I don't expect it will help much Nil Einne (talk) 05:54, 19 May 2009 (UTC)[reply]

There are only a few venomous snakes in California and they are all pretty unpleasant (rattlesnakes—relatively large, mean-looking, lots of warning signals, don't generally live near populated areas) —I doubt your cat would try to mess with them. The smaller snakes cannot do any damage to your cat. Psychologically, your cat is both having fun and trying to please you. There's no reasoning with it about such things. --98.217.14.211 (talk) 18:39, 18 May 2009 (UTC)[reply]

Don't rule out a house cat going after a rattler. For example check this out [24] (Otherwise I can only offer OR.) Males seem to be generally less inclined to do so, but females, particularly the unfixed ones, will tackle snakes. As with other prey there seems to be a size limit, but that can get thrown to the wind if a litter of kittens gets threatened. The good news is that by the time kitty brings one home it is very unlikely to still be alive. (Other than more harmless trophies like mice or rabbits.) Look at the bright side, you'd probably rather have your cat deal with snakes than meeting them yourself while they are alive. BTW. I've never heard of a house cat that got killed by a rattlesnake, so they seem to know what they are doing or stay away if they don't. 71.236.24.129 (talk) 23:05, 18 May 2009 (UTC)[reply]
I did lose a cat to unknown causes but a snake bite was a potential candidate. It died frothing at the mouth. This was in Malaysia however so no rattlers, it could have been a python or something. This ref [25] suggests rattlesnake bites are definitely not unheard of and often fatal but this site seems to suggest rattlesnake bites don't usually kill cats [26] and this one is in between [27] Nil Einne (talk) 05:54, 19 May 2009 (UTC)[reply]
Pythons do not have venom, they are constrictors. 65.121.141.34 (talk) 13:09, 20 May 2009 (UTC)[reply]

8D type batteries

Explain about 8D type Lead acid batteries —Preceding unsigned comment added by Kumar3214 (talkcontribs) 09:30, 18 May 2009 (UTC)[reply]

Explain what? Perhaps you would care to read our article on lead-acid batteries? -- Captain Disdain (talk) 09:47, 18 May 2009 (UTC)[reply]
8D is a standard size of lead-acid battery for marine use [28] [29] that fits within a space 11-1/2 x 21-1/4 x 12-3/4 (height including cables) inches[30]. Do not confuse 8D with 8 D-cell batteries. Cuddlyable3 (talk) 11:04, 18 May 2009 (UTC)[reply]
I don't think D-cell batteries come in lead-acid form. Hence our OP is certainly talking about the 8D marine batteries. These appear to be pretty serious bits of equipment. Prices range from $250 to $700 and those seem to assume you're returning an old battery for recycling - otherwise they stick you with another $70 charge. They look a bit like longer car batteries - but according to the various web sites that deal with them, they have all sorts of specialisations such as vibration protection. Google "8D marine battery"...there's a ton of information out there. SteveBaker (talk) 12:17, 18 May 2009 (UTC)[reply]

Name that fluffy white lint bug!

I'm looking for an ID on this lint-like bug. I'm not the photographer by the way. Thanks. 152.16.223.48 (talk) 18:21, 18 May 2009 (UTC)[reply]

A location would be great if you have one.Popcorn II (talk) 19:37, 18 May 2009 (UTC)[reply]
The photograph was in southeastern Ohio, and I've seen them in North Carolina's Appalachians. 152.16.223.48 (talk) 21:01, 18 May 2009 (UTC)[reply]

It is an Eriosomatid, a kind of aphid. We have article on Eriosomatinae. --Dr Dima (talk) 20:37, 18 May 2009 (UTC)[reply]

An after-thought: Eriosomatidae family is better known as Pemphigidae. I'll update the Wiki pages for Eriosomatinae and Eriosomatidae if no objections land on my talk-page today. --Dr Dima (talk) 21:08, 18 May 2009 (UTC). Done. --Dr Dima (talk) 20:26, 19 May 2009 (UTC)[reply]

I'm not so sure. It doesn't look much like an aphid in my opinion. The antennae make it look more like a moth to me - no idea whatsoever what type though I'm afraid! Smartse (talk) 23:54, 18 May 2009 (UTC)[reply]
No, Smartse, it is definitely an Eriosomatid (a "woolly aphid"). The picture on the Eriosomatinae page is not so good though, that's probably what's confusing you. I wish we could use the Original Poster's picture... In the meanwhile, you can look at the pictures of Eriosoma sp. at this site. --Dr Dima (talk) 00:12, 19 May 2009 (UTC)[reply]
Here is the original image. I suppose someone could ask the user for wikipedia appropriate rights. Sifaka talk 01:44, 20 May 2009 (UTC)[reply]

Radios in tunnels

Why does my car radio continue to work all the way through the Rotherhithe Tunnel? It usually soon dies to static in much smaller motorway tunnels. Do they have a leaky feeder or something down there? Why do you need to know what I was listening to to answer the question? Radio 4 on FM if you must know. SpinningSpark 19:22, 18 May 2009 (UTC)[reply]

"Drivers will be able to listen to normal radio programmes including BBC and commercial radio programmes on FM, AM and LW and also on the increasingly popular DAB (Digital Audio Broadcasting) channels while travelling through the [Rotherhithe] tunnel." -- Press release. Cuddlyable3 (talk) 21:46, 18 May 2009 (UTC)[reply]
I bet there are few of the drivers using that tunnel in rush hour who realise how much technology went into that, if they even noticed anything at all. SpinningSpark 22:52, 18 May 2009 (UTC)[reply]
That is a sign of good technology - people not noticing that it exists. --Tango (talk) 00:23, 19 May 2009 (UTC)[reply]

fuel stability

In reference to storing fuel, Our article on gasoline states that it will go stale in time frames of 3-24 months. How long can you store Avgas before it will go stale? 65.121.141.34 (talk) 19:43, 18 May 2009 (UTC)[reply]

According to this page, the answer is 1–2 years, but that doesn't look like a very reliable source. Bovlb (talk) 22:02, 18 May 2009 (UTC)[reply]
The reason regular gasoline 'goes stale' is because the lighter fractions evaporate to a greater degree than the heavier stuff - but it's the lighter fractions that produce much of the engine power. Avgas has more of the lighter fractions to start with - but if you're going to use it to fly a plane, then you need that. So it doesn't really surprise me that the times are comparable. SteveBaker (talk) 01:22, 19 May 2009 (UTC)[reply]
It would depend on how big a container it is in and how well it is sealed and whether there is oxygen or just pure nitrogen filling the rest of the space. Well anyway I don't believe if it is well sealed that it will go off it is just that it is very volatile so you need a very good container or a huge amount to keep it a long time. I dfon't know what the times quoted are for but I'd guess it's for those small containers people sometimes carry petrol around in rather than tankers. Dmcq (talk) 10:10, 19 May 2009 (UTC)[reply]

Does space feel cold?

I recently got into a discussion with some friends about enduring the vacuum of space. I mentioned that this article states that, because of the lack of a surrounding environment, heat doesn't transfer out very quickly, so freezing to death isn't an "immediate risk". This makes sense, but what then does outer-space feel like, if there's little transfer of heat going on? -- MacAddct1984 (talk &#149; contribs) 21:39, 18 May 2009 (UTC)[reply]

There is a significant pressure difference between the outside pressure your body is adjusted to and the vacuum of space. My guess would be that your body would use existing sensors for heat/cold and pain to signal that. Extreme pain is often felt as extreme chill. A 'burning" sensation is also possible, though. 71.236.24.129 (talk) 22:28, 18 May 2009 (UTC)[reply]
One of the few to have experienced this was Joseph Kittinger during Project Excelsior when his hand was exposed to low pressure. His main reported symptom was severe pain due to swelling. See also Vacuum#Effects on humans and animals. SpinningSpark 23:30, 18 May 2009 (UTC)[reply]
That's not really comparable; the rest of his body was pressurized. Essentially the pressure on the rest of his body was trying to squeeze his insides out through his hand. With full-body exposure to vacuum, obviously you have other problems, but you don't have that one. --Trovatore (talk) 23:47, 18 May 2009 (UTC)[reply]
Keep in mind, I'm asking specifically about the perceived temperature of space, ignoring, if it's even possible, the other effects, such as a the body's reaction to a vacuum. I'm thinking a true vacuum is temperatureless? -- MacAddct1984 (talk &#149; contribs) 00:06, 19 May 2009 (UTC)[reply]
That's correct, the temperature of a true vacuum is undefined. Space, even without the few particles dotted around, is not a true vacuum though, since it is filled with radiation. It is possible to define a temperature of that radiation (for example, the cosmic microwave background radiation has a temperature of about 3K). --Tango (talk) 00:14, 19 May 2009 (UTC)[reply]
A quick google search turned up Phil Plait's article[31] which states:

Space doesn't feel like anything, because there is nothing to feel! Space is a vacuum (or near enough). It's a common question to ask how hot space is (or how cold), but space itself has no temperature. However, the Sun is hot, and radiates that heat in the form of light. You absorb that light and feel heat. Near the Earth, a person floating in space would actually not receive enough light to keep from freezing! You yourself would radiate away your heat, and that's why spacesuits have heaters in them.

-Phil Plait

--MacAddct1984 (talk &#149; contribs) 15:53, 19 May 2009 (UTC)[reply]
That doesn't sound right. Of course you radiate heat, but you also produce heat metabolically. Black body#Radiation emitted by a human body says you radiate about 100 W (oops, see below), which is roughly what you produce (100 W × 1 day ≈ 2000 kcal). So I'd think you'd be reasonably comfortable temperaturewise. That's ignoring heat from the Sun. Insolation at 1 AU is around 1400 W/m2. I don't know what the human cross-section is, but I guess you'd be getting about a kilowatt. I also don't know how much of that you'd absorb (it depends on the color of your clothing/skin), but at any rate it looks like overheating is your problem, not freezing. I guess sweating would help—it would evaporate instantly and at least carry away the heat that was in it at the time (like Tango, I'm unclear on whether it would draw any extra heat as it evaporated). I don't know if it would be enough. Note the Apollo suits were cooled, not heated, and they were white, which seems to support this analysis. (They were in fact evaporatively cooled—I'm not sure what that says about the sweating issue.) -- BenRG (talk) 18:54, 19 May 2009 (UTC)[reply]
Never mind, I just realized that the figure in the article is net radiated heat, not σTbody4, for which I get around 1 kW. So Phil Plait is probably right. -- BenRG (talk) 19:21, 19 May 2009 (UTC)[reply]
I thought space suits have refrigeration units in them, not heaters. Insolation from the Sun is, indeed, about 1400 W/m2 and the side of the person facing the Sun is probably about 1 m2, so that is 1400W from the sun. There is also about 100W from metabolism. By my calculations, completely exposed to vacuum one would radiate about 1000W. That's about 500W left over that needs to be dealt with by refrigeration. --Tango (talk) 19:37, 19 May 2009 (UTC)[reply]
The extreme vacuum would be sucking the moisture out of your skin. I'll bet you'd get frostbite as your sweat boiled away. APL (talk) 02:44, 19 May 2009 (UTC)[reply]
Would you get any evaporative cooling in a vacuum? The sweat wouldn't need any heat in order to evaporate since the boiling point in a true vacuum would be absolute zero, wouldn't it? --Tango (talk) 15:46, 19 May 2009 (UTC)[reply]
You have to supply the latent heat of vaporization in any case, so yes, there would be cooling.
Water doesn't really have a "boiling point" in a vacuum. At pressures below the triple point there is no liquid phase — it simply passes directly from solid to vapor (see sublimation (chemistry)). If this picture is accurate, it appears that the solid phase continues up to almost 200 K even at zero pressure. I suppose every now and then you'd lose a molecule to space and it wouldn't come back, so it doesn't seem like a true equilibrium, but I gather that there isn't any tendency for this to happen quickly. --Trovatore (talk) 19:37, 19 May 2009 (UTC)[reply]
Excellent point, I completely forgot about the latent heat! Thank you for that. If the loss of pressure were gradual (a leaking spacecraft/spacesuit say), then the sweat would presumably boil before the pressure reached the triple point (if I'm reading that graph correctly), so the lack of a liquid phase isn't an issue. If the loss of pressure were more sudden, what would happen? What happens if you put a liquid into conditions in which there isn't meant to be any liquid? Presumably it would boil as well. Once the sweat that was produced prior to the loss of pressure is all gone, does anyone know what would happen? Are sweat glands capable of producing sweat in a vacuum? The pressure of the skin would allow liquids to exist inside the body, but how would pores behave? --Tango (talk) 19:51, 19 May 2009 (UTC)[reply]
I think if you put liquid water into a vacuum it would simultaneously boil and freeze. As it boils, giving taking up its latent heat, the temperature drops and the remainder freezes.
As for the human body, I don't really know. At some point I'm going to go challenge the information in the articles about "skin swelling to twice its normal size" or whatever it claims now. There isn't a cite for it (or wasn't last I checked), and I suspect that someone may have extrapolated from Kittenger's experience. I explained above why that's not a valid inference. --Trovatore (talk) 20:07, 19 May 2009 (UTC)[reply]
My understanding is that the elasticity of the skin is sufficient to keep internal pressure normal with only minimal swelling. Death is from suffocation (air gets "sucked" out of the lungs [assuming you don't try and hold your breath and make them explode], oxygen then defuses from blood to empty lungs very quickly and you lose conciousness within seconds). What happens to your corpse, I don't know. I would guess not a great deal at first and it eventually freezes or burns depending on whether or not you are in sunlight. --Tango (talk) 20:22, 19 May 2009 (UTC)[reply]
The pressure axis on that diagram is logarithmic. I suspect that only the gas phase exists in the limit of zero pressure, because of the irreversibility of casting a molecule out into the infinite void. Of course, at absolute zero the solid phase kinda has to prevail. --Tardis (talk) 22:11, 19 May 2009 (UTC)[reply]
Those molecules would only be cast out very slowly, though, there would be a solid for a long time. --Tango (talk) 22:30, 19 May 2009 (UTC)[reply]
I'm a latecomer to the party, I know, but remember that the temperature of human skin is much greater than 200 K, so a great deal of evaporation/sublimation would occur initially, until the skin cooled to well below what we'd consider "frozen solid". -RunningOnBrains(talk page) 17:19, 20 May 2009 (UTC)[reply]

Plastics, Playdough and thermoelastisity

Several questions here.

What is the melting point of #2 plastic (used in milk jugs in usa)

What is the melting point of playdough

If I make an object with #2 plastic using a playdough mold, how much smaller would the final object be from the original thing pressed into the playdough after the plastic cools?

thankyou and have a nice day. —Preceding unsigned comment added by 173.25.242.33 (talk) 21:52, 18 May 2009 (UTC)[reply]

According to Plastic recycling, a #2 cycle code is for High-density polyethylene - and according to that article: "HDPE has little branching, giving it stronger intermolecular forces and tensile strength than lower-density polyethylene. It is also harder and more opaque and can withstand somewhat higher temperatures (120 / 248 °F for short periods, 110 °C /230 °F continuously)". According to playdoh, "Play-Doh's current manufacturer, Hasbro, reveals the compound contains water, salt, and wheat flour. While its 2004 US patent indicates its composed of water, a starch-based binder, a retrogradation inhibitor, salt, lubricant, surfactant, preservative, hardener, humectant, fragrance, and color.[7] A petroleum additive gives the compound a smooth feel, and borax prevents mold from developing.". This seems like a bad thing for your idea - the plastic won't melt until 110°C to 120°C - but the water in the playdoh may well be boiling by that point...that might not be a good thing. The RepRap Wiki says that: "HDPE Is one of the more finicky plastics...It does not stick together well, and...it shrinks a lot when it cools tends to distort readily."...unless you REALLY specifically want to do recycling, you'd be better off heading to an art supply store and getting some Polycaprolactone - which is sold under various trade names as stuff to make jewellery out of. It softens in warm water and melts at about 60°C - so you can even mold it with your fingers. SteveBaker (talk) 01:16, 19 May 2009 (UTC)[reply]
Maybe another modeling clay would be better, like the polymer based Fimo (can be heated to higher temps, and is harder).YobMod 07:16, 19 May 2009 (UTC)[reply]
One warning from the palycaprolactone article... "PCL has been known to become brittle, lose its tensile strength and fall apart after several months so is not suitable for permanent or critical applications." So I wouldn't make anything too important out of it. Low density polyethylene might be a better answer as it has a melting point of 95 C. 65.121.141.34 (talk) 13:06, 19 May 2009 (UTC)[reply]
Yeah - I noticed that. But I have stuff that I made of polycaprolactone (specifically the "Polymorph" brand) that's at least 16 years old that hasn't obviously degraded (I know it's that old because Polymorph is the British name for the stuff - so I must have made it while I still lived in the UK - which must be more than 16 years ago). I suspect that our article is referring to the plastics based on polycaprolactone that have starch mixed in with them specifically to make them biodegradable. The article mentions this - but I wonder whether the authors have correctly put two and two together. SteveBaker (talk) 19:58, 19 May 2009 (UTC)[reply]

Identifying some Australian animals from Sydney Wildlife World

Hi all. I'm looking for help identifying the following Australian animals that I photographed at Sydney Wildlife World in April 2007 so that I can upload them to Commons:

Thanks for any help. :-) Dcoetzee 22:11, 18 May 2009 (UTC)[reply]

We don't have a page, but the Psudocheirus cinereus - Daintree Ringtail Possum [32] looks like the "nocturnal creature in its nest" I'm not into creepy crawlies so I'll leave those for others. 71.236.24.129 (talk) 23:58, 18 May 2009 (UTC)[reply]
o.k. one more: Your "Grashopper" may be a migratory locust [33] the wings are a bit short which would indicate it's not an adult or another species.

May 19

Tree questions

Is tree really absorbed CO2 and release O2 in day time, and the contrary at night? Are all kind of trees have the same characteristic? And what species of trees produce the largest amount of O2? When we sleep in the forest at night, can we suffered from lack of oxygen? Thanks for the responds. roscoe_x (talk) 01:46, 19 May 2009 (UTC)[reply]

All plants perform cellular respiration, which consumes oxygen and releases CO2. However, this effect is completely overwhelmed by the action of photosynthesis, of which oxygen is something of a side-effect. Because plants manufacture their own oxygen, the "drain" on atmospheric oxygen is minimal. Go ahead and sleep outdoors; it'll do you good! Incidentally, there is an urban legend that you shouldn't bring flowers to people in hospitals because the plants supposedly suck the oxygen out of the room. Complete nonsense. Matt Deres (talk) 03:10, 19 May 2009 (UTC)[reply]
(EC) Matt, I'm no scientist, but I think there is something wrong with that last post. It implies that plants take in oxygen, then release CO2 (contrary to what I and the OP have been told in primary school) and yet still have enough oxygen to release that too, on account of photosynthesis. So, the idea of the Amazon rainforest getting depleted adding to the CO2 level(or at least not reducing it) and thus causing global warming is nonsense? Planting more trees would not help, because they suck oxygen from the atmosphere and not CO2? Are you sure about this? Question asked in all good faith - not sarcastically --KageTora - (영호 (影虎)) (talk) 03:22, 19 May 2009 (UTC)[reply]
Ha! Either I misread the answer or someone changed it while I was writing my query. Forget it. Been a long night.--KageTora - (영호 (影虎)) (talk) 03:25, 19 May 2009 (UTC)[reply]
Yeah - exactly - the amount of O2 they produce (and CO2 they absorb) during the day more than makes up for the small amount of O2 they consume and CO2 they produce at night. Also, it's not just trees - all green plants from the teeny-tiniest green algea to the giant redwoods do that. SteveBaker (talk) 03:17, 19 May 2009 (UTC)[reply]
This kind of question keeps coming up. The Plant article seems to be hopelessly lost in taxonomy and basic plant concepts like what parts do what and the question above get one on a wild goose chase through highly specialized articles. If one can find anything to begin with, that is. Thinking of the post about organizing information in a tree structure earlier this month maybe someone with enough knowledge could sort out an article for the non-specialists. I don't feel comfortable starting something, because my last biology lesson was a couple of decades ago and in my experience any article started without enough ooomph just gets deleted. 71.236.24.129 (talk) 06:53, 19 May 2009 (UTC)[reply]
I think your idea is good. I think Wikipedia should have a FAQ page on each of its article. roscoe_x (talk) 14:25, 19 May 2009 (UTC)[reply]
There is always the Simple English Wiki - although in this case, their plant article is probably waaay TOO simple. SteveBaker (talk) 02:53, 20 May 2009 (UTC)[reply]
Not all plants absorb CO2 and release O2 in day time and do the opposite at night. Keeping it simple, CAM Plants take in CO2 during the night and convert it to oxygen during the day as a way to keep the plant from losing too much water. I'm not sure when most of the oxygen is released, but my hunch it happens mostly during daylight hours. As to what plant produces the greatest net O2, I'm not sure, but it is likely not a species of tree. Someone answering a similar question stated that "the largest fraction of the planet's molecular oxygen is not produced by tree and other landbased plants but photosynthetic organisms in the world's oceans." I can't validate their answer, but I wouldn't be surprised if it was true. Sifaka talk 03:31, 20 May 2009 (UTC)[reply]

dimensional question

when a wave traverses a medium, the displacement (Y) of a particle located at x at time t is a*sin(bt- cx), where a, b and c are constants. what are the dimensions of b??

the dimension of Y is [L], and that of t is [T]. but should i write the dimensions of x as [L} or ... something else? 122.50.135.154 (talk) 05:44, 19 May 2009 (UTC)[reply]


Ya you're right. x is [L]. Which means c must be [L-1], as you can't take the sine of anything except a number. Similarly, b is [T-1], and the sine of anything is just a number, dimensionless. So a is [L].... There seem to be a lot of questions on dimensional analysis :-)..Rkr1991 (talk) 07:28, 19 May 2009 (UTC)[reply]

Almost right. "c" is actually radians/L, and b is radians/T, assuming your sine function is in radians, as it usual in mathematics. If your sine function is in degrees, replace "radians" with "degrees". The "a" is an amplitude, "b" is frequency, and "c" is phase shift.-Arch dude (talk) 20:31, 19 May 2009 (UTC)[reply]
Radians are generally considered to be dimensionless. They are the ratio of two lengths (a radius and an arc length). --Tango (talk) 20:38, 19 May 2009 (UTC)[reply]

explosive force calculation based on weight.mythbusters??

Hi

I am confused about something I saw on mythbusters the other day, they make a cannon that launches a 6 pound bowling ball 1500 feet. Yet the cannon itself barely moves backwards. shouldnt a force that can move 6 pounds 1500 feet move a cannon that weighs around 150 pounds about 50 feet? or does something else effect this ? I mean i would think that a force powerfull enough to push a 6 pound cannonball 1500 feet would have at least enough force in the other direction to move a 150 pound cannon about 20-30 feet, but the cannon just rolls back like a foot and falls over.

how does the calculation for the amount of pressure needed to move heavier and heavier objects work?

Thank you. —Preceding unsigned comment added by 79.68.210.245 (talk) 08:23, 19 May 2009 (UTC)[reply]

The canon would move a distance at a ratio inversely proportional to the ratio of masses, if there were no other forces acting (eg in space). But in this case, the frictional force between the canon and ground would presumably be much higher than that between the canon and ball, by design. So the reaction force is mostly balanced out by this frictional force, leading to little acceleration. Friction is dependant upon both mass and the friction coefficients of the materials used, hence heavier objects need more force to move on Earth than a simple momentum calculation would suggest. Recoil might have more info (it might have had a recoiling barrel on springs for example, so the momentum is aborbed by that and prevents large backwards movement).YobMod 09:22, 19 May 2009 (UTC)[reply]

AAAh thank you, your suggestion of friction, plus the recoil article helped. apparently some cannons and guns can be designed to have more chemical energy tranferred to the projectile than the gun. That would explain it. this is the quote "bullet fired from an M16 rifle has approximately 1300 foot-pounds of kinetic energy as it leaves the muzzle, but the recoil energy of the gun is less than 5 foot-pounds. The reason mechanical energy is not conserved is because much more of the chemical energy released during powder combustion is transferred to the bullet than is transferred to the gun."

How would they do that? —Preceding unsigned comment added by 79.68.210.245 (talk) 10:08, 19 May 2009 (UTC)[reply]

Actually, nearly any gun will do that by the simple laws of physics. Action=reaction applies to impulse - the gun and the projectile (plus the escaping gas, if we want to nitpick) have to have a total impulse that is zero - in other words, the impulse of projectile and the gun will have the same magnitude but opposite directions. Since impulse is mass times velocity, but the gun is a lot more massive, the projectile is much faster than the gun. Kinetic energy is mass times velocity squared, so that higher speed translates to a much higher kinetic energy. --Stephan Schulz (talk) 11:22, 19 May 2009 (UTC)[reply]
Working through the equations for parabolic motion and conservation of momentum, if the cannon were on frictionless wheels, it would move backward at somewhat less than 5 mi/hr after the shot. If it isn't on wheels, and considering that the shot is probably angled upward at about 45 degrees resulting in a strong downward push, friction would probably keep it from moving.

I am surprised that no one on refdesk spotted that, "The reason mechanical energy is not conserved is because much more of the chemical energy released during powder combustion is transferred to the bullet than is transferred to the gun." is a non sequitor that makes no sense.

  • "The reason mechanical energy is not conserved is because" there is no law of conservation of "mechanical energy". In any case, energy is not a vector quantity and the energy of the gun moving backward will not somehow cancel the kinetic energy of the bullet.
  • In a gun/cannon, chemical energy (from the explosive) is converted into mechanical energy (+ heat, sound etc), and total energy is of course conserved.
  • More importantly, for understanding recoil: Momentum is also conserved. The "positive" momentum (mass*velocity) of the bullet is matched by the "negative" momentum of the gun + shooter + earth + exhaust gases.

Can someone take a stab at rewriting the Recoil section ? Abecedare (talk) 16:27, 19 May 2009 (UTC)[reply]

What we're missing here is that the total momentum of the entire system is constant - that includes bullet, gun AND the gasses produced by the propellant. So in a "recoilless" gun (well, they have some recoil - but less), some of the propellant is sent sideways and backwards at very high speeds - and this removes some of the momentum from the recoil. But certainly the bullet's mass is much less than the gun - so the force acting on the gun is the same as the bullet - but the resulting acceleration is much lower. In the case of the Mythbusters' cannon - the friction with the ground is transferring some of that momentum to this big planet that has a REALLY big mass and consequently hardly accelerates at all. It's also common to have some kind of elasticity in the system (eg a padded shoulder stock) that spreads the recoil's energy out over a longer period of time - thereby reducing the instantaneous force to something much easier to handle. SteveBaker (talk) 19:52, 19 May 2009 (UTC)[reply]
Also, what everyone is forgetting is that the cannon is not fired straight parallel to the ground. If it were, it would likely roll backwards some distance. The cannon is usually fired at an upwards angle, which means much of the recoil force is directed into the ground. Much of the energy therefore goes into deforming the ground under the cannon rather than pushing the cannon backwards. --Jayron32.talk.contribs 00:56, 20 May 2009 (UTC)[reply]

cats landing on their feet

House cats always seem to land on their feet. Does this self righting reflex extend to other members of the cat family, such as jaguars, cougars, or tigers? 65.121.141.34 (talk) 14:09, 19 May 2009 (UTC)[reply]

Probably only if they are habitual tree climbers, like leopards and unlike lions. - GlowWorm. —Preceding unsigned comment added by 174.130.252.35 (talk) 14:15, 19 May 2009 (UTC)[reply]
As an interesting sidenote: Cats can only self right if the height from which they fall gives suffivient time to execute the reflex. Some studies (and i wish i still had that link!) showed that cats falling from either very low or high altitudes survived the fall more often then cats falling from heights in which they cannot execute the full reflex - The writers of course noted that there was a maximum height in which cats tended to survive more then cats falling from lower heights. Excirial (Contact me,Contribs) 14:35, 19 May 2009 (UTC)[reply]
No, Wikipedia does not have one page for this...it has two!
The High-rise syndrome article has some good cites supporting the number-of-floors effect. Might want to copy them to the Cat righting reflex article, which presently only cites Straight Dope. DMacks (talk) 15:37, 19 May 2009 (UTC)[reply]
The question could be rephrased "Can big cats right reflexively, or is this just a 'little cat feat?'" See also Buttered cat paradox. Edison (talk) 16:44, 19 May 2009 (UTC)[reply]
Cats I have known could all right themselves when dropped from amazingly low heights, like 1 foot. Edison (talk) 18:36, 19 May 2009 (UTC)[reply]
The height from which cats suffer the highest fatality rate, ie. heights higher or lower than this are more likely to allow the cat to right itself properly and survive, is approximately seven storeys. This is because acceleration due to gravity causes the cat to become rigid, but once the terminal velocity is reached, the cat has an easier time trying to right itself. ~AH1(TCU) 01:59, 20 May 2009 (UTC)[reply]
So does this work with the big cats too? 65.121.141.34 (talk) 15:09, 20 May 2009 (UTC)[reply]
I suspect that some big cats have a righting reflex, but that it is less effective because big cats are more massive and so must absorb considerably greater force of impact for the same velocity. Small wildcats probably pull this off better. Dcoetzee 05:55, 21 May 2009 (UTC)[reply]
Which is worse, a drop-bear or a drop-tiger? DMacks (talk) 06:27, 21 May 2009 (UTC)[reply]


In reading through this, I checked out the Buttered Cat Paradox which led to the Irresistable Force Paradox. My question is about this latter. IF an irresistable force and an unmovable object did exixt and were to meet, wouldn't they simply cancel each other out (i.e. annihilate each other)? Why or why not (other than these forces/objects can't exist)? —Preceding unsigned comment added by 209.161.223.123 (talk) 03:42, 22 May 2009 (UTC)[reply]

mass increase or mass energy equivalence

which one did einstein formulate first? mass increase in relativity or mass energy equivalence? if he formulated mass energy equivalence first, can some one give the derivation first?--harish (talk) 14:59, 19 May 2009 (UTC)[reply]

Both came in 1905 papers, but the mass increase came first, in "On the Electrodynamics of Moving Bodies". Mass-energy equivalence was first directly presented, I believe, in the later 1905 paper "Does the Inertia of a Body Depend upon its Energy Content?". See special relativity and mass-energy equivalence for more information. Looie496 (talk) 15:31, 19 May 2009 (UTC)[reply]
Weren't the units of work or energy (such as the joule in the MKS system) given in kilogram meters2 per second2 before 1905? This meant that energy was already in units of mass times velocity squared. Was Einstein the first to notice this? Edison (talk) 17:06, 19 May 2009 (UTC)[reply]
was known long before Einstein, that gives you the units. --Tango (talk) 17:15, 19 May 2009 (UTC)[reply]
Right, but having the same units is not the same as being interchangeable. (Think about how much pathological physics would result...) Einstein not only noticed the units, but explained a correct equivalence relationship. Nimur (talk) 05:44, 20 May 2009 (UTC)[reply]
Of course, I was just pointing out that Einstein wasn't the first the notice the units, they were common knowledge long before he came along. --Tango (talk) 12:06, 20 May 2009 (UTC)[reply]

artifact hoaxes

Which scientific or historic artifact was accepted as genuine for the longest period of time before it was later discovered to be a hoax or forgery? 65.121.141.34 (talk) 15:08, 19 May 2009 (UTC)[reply]

This is totally unauthoritative, but:
  • I think it must be a religious artifact of some sort, like a church claiming it has part of the True Cross or some other relic — at this point these claims are probably being made in good faith but human nature suggests that most of them involve somebody lying at some point. Does this still comprise a hoax?
  • The Talpiot Tomb is around 2000 years old and was discovered in 1980; some claim that one inscription reads "Yeshua bar Yehosef", or "Jesus son of Joseph". Hoax? Concidence? Who knows.
  • More contemporaneously, the 1934 "Surgeon's Photo" of the Loch Ness Monster was revealed as a hoax in 1994. That's sort of an artifact.
The right answer may be some hoax that is as of yet undetected. Tempshill (talk) 15:39, 19 May 2009 (UTC)[reply]
Piltdown Man is one that creationists often point to that "discredits evolution". It was exposed as a hoax in 1953, but some scientists were skeptical from the start. Also, what about those inscribed balls found in South America? The name is eluding me... -- MacAddct1984 (talk &#149; contribs) 16:02, 19 May 2009 (UTC)[reply]
The consequences of Piltdown Man went on long after the hoax was exposed. I wasn't born until 1955 - and I clearly recall being taught that Piltdown Man was 'the missing link' - and that must have been around 10 years later! It takes a long time for syllabuses and school books to get rewritten! If you ever get the chance to visit the little village of Piltdown - be sure to drop into their pub: "The Missing Link" - it serves some of the finest Real Ale I've had the pleasure of sampling. SteveBaker (talk) 04:07, 20 May 2009 (UTC)[reply]
I'm betting it's a book of some kind. As I recall, there are works that were attributed to writers like Aristotle for over two thousand years but eventually demonstrated by textual analysis to be forgeries. Unfortunately I have only the vaguest recollection of any of this, not enough to dig up sources. Looie496 (talk) 16:55, 19 May 2009 (UTC)[reply]
Klerksdorp Spheres? --Sean 17:39, 19 May 2009 (UTC)[reply]
Our literary forgeries article says One of the longest lasting literary forgeries is by Pseudo-Dionysius the Areopagite a 5-6th century Syrian mystical writer who claimed to be a disciple of Paul the Apostle. Five hundred years later Abelard expressed doubts about the authorship, but it was not until after the Renaissance that there was general agreement that the attribution of the work was false. In the intervening thousand years the writings had much theological influence. Looie496 (talk) 17:47, 19 May 2009 (UTC)[reply]
Turin shroud? The Bible? 89.242.109.25 (talk) 09:16, 20 May 2009 (UTC)[reply]
I have not seen anything proving that either of those are hoaxes or forgeries, though the church does not seem to allow much in the way of testing such relics as the shroud. 65.121.141.34 (talk) 15:07, 20 May 2009 (UTC)[reply]
See Radiocarbon 14 dating of the Shroud of Turin. 78.146.198.122 (talk) 20:50, 20 May 2009 (UTC)[reply]
Why would the Bible be a hoax? It's pretty well researched when it's specific books were written, and I've never heard of anyone claiming that it was written a lot later. --131.188.3.21 (talk) 20:51, 20 May 2009 (UTC)[reply]

momentum and relativity

take two coordinate systems K and K' . K' is moving with a velocity of v relative to K. An object is moving with a velocity of u relative to K'. will the observers in K and K' agree with the momentum and velocity of the object?--harish (talk) 16:11, 19 May 2009 (UTC)[reply]

Looks like a homework problem (which we don't do). Looie496 (talk) 16:57, 19 May 2009 (UTC)[reply]
It looks too general for a homework problem to me. --Tango (talk) 16:58, 19 May 2009 (UTC)[reply]
No. Velocity and momentum are always relative to a coordinate system, different systems will get different values for them. What velocity observers in K will measure depends on whether you are talking about simple Galilean relativity, in which case you can just add the velocities, or Special Relativity, in which you need the appropriate velocity addition formula. --Tango (talk) 16:58, 19 May 2009 (UTC)[reply]

biomedical projects?

i am applying for a scholarship for which i am supposed to do a BIOMEDICAL PROJECT.i need some information regarding this.what is a biomedical project?what am i supposed to do?is any paper work included?is it all about collection of some data or woking out something new? —Preceding unsigned comment added by 115.241.146.220 (talk) 17:25, 19 May 2009 (UTC)[reply]

:Pardon me for this observation, but if you don't know what a biomedical project is why are you wasting your time applying for the scholarship? --TammyMoet (talk) 19:01, 19 May 2009 (UTC)[reply]

BIO stands for Biological, while Medical stands for MEDICINE. In short, i assume that you will have to do some medicine related project. As for paperwork and working data out: How could we know? A project can be anything; it all depends on how you define the goals of the project. However i tend to agree with the user above me - If you don't even have a basic clue about the mere meaning of the word is it wise to apply for such a project? Most times at least some background knowledge is required to successfully complete a project Excirial (Contact me,Contribs) 19:08, 19 May 2009 (UTC)[reply]
Most likely "doing a biomedical project" means finding a mentor who has a lab that does biomedical research, working out with the mentor a project that involves lab work, and then writing up the results. Beyond that, the details may vary. Looie496 (talk) 19:41, 19 May 2009 (UTC)[reply]
The best advice is to contact the people offering the scholarship directly. They aren't trying to mess with you; they will answer your questions directly. It's always best to contact the people directly involved rather than random strangerz on deh intrawebz. Those people will be able to answer your question directly. --Jayron32.talk.contribs 00:51, 20 May 2009 (UTC)[reply]
I've gotten a few scholarship before, which required having a project planned out. I basically just wrote to a (large) number of professors whose web-pages looked interesting, explaining the requirements for the scholarship. Most profs are very happy to help if you will be be bringing your own funding. However, i also had all the qualifications needed - for a biomed project scholarship, i would think the applicant should have experience in biology or medical research, at least at the undergraduate level.YobMod 09:49, 20 May 2009 (UTC)[reply]

Number of individual lifeforms on this planet?

Is there an estimate of the number of individual lifeforms on our planet? Meaning, all 6 billion humans, every individual insect, arachnid, invertebrate, blade of grass, etc. My second question is, are there more galaxies in the universe, than there are individual lifeforms on our planet? ScienceApe (talk) 17:58, 19 May 2009 (UTC)[reply]

If you're just counting lifeforms, virtually all of them are bacteria or archaea. They're hard to count, but one estimate is on the order of 5 x 1030. Looie496 (talk) 18:19, 19 May 2009 (UTC)[reply]
As for the amount of galaxies: As estimate is that there are between 10 billion and 1 trillion stars, all which may or may not have planets orbiting them. If we were to count every metheorite, planet and star then the numbers would be drastically higher. Higher then the amount of lifeforms we have? Possibly - the galaxy is a massive place and our knowledge about it is still very localized. Excirial (Contact me,Contribs) 19:01, 19 May 2009 (UTC)[reply]
Sorry, you didn't ask for the amount of stars, but for the amount of galaxies. If i remember well the last estimate i saw was between 60 and 110 billion galaxies. Far less then the amount of estimated lifeforms. Excirial (Contact me,Contribs) 19:03, 19 May 2009 (UTC)[reply]
It is necessary to clarify the word "universe". The entire universe may well be infinite and, thus, contain infinitely many galaxies. The observable universe is finite and our article says it contains "more than 80 billion galaxies". --Tango (talk) 19:11, 19 May 2009 (UTC)[reply]
Excirial, I think your stars estimate is way off. Our galaxy itself contains 100 billion stars. It's a hundred thousand light years side to side. It bulges in the middle, 16 thousand light years thick; but out by us it's just 3 thousand light years wide. We're 30 thousand light years from Galactic Central Point; we go 'round every 200 million years; and our galaxy is only one of millions of billions in this amazing and expanding universe. Tempshill (talk) 00:02, 20 May 2009 (UTC)[reply]
Has life got you down, Mrs. Brown? --Jayron32.talk.contribs 00:49, 20 May 2009 (UTC)[reply]
100 billion is between 10 billion and 1 trillion. Excirial initially misunderstood the question and thought it was about the number of stars in our galaxy, rather than the number of galaxies. --Tango (talk) 00:31, 20 May 2009 (UTC)[reply]
Since regardless of how you do it there are millions of times more bacteria on Earth than stars in the entire universe, it's all sort of irrelevant. Looie496 (talk) 02:44, 20 May 2009 (UTC)[reply]
Observable universe. --Tango (talk) 12:07, 20 May 2009 (UTC)[reply]

Film speed vs. digital gain?

I understand that film speed and signal gain in photography are about the same thing: sensitivity to light, and work pretty much the same way. Greater sensitivity allows for shorter shutter speeds while still keeping the image lightness identical, but produces noisier and grainier images. My question is, because once the light gets through the shutter (how much and for how long the shutter is open is identical), what happens then is fundamentally different between film and digital photography. The sensitivity is labelled with the logarithmic ISO scale in both anyway, using the same numbers. How well do these correspond with each other? How is it determined which film speed corresponds to which signal gain? Are even the bases of the logarithms the same? JIP | Talk 19:34, 19 May 2009 (UTC)[reply]

Film speed answers your question. In short, yes they should correspond with each other. --antilivedT | C | G 05:06, 20 May 2009 (UTC)[reply]

Amount of vegetable fat in ml

I've been rather surprised to learn that recent medical advice is that consuming reasonable amounts of vegetable oil (but not hydrogenated or trans fats) is better for you than trying to avoid all fats. http://heartdisease.about.com/cs/cholesterol/a/raiseHDL_2.htm "The best advice regarding fat in the diet appears to be this: 1) reduce the fat intake to 30 - 35% of the total calories in the diet - but probably no lower than 25% of total calories". How much in ml of vegetable oil is 1% of total calories in one's diet going to be? 78.149.232.7 (talk) 23:44, 19 May 2009 (UTC)[reply]

According to this page, vegetable oils contain about 120 calories per tablespoon (15 mL). Caloric intake will vary from person to person, but USDA/FDA standards are based on a 2000-calorie daily diet. 1% of 2000 calories would be 20 calories, so 2.5 mL of vegetable oil would supply about 1% of a typical diet's caloric intake, assuming vegetable oil is mainly fat, which it is. 4.242.147.206 (talk) 00:36, 20 May 2009 (UTC)[reply]
As a side note, why did you think a completely fat free diet would be healthy? This reminds me of the overly-simplified definition of healthy food 'low as possible in fat, high as possible in fibre' which would make paper the healthiest food. 80.41.33.31 (talk) 07:37, 20 May 2009 (UTC)[reply]

Involuntary digestive organs?

Is there a common English term for the part of the body where food goes after being swallowing and before being felt as a need to poo? If we had to use anatomical terminology, I guess that would be from the pharynx to the... wherever the threshold of feeling a need to poo is. I was thinking of calling it "involuntary digestive organs" or similar... basically including oesophagus, stomach and the guts. Or is there a better term or even a non-medical plain English term?--Sonjaaa (talk) 23:45, 19 May 2009 (UTC)[reply]

Gastrointestinal tract. Dauto (talk) 23:51, 19 May 2009 (UTC)[reply]
Teenagers in school would call it the digestive system in Biology lessons - the link shows it's the same thing, so take your pick. 90.193.232.41 (talk) 18:05, 20 May 2009 (UTC)[reply]

Climate Variation

Is there any kind of technology on the internet that lets you take a climate chart of a particular location, provided by the site, and do a search to come up with all the climate charts around the world that correspond most closely with the initial chart? This climate chart would include average monthly highs and lows and average monthly precipitation. —Preceding unsigned comment added by 189.4.53.90 (talk) 23:52, 19 May 2009 (UTC)[reply]

You could start at the Weather Underground, http://www.wunderground.com/, which has all sorts of great climate and historical weather information from many many cities around the world. Oh, and dispite the name, they have nothing to do with these people. --Jayron32.talk.contribs 00:46, 20 May 2009 (UTC)[reply]

May 20

What labs, books, and conferences are there about research on the neuroscience of empathy?

I want to make this list more complete.

Specifically, what are the: a) labs or primary researchers b) books c) conferences d) one-off papers

... which have to do with the neuroscience of empathy?

Please include only primary academic sources - i.e. not popular press, unscientific publications, or the like. (Exception: popular-format books by people who are in fact real researchers are OK, e.g. most of Paul Ekman's recent books.)

Most important to me is the list of labs / researchers, preferably with links to pages where I can directly download PDFs of their papers, as the primary use for this list is as a list of places I should apply to for my PhD.

Thanks!

Sai Emrys ¿? 00:26, 20 May 2009 (UTC)[reply]

You could start here. --Tango (talk) 00:29, 20 May 2009 (UTC)[reply]
The Society for Neuroscience annual conference is in Chicago, October 17-21. You may consider attending and finding out for yourself what you like and where you should apply. I don't know if you have to be a member to attend (I'm a member anyway), but it's easy to find out. The SFN homepage is at http://www.sfn.org/ . Also, Tango's advice is really good. Just google or google-scholar for "empathy", "neuroscience", "mirror neurons", "insula", "anterior cingulate", or any combinations thereof, and you will find all you are looking for and more. And, good luck! Empathy is really a fascinating research topic. --Dr Dima (talk) 01:35, 20 May 2009 (UTC)[reply]
Hmm, I wouldn't really recommend going to SFN in order to find a school, it's kind of an overwhelming experience. Anyway, if I had to recommend a place to somebody who could get in anywhere, I would recommend University College London, with the Friths, Ray Dolan, etc. It's a hot topic, though, and there are people all over the place working on it. Looie496 (talk) 02:38, 20 May 2009 (UTC)[reply]
I'm afraid the $550 (+ hotel & travel) non-member price is a bit too steep for me; otherwise, it looks interesting... but very general. I'm only interested in (dedicating my life to) limited parts of the field of neuroscience; I feel a general conference would be a bit too shotgun. (FWIW, I'm very disappointed I missed the Berkeley conference on mirror neurons recently - I heard of it too late. I live in Berkeley, it was exactly on MNs, and it was free to boot! Sigh.)
Yes, I know how to use Google Scholar; my hope was to find somewhat more personal pointers, as GS can be a bit like finding a specific kind of needle in a stack of popularity-ranked hay. I'd love to, but practically speaking there's no way I can actually read through all the stuff there to find out the handful of people whose research is really what I want. This is something that domain knowledge is very helpful for making a lot easier. Thanks! Sai Emrys ¿? 02:48, 20 May 2009 (UTC)[reply]
Many conferences use student volunteers to man the doors, check badges and such like. When you do that, you can generally hang out at the back of the conference hall while sessions are in progress and catch most of the event for free. Obviously travel and accommodation is still an issue. But it's worth calling the conference organizers - expressing your enthusiasm and interest for this narrow field - they may be able to cut you a deal - or even offer a small grant/scholarship to help to get you there. It's definitely worth a try - especially with such a tightly focussed conference. SteveBaker (talk) 03:59, 20 May 2009 (UTC)[reply]
Ah, you're in Berkeley -- and I see from your resume that you got an undergrad degree there. The best advice I can give is to try to get a "technician" job in a lab that does something similar to what you're interested in -- there are several people there interested in social neuroscience, and all of them are top notch. There's nothing better than hands-on experience for getting a good fix on who is doing the stuff you care about most, and it's also a big plus for getting into a top-notch program. (To Steve: SFN is one of the largest academic conferences in the country, with typically about 50,000 attendees -- tightly focused is not what I would call it. It's a circus. They do provide a bit of support, but they wouldn't support somebody who isn't currently in an academic program.) Looie496 (talk) 04:26, 20 May 2009 (UTC)[reply]

Throwing a ball of wool (or twine) to a lion/tiger...

What happens? Do big cats also find it entertaining to play with suchlike? --Kurt Shaped Box (talk) 02:22, 20 May 2009 (UTC)[reply]

The Desert Museum puts ice in the mountain lion cages for their enjoyment if I remember correctly. A google of zoo lion enrichment found this website which says: "To stimulate predatory behavior, keepers toss hay-filled burlap sacks or cardboard boxes into the lion and tiger enclosures, which the cats pounce on and shred as if they were prey. Keepers may add olfactory interest to these items by scenting them with zebra or camel urine, perfumes, hunting lures, and herbs. In the tiger yard, keepers can hook up to a tree a giant spring covered with PVC pipe to which they attach a hard plastic Boomer Ball® or burlap sack. The tigers attack and tug at the unyielding item as if it were struggling prey. For the small cats, keepers hide meat and prey items throughout the enclosures. In the summertime, the fishing cats hunt for live goldfish released into their pools." A search for zoo enrichment might yield some more results. My guess is that a suitably sized ball of twine would generate interest from a large felid, but a zoo might not put an object which might present a tangling or choking hazard in their enclosures. If they could keep them from unravelling it enough to choke or tangle it might work. A zoo keeper would be a good person to ask. Sifaka talk 03:10, 20 May 2009 (UTC)[reply]
Certainly I've seen balls in big cat zoo exhibits - so I presume they play with them. I doubt it matters much whether it happens to be a ball of wool or a plastic soccer ball or whatever. Of course this is all about zoo animals - who need all of this 'enrichment' to keep them interested in life in their tiny enclosures with all of their natural instincts being effectively turned off. Whether a wild Lion or Tiger out there in it's native habitat would give a ball of wool a second glance is an entirely different matter...I suspect the very young ones would - but not the adults. SteveBaker (talk) 03:52, 20 May 2009 (UTC)[reply]

Genus

Where can I find information describing when a genus split from others in its family? Specifically I am writing about genus Abramites, though specific information isn't neccesarily needed, just how to find it.Drew Smith What I've done 07:13, 20 May 2009 (UTC) [reply]

First a remark: I'm no expert in fishs or phylogenetics, just a geneticist. I think, there is no clear answer to your question. Even for families that are "hot research topics" like hominidae the exact time points of speciation are unknown. All you can do is apply a model of the time course of genetic changes, and then come up with some rough estimate how much time it took to accumulate the differences you see. A quick googling didn't bring up any research paper that did this for Abramites, though, so I think the answer is: No one really looked into this. TheMaster17 (talk) 09:14, 20 May 2009 (UTC)[reply]
Meh. I came to the same conclusion, just wondered if anyone knew of a site or book that specialised in this kind of thing.Drew Smith What I've done 11:17, 20 May 2009 (UTC)[reply]
So, for the taxobox, is it ok to use the time period that the order first appeared?Drew Smith What I've done 11:19, 20 May 2009 (UTC)[reply]

Butterflies/moths identification

I took some pics of butterflies a couple of days ago and I wasn't able to fully identify them. I have some hints about the genus/species for two of them, but I can't be sure. The pics were taken in southern Romania, in a forest along the Dâmboviţa river. Can anyone help? bogdan (talk) 09:45, 20 May 2009 (UTC)[reply]

The first is a Diacrisia sannio, the second probably a Pyrgus armoricanus or a Pyrgus malvae and I can't be sure about the last. BTW I used this amazing website to do the identifications [34]. Mikenorton (talk) 15:45, 20 May 2009 (UTC)[reply]
Thank you, both for the identification and for the link. bogdan (talk) 20:43, 20 May 2009 (UTC)[reply]

Work = Force*Distance or = Force*Time?

I have a textbook saying the work done is the product of the force applied and the distance the object moved. But is this ultimately right? Because if I apply the same force for the same time to a faster-moving object, it would result in more work done. Or the distance moved by initial speed of the object must be subtracted from the distance? Work = Force*Time seems better to me. Like sushi (talk) 11:44, 20 May 2009 (UTC)[reply]

No, it's right. Force*Time is momentum. Kinetic energy ("work" just means "energy added") is proportional to the square of velocity, rather than velocity itself, that is the key point. The energy required to make a 1kg object go 1m/s faster depends on the initial speed - if it starts off stationary you need 1/2mv2=1/2J, if it starts off at 10m/s you need 1/2m(112-102)=10.5J. --Tango (talk) 12:13, 20 May 2009 (UTC)[reply]
Thank you. Then the same force applied for the same time could mean different work done?Like sushi (talk) 12:25, 20 May 2009 (UTC)[reply]
Yes. Consider a constant 10N force applied to a 1kg mass starting from rest. In the first second the mass accelerates from rest to 10 ms-1, so the work done is 50 J. In the next second the mass accelerates from 10 ms-1 to 20 ms-1, so the work done is 150 J. Gandalf61 (talk) 12:36, 20 May 2009 (UTC)[reply]

Does that mean the object moving, no force applied, also, well, having work?
Like sushi (talk) 13:09, 20 May 2009 (UTC)[reply]
An object moving with constant velocity has constant kinetic energy. Work is the increase in energy, which would be zero. --Tango (talk) 13:19, 20 May 2009 (UTC)[reply]

We must note that the formula Work = Force x Distance is not exactly right. There are two..erm.. errors. (i) It should be displacement, not distance. Note the difference. Distance is the total length of the path traversed. Displacement is just the distance between the initial and final points alone. For example, if you move on a semicircle, the distance is the length of the path, namely pi*radius. But the displacement is merely the distance between the initial and final points, which is twice the radius. (ii) This formula is valid only for a constant force. For a time varying force, this must be integrated along the path to get the work done, so keep it in mind to apply this formula only for constant forces. Rkr1991 (talk) 12:46, 20 May 2009 (UTC)[reply]

Even though I have no doubt about the formulas, it has always bugged me that KE is quadratic in velocity, and that (as a result) it's not force times duration. What your intuition is picking up on is momentum, not work; the (change in) momentum is the product of force and time, and is called impulse. --Tardis (talk) 13:09, 20 May 2009 (UTC)[reply]
There's one more problem. It should be the dot product, not the cross product. The cross product will get you torque. — DanielLC 15:15, 20 May 2009 (UTC)[reply]
Did anyone say cross product? I think we were all talking about the 1D case. --Tango (talk) 15:18, 20 May 2009 (UTC)[reply]
Still, it doesn't cost any extra to adapt it for the more general case. I was having the same thought as DanielC. —Tamfang (talk) 02:24, 21 May 2009 (UTC)[reply]
It costs understanding for people that aren't familiar with vectors. You don't know if the OP knows anything about vectors or not. --Tango (talk) 14:54, 21 May 2009 (UTC)[reply]

How to best boil water?

If we try to heat something by burning fuel, common sense tells us that there is a certain most suitable magnitude of fire. Setting aside the effect of incomplete combustion, (if we use a mixed gas of combustable and oxygen,that will be avoided) Is it better for energy efficiency to make the fire as large as possible, or as small as possible, or there is a golden middle? If we heat it too slowly, the heated object seems to cool down while heating, on the other hand, if we are not trying to heat all the surroundings, the heated gas seems to drive away the gas still not having cooled down yet, and thus, a loss of energy. Like sushi (talk) 12:16, 20 May 2009 (UTC)[reply]

It's not really the size of the fire, but the efficiency of getting the heat from the fire to the water. Storm kettles are very good in that regard. The most efficient way to boil water is, I believe, with an electric kettle, though. --Tango (talk) 12:40, 20 May 2009 (UTC)[reply]
My mother's coffee-making device heats the water as it's coming out. This has the distinct advantage of not having to heat more of the water than you're going to use. This is somewhat off-topic, though, since you're asking about fire.... 90.193.232.41 (talk) 18:02, 20 May 2009 (UTC)[reply]
To efficiently use a kettle or pan of water on the gas range (or "hob"), adjust the flame so that the bottom of the vessel is heated without excess heat escaping around it. This will not make it boil faster, but will get the job done with less fuel. A larger vessel may need a larger flame. A covered pan will boil faster than an open one. The use of water from the "hot" tap is to be avoided for cooking, since it may have more dissolved undesirable substances. With an electric Calrad type burner, select a setting which again matches the size of the heated area to the size of the bottom of the pan. The same quantity of water should boil quicker in a broad pan than a narrow one due to the larger heated surface. A pan with copper cladding might heat quicker than an all steel or thick cast iron one. I suspect that a black or dark bottom pan would absorb heat faster than a white or reflective metallic one, unless the dark coating is thick enough to act as insulation. Edison (talk) 23:06, 20 May 2009 (UTC)[reply]
You would be better off just moving to a country with a mains voltage that allows decent electric kettles. In the UK you can easily get 3kW kettles that boil water far more quickly that you would on a hob. --Tango (talk) 16:25, 21 May 2009 (UTC)[reply]
Yes, those can be extremely efficient. I'm not entirely sure how they work, do they need time to charge inbetween uses? --Tango (talk) 16:25, 21 May 2009 (UTC)[reply]

Unability to lose fat?

I have been working out for the past six months and I have lost a lot of body fat and gained muscle which I am very happy. I am female and have naturally very thin arms and legs so, as you can guess, I tend to gain weight in my torso. However, all these months, my stomach and back fat have remained the same. It doesn't matter how many miles I jog, sit ups and push ups I do, kickboxing (one hour session - 5 days a week) AND playing basketball and volleyball per week. FYI - doc said at my checkup that I am healthy and eat right but I need to lose weight in my middle. He said something about women who are apple shaped are likely to have heart attacks when older. I know that I am genetically predispositioned (mom is short and pudgy) to have a pot belly but it possible for some people to never lose that fat? --Reticuli88 (talk) 12:50, 20 May 2009 (UTC)[reply]

Your question appears to be asking if there is a medical justification for your inability to lose fat. That requires a medical diagnosis. It is impossible to give a medical diagnosis over the Internet. If you simply want information on medical studies about fat loss, please ask specifically for information about medical studies. Do not include personal medical information. As it is, your question is too much a request for medical advice to be answered. See the guidelines at the top of the page - we will not provide medical advice. -- kainaw 12:56, 20 May 2009 (UTC)[reply]
(Non medical oppinion). A lot of the health freaks at my gym talk about hitting "plateau", for either weight loss or gain. Seems the human body gets used to certain level of regular exertion and calorie intake. They generally suggest changing your regime to suprise the body again - so either a further reduction in calorie intake (booo!), or mixing up your excercise for a while, such as swimming / lifting weights, while giving the running a rest.
Does anyone know how much the body adapts to a diet (or can point to a journal)? I know i can reduce by 500 Cal/day from the average 2500 and it makes no difference, but reducing by 1000 Cal/day causes weight loss (and 500 Cal/day extra gives weight gain). There must be research on the calorie level when these things happen on average.YobMod 08:18, 21 May 2009 (UTC)[reply]
This article [[35]] may point you towards the research that prompted your doctor's comments. Also see Adipose tissue#Visceral fat. It is reportedly harder to get rid of than the one stored in other tissue. When starting an exercise program or going on a diet a lot of excess water is often lost from tissues. This is frequently mistaken for "loosing fat". Miracle diet ads create unrealistic expectation as to how fast health weight reduction can be achieved. 71.236.24.129 (talk) 10:54, 21 May 2009 (UTC)[reply]

CO2

The en:wiki (:en:Carbon dioxide) has for the meltingpoint of CO2: -78.5 °C, the german de:wiki (de:Kohlenstoffdioxid) tells us it (called Schmelzpunkt) is: -56.6 °C.
The en:wiki states for the boiling point of CO2: -56.6 °C, the german de:wiki gives for the same (called Siedepunkt) -78.5 °C.
Why did it go so wrong, and how to fix it? --VanBurenen (talk) 12:53, 20 May 2009 (UTC)[reply]

FWIW, Wolfram Alpha says the melting point is -56.56C and the boiling point -78.5 C. -- Aeluwas (talk) 13:05, 20 May 2009 (UTC)[reply]
When heating a solid first it will melt, then boil. Thus the boiling temperature has to be the higher of the two. The -56C is higher (hotter) then the -78C. So I think the en:wiki is (now) right and all the other wiki's (and Wolfram Alpha) are wrong... (Or I am nuts?) --VanBurenen (talk) 13:23, 20 May 2009 (UTC)[reply]
Don't ask me, I'm about as reliable as... uh, not sure what, today. I did think that it looked pretty odd when I wrote it, but I'm sure that's what Alpha says. I'm NOT sure that it's correct, on the other hand! -- Aeluwas (talk) —Preceding undated comment added 13:53, 20 May 2009 (UTC).[reply]

Under typical lab conditions (~1 atm pressure), solid CO2 sublimes directly to the gas at -78°C, and has no liquid form. Liquid CO2 only exists under certain pressure conditions, so separate melting/boiling points only make sense at high pressures. See File:Carbon dioxide pressure-temperature phase diagram.svg. I think the problem with the en:wiki article is that is uses "melting point" as a standard field, which is incorrect in this case. So the melting point given is really the sublimation point, and note the boiling point is given for a particular high pressure.YobMod 14:51, 20 May 2009 (UTC)[reply]

Hubble Propulsion

Greetings, giant brains! Please tell me where I begin to veer off course during the following series of postulates: A) The Hubble Space Telescope has a mechanism that allows it to change direction so that it can track pieces of the sky for long periods of time, and turn to other subjects when its ground-based masters desire. B) Since the Hubble cannot store years worth of propulsive material to eject and change direction, it must use forces generated by the movement of internal battery-powered mechanics to accomplish this trick. C) It is therefore possible to create a giant solar-powered, wing-flapping, bird-like spaceship to travel all over the solar system. No ejection of propulsive material required. Sappysap (talk) 13:28, 20 May 2009 (UTC)[reply]

Birds can use flappers because they push against air. In space there is no air. --VanBurenen (talk) 13:31, 20 May 2009 (UTC)[reply]
Changing orientation is easy since it only requires a temporary change in angular momentum (which can be offset by an opposite temporary change in angular momentum in another part of the craft). Moving around (at least, moving further than the size of the craft) requires a change in linear momentum which cannot be offset by part of the craft, it needs to be offset by something leaving the craft (generally exhaust gasses from a rocket engine). --Tango (talk) 13:37, 20 May 2009 (UTC)[reply]
Hubble can't move itself from one place to another without propellant. It can ROTATE by moving its gyroscopes around. Gyroscope goes clockwise, Hubble goes counter-clockwise. And vice versa. You can't move around like that. (Sure you could throw the gyroscope out the back and the Hubble would move forward slightly, but then you'd just be using gyroscopes as propellant.) APL (talk) 13:51, 20 May 2009 (UTC)[reply]
You might be interested in ion propulsion, which can indeed create a great deal of thrust with only a little bit of ejected matter simply by ejecting it at very high speeds. --Sean 16:26, 20 May 2009 (UTC)[reply]
Hubble definitely uses gyroscopes - those can be powered from the solar panels. The basic idea is that if you spin a gyroscope's wheel in one direction - equal and opposite reaction means that the even bigger, heavier satellite spins in the opposite direction. With three sets of wheels at right angles - you can rotate in any direction. Hence, no (C) - sorry. You can use this trick to rotate a spacecraft - but to accelerate it bodily - you need some reaction mass. The faster you can throw the mass away - the less fuel you need - hence Ion drives which send a very small amount of material out at extremely high speeds. You could also use a solar sail - which uses light, bouncing off of a very large mirror as the 'reaction mass'. However, Hubble doesn't need anything like that. The little propellant it has left is being conserved in order that it can be used for its final de-orbit burn at the end of its life. SteveBaker (talk) 17:57, 20 May 2009 (UTC)[reply]
You might be interested, however, in the idea of solar sails. TastyCakes (talk) 18:03, 20 May 2009 (UTC)[reply]
Get a rotating office chair and a bowling ball. Put chair in center of room, self in chair and hold the ball. Keep your feet off the ground. Demonstrate that you can achieve B) rotary motion by swinging the ball. Try to achieve C) linear motion by any amount of flapping etc. If it's not possible in two dimensions it won't be possible in three dimensions. Cuddlyable3 (talk) 20:56, 20 May 2009 (UTC)[reply]
The trouble is that there is friction involved in the office chair scenario - and because you get different amounts of static friction than dynamic friction, you can move your arms slowly forward without the chair moving because of the high static friction - then fling them back violently and move forwards a few inches. This process can be repeated as often as needed to propel oneself around the room. However, in the almost fictionless environment of space - this trick doesn't work. SteveBaker (talk) 01:42, 21 May 2009 (UTC)[reply]

Setup to play music in apartment

I am looking for a way to listen to music / podcasts in multiple locations in my apartment. All my music / podcasts are on my PC - in my old place I could hear the music from the speakers next to the PC anywhere in the apartment, but in my new (bigger :) ) apartment the PC sits at one end of the apartment, so as soon as I enter another room I can no longer hear it.

I am willing to lay down some cables, but wireless solutions would be wonderful (especially as the distance from my PC to the kitchen would be a good 25-30m of cable, to go around doorframes etc). I would love to have sound in two locations (in addition to the speakers next to the PC).

I have thought about just taking the audio signal from my PC and running it through some y splitters and feeding it to multimedia speakers which I would install in the other rooms. Unfortunately, this involves wiring (a lot of it, I'd have to run wires from my PC to two locations), and I'm not sure what splitting the signal like this (not to mention the 25m of cabling!) would do to the sound quality. I have also thought about feeding the signal to an amp, which would then output to two independent sets of stereo speakers - but, again with the cabling, and some cursory shopping around has revealed no amps with several speaker outputs, does this even exist? Lastly, a colleague has mentioned using some kind of internet radio, which would listen wirelessly (over my WLAN) to a stream from my PC, but I don't know where to start with such an idea.

What would be good solutions that I should look at? Any pointers would be appreciated! — QuantumEleven 15:24, 20 May 2009 (UTC)[reply]

Wireless devices specifically intended for this do exist. A quick bit of googling found this, but I make no recommendations. I believe there are various such devices on offer, so you'll need to do your own research to find what is right for you. --Tango (talk) 15:58, 20 May 2009 (UTC)[reply]

Apple's Airport Express allows wireless streaming of iTunes music. You can download add ons that let the system stream other things - Airfoil I think is one of the products. I personally have an Airport Express and it works really well for linking your iTunes up to another set of speakers - and with my laptop on the network I can use that as 1 (admittedly giant) remote control for my stereo. I must warn, however, that user reviews on the product have been mixed with many reports of the product dieing after a year or two. ny156uk (talk) 16:27, 20 May 2009 (UTC)[reply]

I dont' know if this is what you have in mind, but if you would be happy playing it through your television/whatever speakers are hooked up to your television, I bought a WDTV this weekend and have been very happy with it. You plug in any kind of USB drive and it'll play it on your television over composite or HDTV cables. It's primarily for playing computer videos on your tv, but it will play mp3s (and pictures) as well. The only problem (for music) is it doesn't seem very good at splitting up by album: you have to go by artist or folder, and then it gives the files in those folders by alphabetical order. I'm not sure if it can play playlists... Also it doesn't have a network card. You could conceivably hook up a networked drive to it (such as the My Book World Edition) but it probably makes more sense to just put all your music on a separate hard disk and attach it. I got one for $150 Canadian at Best Buy, on Amazon they are $105 US.TastyCakes (talk) 16:56, 20 May 2009 (UTC)[reply]

This inexpensive gadget is a tiny FM transmitter that plugs into the sound output of your PC. Then you can hear the sound on any FM radio within range. Cuddlyable3 (talk) 20:43, 20 May 2009 (UTC)[reply]

If you don't mind headphones you should get a pair of decent wireless headphones. I love my Sennheiser HDR-130, super comfy with good sound quality, but tend to fall down at the slightest tilt from my head (so far it has survived what, 50 1.8m drops?). --118.90.137.39 (talk) 04:58, 21 May 2009 (UTC)[reply]

My suggestion is a receiver with a multi-room output. This mean that you will plug 2 (or more, but 2 is common) pairs of speakers into the receiver, and then you can choose which of the pairs are on at any given time, regardless of the signal coming in (you can have none, either or both on at a time). I have this set up at my house: My iPod plugs into the receiver, and I can listen to it in my basement. If at any time I want to walk into my backyard and keep listening, I can hit the zone 2 button on my receiver and the outside speakers will turn on and mirror what's playing while the music continues in my basement. This also fixes your costly wiring problem, since you just need 2 speaker wires for each pair, and speaker wire is generally pretty cheap if you get it in bulk. I would stay away from many wireless devices, ESPECIALLY those fm radio transmitters, because the quality is horrible, which really really bothers me (but if you don't mind horrible sounding fuzzy music than that's the easiest solution). -Pete5x5 (talk) 16:07, 21 May 2009 (UTC)[reply]

  • I've been successful before in installing an audio driver that allows you to transmit audio output over Ethernet from one PC to another. I combined two programs, one that allows you to digitally redirect a virtual audio output back into a virtual audio input channel (you can also do this with a 1/8" male-male cable and your speaker output/mic in), and one that allows mic input to be broadcast over a LAN. Combine this with standard wireless Ethernet and cheap netbooks, and it's quite easy to get audio from one place to another. Dcoetzee

Human Skulls

Do people with different face shapes have different shaped skulls? Or is the difference made by skin, muscles and cartilage? —Preceding unsigned comment added by 82.44.54.169 (talk) 16:59, 20 May 2009 (UTC)[reply]

See Forensic facial reconstruction. The skull, otherwise they wouldn't have much hope doing it. Dmcq (talk) 17:18, 20 May 2009 (UTC)[reply]
Well, I think it is a combination of factors. I agree that the skull does play a significant role. I remember when I was reading about the first face transplant the doctors saying the recipient would end up with a face that is somewhere between their old face and the donor's face since they kept the same bone structure but got new skin, etc. --Tango (talk) 17:44, 20 May 2009 (UTC)[reply]
The bones make an enormous difference. The triangle formed by the eyes and mouth are essential in both natural and machine facial recognition, and that triangle is determined almost entirely by the skull. If the skull wasn't a big deal, you could put facial prosthetics on a person to make them look just like anyone larger than themselves, but that's just not the case -- Mission Impossible to the contrary. --Sean 17:49, 20 May 2009 (UTC)[reply]
Cartilage and soft tissue attaches to bone at some points and vessels feed through holes. There used to be a saying in paleontology that the fossil of an Elephant would be reconstructed to look like a Hamster because the soft tissue didn't leave a trace. This has been changed with the more recent developments in the field where similarities in fine bone structure of (recently dead) living creatures is compared to pits and holes and attachment ridges left in the fossilized bones. Bone is not a stable structure. It grows and cells get replaced. Soft tissue differences leave a trace on the bone. 71.236.24.129 (talk) 07:08, 21 May 2009 (UTC)[reply]
All the tissues impact the appearance of the face. Notice that if a person gains or loses a lot of weight, the appearance of their face changes substantially, without their bone structure changing - this shows that the face isn't determined entirely by bone structure. Dcoetzee 10:46, 21 May 2009 (UTC)[reply]

mt doom

I am drawing a satirical comic parodying Lord of the Rings. Basically Sam and Frodo get into a fight at the top over who has to buy the beer back at the Shire and Frodo gets pushed in. I need to know if it is likely that his hair would catch fire before he hits the lava, assuming hobitts have a similar terminal velocity and hair burning properties as humans would and that the fall distance to the lava is sufficient to achieve terminal velocity. 65.121.141.34 (talk) 20:17, 20 May 2009 (UTC)[reply]

Does it really matter? Dauto (talk) 20:47, 20 May 2009 (UTC)[reply]
Hair won't catch on fire because of the free fall itself. Terminal velocity for a flailing human is too slow - it won't generate enough friction, as evidenced by all the skydivers which reach terminal velocity without catching fire. In the absence of a spark, you need to get a substance to the autoignition temperature in order for it to catch fire. I couldn't find a figure for the autoignition temperature of hair, but for paper it's around 450 F (230 C - although some sources claim it's higher, around 840 F/450 C. It of course depends on the type of paper, etc). Lava can get up to 2,200 F (1,200 C), so it might be reasonable to claim that at some point above the lava the temperature reaches the 450-840 F range for autoignition of paper. If hair's autoignition temperature is similar to paper, hair should spontaneously ignite at that point. How long it'll be on fire will depend on how far away from the lava the temperature reaches the autoignition temperature. But a satirical comedy featuring hobbits in a magical universe probably can take some license with the facts ;-). -- 128.104.112.117 (talk) 20:50, 20 May 2009 (UTC)[reply]
Did you get the 450F from Fahrenheit 451? TastyCakes (talk) 20:54, 20 May 2009 (UTC)[reply]
I actually got it from averaging and rounding the 424-474 F range listed in the autoignition temperature. But I knew about Fahrenheit 451 - in fact footnote 1 in that article is where I got the "some sources claim it's 840 F" info. -- 128.104.112.117 (talk) 15:56, 21 May 2009 (UTC)[reply]

True, I don't think my crazed LOTR fan friend is too big into real science anyways. 65.121.141.34 (talk) 20:53, 20 May 2009 (UTC)[reply]

Things that one need to know about are where to find food, water and shelter. But flaming hairy hobbits? Cuddlyable3 (talk) 21:04, 20 May 2009 (UTC)[reply]

Have you read Bored of the Rings? -Arch dude (talk) 01:43, 21 May 2009 (UTC)[reply]

Professional sport gamblers

Is it possible to be good at sport gamble or is it just good luck? Since a player is competing against the mob and the mob is always right, is it possible?--88.6.117.202 (talk) 20:52, 20 May 2009 (UTC)[reply]

The "mob" isn't always right. The odds are based on what the mob thinks is likely to happen, if you know more than the mob then you can profit (consistently - anyone can profit inconsistently with just luck). You usually can't know more just by being clever, you need better sources of information. If you have contacts involved in the sport feeding you information (eg. "so-and-so's injury is worse than they're making out, he probably won't last the whole match") you can get an advantage. Personally, I prefer to gamble on things where I can increase my own chances using skill - eg. poker. --Tango (talk) 21:13, 20 May 2009 (UTC)[reply]
Umm, regarding the above tantalizing comment, and this little tidbit and reviewing Poker, Betting (poker) and Bluff (poker), I think there is an article opportunity here: Poker, how to play intelligently. Sorry about the short notice but I just saw your comment and thought that with all of the trivial articles we have around here that this one would stand as non-trivial, for sure. I think that you've got something to add but, that's just a guess. Ya think? -hydnjo (talk) 03:14, 21 May 2009 (UTC)[reply]
Wikipedia is not a how-to guide. There are plenty of websites offering tips on how to play poker, many of them are quite good (for a complete beginner wanting to get good enough to win money off their friends, you would need something a bit more than google to learn how to win major tournaments). --Tango (talk) 14:51, 21 May 2009 (UTC)[reply]
Like the stock-market, there are a few anomolies, but in reality it is unlikely to be worthwhile. See the efficient markets hypothesis. Sports gambling is a kind of market, and a lot of academic research by economists has been done. Anomolies include the favorite-longshot bias. It used to be possible to sometimes exploit the tote in the past, but the rules have been changed. You might in theory be able to develop a handicapping system that gave you a small advantage over chance. But, if you are smart enough to be able to do all this, then you would certainly be able to earn far more money doing something else. In reality, you will almost certainly lose money in the long run. 78.146.198.122 (talk) 21:17, 20 May 2009 (UTC)[reply]
Wouldn't consistently losing money be just as hard as consistently winning? — DanielLC 05:02, 21 May 2009 (UTC)[reply]
The house gets a cut, the vig. If someone bets randomly on sports, over time that will erode his capital down to nothing. Dragons flight (talk) 05:30, 21 May 2009 (UTC)[reply]
The trick is always to exploit inefficiencies in the market. In this case, the fact that not everyone has the same information. --Tango (talk) 14:51, 21 May 2009 (UTC)[reply]

You could search the bookies for Arbitrage (spelling) opportunities whereby you bet on all outcomes and are guaranteed a (usually small) win. E.g. A league football (soccer) game has 3 outcomes. Home win, draw, home loss (or away win if you prefer). If site X has odds of 4/1 for home win, site Y has odds of 4/1 for the draw and site Z has odds of 4/1 for home loss you can put money on all possible outcomes and be guaranteed a return. These sorts of situations will occasionally come up but given the difficulty and effort of finding (and taking advantage) of them they're not really viable as a 'pursuit'. Lots of betters will use statistical tactics to hedge bets and reduce down their odds of losing, whilst trying not to reduce the value if they do win. Professional Gamblers certainly do exist, I know one. I don't know what you mean by the mob but bookmakers certainly arrange their odds so that they win regardless of the event outcome. ny156uk (talk) 21:40, 20 May 2009 (UTC)[reply]

In fact we have an Arbitrage betting article, guess that's worth a read! ny156uk (talk) 21:41, 20 May 2009 (UTC)[reply]

Tango means that the odds you can get (or the line) depends on the alleged wisdom of crowds. If the Patriots are favored by 8 points over the Rams, and masses of bettors ("the mob") put money on the Patriots, then, as Ny156uk implies, the bookmakers will adjust the odds so that the Patriots are favored by 8 and one-half points, and then by 9, and so forth until the betting is roughly equal on both sides, so the bookmaker is finally sure to profit no matter who wins because he's only paying out $10 for every $11 that was bet (or so).
To answer the original anon's question, it's possible to win, yes, which can be illustrated easily if you consider illegally bribing players or referees, which certainly has happened many, many times. Unfortunately I don't know any references that discuss how many people are able to do this (more legally) for a living. Tempshill (talk) 22:35, 20 May 2009 (UTC)[reply]
Yes, one of the most effective ways of getting more information than the "mob" is to bribe people so you have a key piece of information others don't have - that so-and-so is going to throw the match. --Tango (talk) 14:51, 21 May 2009 (UTC)[reply]
When you are thinking of placing a bet of any kind, it is wise to consider the multi-billion dollar casino the "house" has managed to build - and how they raised the capital to build it. The stakes are always against you; the Bellagio didn't get built by having the odds in your favour, right? Matt Deres (talk) 01:46, 21 May 2009 (UTC)[reply]
For random games, like roulette, that is absolutely correct. There is no way to win at roulette other than getting lucky (or cheating!). The results of a horse race aren't random, though, which opens up possibilities. --Tango (talk) 14:51, 21 May 2009 (UTC)[reply]
You can win at horse racing if you base your bets on the speed of each horse rather than its form. AFAIK horse speeds are not published (presumably because that would spoil the fun), and you have to work them out for yourself. But this does work; success has everything to do with spending a few hours on your homework and very little to do with luck.--Shantavira|feed me 08:20, 21 May 2009 (UTC)[reply]
The speed of a horse is not constant, otherwise you would just have to go through results for the last few weeks and find out which horses in the race you are betting on have beaten which other horses in the race previously. You don't need much of an advantage to get profitable in horse racing, the house edge isn't particularly great. It doesn't matter if you lose the occasional bet, as long as you win more than your lose. --Tango (talk) 14:51, 21 May 2009 (UTC)[reply]
I sometimes get junk mail about horse racing betting systems - where for a large subscription you get sent betting tips which have been produced by some secret system. Is there an article about thse? I'm confident that such systems do not work, but I'm curious about what the secret system may be. Similarly with other secret money-making schemes that I also get sent junk mail about. 78.146.67.27 (talk) 12:16, 21 May 2009 (UTC)[reply]
The secret system will just be a load of nonsense, if you receive anything at all after giving them your money. It is obviously a scam because anyone knowing such a system could make far more money just using it themselves than they could make selling it. --Tango (talk) 14:51, 21 May 2009 (UTC)[reply]
One "secret system" (which a sometime-gambling friend once experienced) works as follows (numbers are for example only). The scammer identifies by adverts or otherwise 400 receptive gamblers (unknown to each other), and sends each a free "exclusive" prediction of the outcome of a race. If there are 4 horses in the race he'll send 100 a prediction of horse A winning, 100 horse B and so on. After the race, 100 will think he called it correctly. He then repeats the process with another race (perhaps for a small fee), then another (for a larger fee), until he's got about 6 marks who think he really knows his stuff. Now he offers to sell them the next "prediction" either for a fairly hefty sum, or on the condition that they put on a sizeable additional bet on his behalf, or both. Assuming they fall for it, he's assured of 6 hefty fees, and/or the proceeds of at least one large winning bet that he didn't have to stake. I'm glad to say that my friend figured out the scam before committing himself to the last step. 87.81.230.195 (talk) 23:58, 21 May 2009 (UTC)[reply]
See Derren Brown, The System. 89.168.85.22 (talk) 10:32, 22 May 2009 (UTC)[reply]
Ah! An interesting corroboration I was unaware of as I avoid most TV. FWIW, the experience I recounted occurred around 1980, but I'm sure the method (or System) has been around for a lot longer. 87.81.230.195 (talk) 11:42, 22 May 2009 (UTC)[reply]

Early primate Darwinius shown at keyboard?

Why does our supposed ancestor, "Ida" Darwinius,

look like she is sitting and working at a keyboard? Edison (talk) 21:17, 20 May 2009 (UTC) [reply]

This?Popcorn II (talk) 21:32, 20 May 2009 (UTC)[reply]

She seems to have better posture, and a shorter tail, in photo number 2. Edison (talk) 22:49, 20 May 2009 (UTC)[reply]
Ya, and at the moment, she is trying to google why she is suddenly so famous after 47 million years. More than that she is surprised at why her food is taking so long to digest. Unfortunately she cannot write anything in her own article at Wikipedia because of conflict of interest. But expect a question or two here at refdesk in few days maybe, because she will know many more startling things about herself in coming days and more temptation to come and ask here. - DSachan (talk) 23:10, 20 May 2009 (UTC)[reply]
Then does WP:BLP apply? We would not want to embarrass her. Computer geeks have feelings, too. Edison (talk) 04:57, 21 May 2009 (UTC)[reply]
She's waiting for 999 Shakespeare-loving friends to show up? Clarityfiend (talk) 05:34, 21 May 2009 (UTC)[reply]
On a more serious note: Rigor mortis may help explain some of the position of the limbs. The creature was using Arboreal locomotion. When in pain or near death many creatures curl up in a near fetal position. Humans tend to recognize known images in non related visual input (google "Face on Mars" "Devil in the smoke") Evolutionary this enabled us to recognize hidden predators. But sometimes a primate fossil is just a primate fossil.71.236.24.129 (talk) 06:57, 21 May 2009 (UTC)[reply]
We have an article for spurious pattern recognition by humans: pareidolia. Dcoetzee 09:31, 21 May 2009 (UTC)[reply]
They laughed when she sat down at the piano. Deor (talk) 11:56, 21 May 2009 (UTC)[reply]

May 21

Coraline and Real-D

Coraline was easily one of my favourite films of the past year. Even after seeing it in normal 2-D, it was well worth the price of seeing it again in Real-D (if only for the folded paper mice at the very, very end, after the theatre was empty!). Our article says that the DVD release will include both versions, but our article about Real D Cinema indicates that that process requires some pretty heavy duty specs to show in 3-D. Are those specs only applicable to projected films (i.e. a theatre) or will the normal DVD/Blu-Ray versions be the same quality as the film? Matt Deres (talk) 01:38, 21 May 2009 (UTC)[reply]

I don't see how they can show the 3D version on a standard or HD TV screen. In movie theatres, the 3D effect is produced by displaying the left eye and right eye images with different planes of polarization - and the 3D glasses filter out the undesired image for each eye. But a TV screen can't do that. There are 3D technologies that do work with TV - for example by drawing the left-eye image in red and the right-eye image in cyan and using glasses with colored lenses - but that produces nasty flicker and weird color fringing. Alternatively, they can display left and right eye images in alternate frames and use rather expensive 'LCD shutter glasses' which alternately blank out the left and right eye images alternately - but those have to be synchronised to the video stream somehow - which entails special electronics in the video player (or in the cable between the video player and the TV) to send a signal to the glasses (typically via infra-red) to tell it which eye to blank out for the following 1/60th second (1/50th in the UK). Some company once did a similar thing with a little gadget with a rubber suction cup that you stick onto the bottom-left corner of your TV screen that picks up light from a flickering black or white square and relays it to the glasses. But the cost of shutter glasses is pretty steep - and the red/cyan glasses suck - so I can't see how they plan to do this for the DVD/BluRay market. SteveBaker (talk) 12:50, 21 May 2009 (UTC)[reply]
It will probably be red/cyan. Other films shot in the Real-d process have been released to Blu-ray in red/cyan. (example: My Bloody Valentine 3D)
Personally, I enjoy movies in red/cyan, but a lot of people have trouble converging the image, get headaches, etc. APL (talk) 16:45, 21 May 2009 (UTC)[reply]
It seems as though doing polarization-based 3D on an LCD screen would be pretty easy. LCDs require an outer polarizing layer anyway, and if you replaced it with alternating horizontal and vertical polarizers then ordinary viewing would be unaffected but H-V polarized sunglasses would send each pixel to only one eye. If they'd shipped LCDs like that from the beginning then there'd be a huge installed base of 3D ready screens now. Seems like a sad missed opportunity. -- BenRG (talk) 20:40, 21 May 2009 (UTC)[reply]

Velocity profile of typical 12V DC electric motor

Hi, I'm wondering what the velocity profile is of a typical 12V electric motor? I've noticed (just empirically) that relatively low speed electric motors get up to speed pretty much straight away but I'd like a slightly more quantified estimate if possible. I think this question is best expressed in an example. So lets say I buy this motor and use it to run a little car. Approx how long would it take to get to its maximum speed of ~500 RPM? For arguments sake the load is approx 2kg.

Also on a related note, how do you convert the torque specification of 5 kg-cm to Nm?

Thanks --118.139.3.77 (talk) 03:02, 21 May 2009 (UTC)[reply]

Consider the moment of angular momentum versus the torque. This should answer your question. If a motor were trying to spin up a rotor of large moment, it would take longer than if the motor were spinning free of any load. More torque would get the system up to a given rotational velocity quicker. Edison (talk) 04:55, 21 May 2009 (UTC)[reply]
Would the load make that much difference? Wouldn't the load just make it run slower? At any rate I'm not sure on how you relate the moment of angular momentum to the torque of the motor to work out the time it would take to get to the maximum speed. Because say the output shaft of the motor was attached to a solid cylinder with a moment of inertia of 1 kgm^2, wouldn't that give a constant angular acceleration: T_motor = I_cyl*alpha_cyl => alpha_cyl = T_motor/I_cyl. And since the alpha is constant wouldn't that mean it would keep on spinning at ever increasing angular velocity? When surely it has to peak out at 500RPM? And it wouldn't really make sense to say it accelerates according to alpha_cyl until it reaches 500RPM and then suddenly stops because then what is balancing out the torque of the motor? Thanks, --118.138.152.143 (talk) 07:27, 21 May 2009 (UTC)[reply]
The motor isn't lifting the weight of the vehicle vertically. Assuming you're on level ground: You need to know the frictional forces (both static and dynamic) and the air resistance (if it's significant at the speeds you expect to travel). Once you know how much force the motor has to apply to overcome those things - you can take the torque and the wheel diameter and figure out how much force there is left over to accelerate the vehicle. You know the mass of the vehicle and the force available from the motor - so you can calculate that acceleration. Then you can turn linear acceleration back into rotational acceleration (again, knowing the wheel diameter) and you'll know the rate of rpm increase. SteveBaker (talk) 12:38, 21 May 2009 (UTC)[reply]

Was the ice age cold enough?

According to Quaternary_glaciation, The earth was 8 degrees (C) cooler at the peak of the last ice age. This sent glaciers all the way down into in Upper Midwest. But such a temperature change doesn't seem to me to be enough to cause such glacial expansion. Consider Minneapolis, for instance, whose geology is influenced by past glaciation. According to Minneapolis#Geography_and_climate the average high in July is 29 C. If it were 8 degrees cooler it would still be a warm 21 C, more than enough for the snow to melt through the year. The average year-round temperature would fall to -1 C. Seems cold enough, but Fairbanks, Alaska is colder at -3C [36] and is not currently covered by glaciers. What am I missing?

140.247.125.9 (talk) 04:27, 21 May 2009 (UTC)[reply]

Who in Minneapolis talks about temperature in centigrade/Celsius? Edison (talk) 05:44, 21 May 2009 (UTC)[reply]
nobody, but Wikipedia's graphs are all in celsius. 140.247.125.15 (talk) 17:06, 21 May 2009 (UTC)[reply]
The temperature changes more at the poles than at the equator so currently the poles are melting whereas the temperature has only changed a little at mid to equitorial latitudes. In an ice age the poles get much colder and the effect spreads down but the equator isn't all that much colder. The big expanse of ice causes a feedback effect cooling the area that is iced up. Dmcq (talk) 07:50, 21 May 2009 (UTC)[reply]
(edit conflict) Temperature change is not uniform. The number you reference, presumably from the plot, was for Central Antarctica. The global average change during the ice age was ~3 C, with the tropic changing much less than the poles. Central Greenland, by way of contrast saw a temperature change of ~30 C. Ice sheets grow by starting at places that are very cold and spreading outward. As they spread, the effect of covering more areas with ice and snow for more of the year often causes further cooling by reflecting sunlight into space (an albedo feedback). In addition, ice sheet growth also requires an available supply of moisture to allow for regular snowfall. Fairbanks, to use your example, is far enough North that the prevailing winds are polar easterlies. This means most of its weather comes from dry Canadian air rather than from the Pacific. As a result it is harder to form glacier there than on the East coast of North America at similar latitude, where one will get snow from Atlantic moisture. Hence ice sheet in North America start in the far northeast and grow out and down as temperatures fall. Dragons flight (talk) 07:52, 21 May 2009 (UTC)[reply]

When birds sleep...

Why do they usually turn their heads through 180 and tuck their beaks into their back feathers? Is it just to keep their faces warm, or is there a physiological reason for it? —Preceding unsigned comment added by 81.77.140.54 (talk) 08:42, 21 May 2009 (UTC)[reply]

Perhaps, it acts as a natural and ready to use pillow for them, which provides comfort (in the sense of softness) and warmth both. - DSachan (talk) 09:23, 21 May 2009 (UTC)[reply]
In Britain, for example, during the winter a large proportion of birds are killed during the nights from the cold, particularly when they are short of food and hence the energy to keep them warm. Anything that reduces their surface area will help them survive. (I remember hearing or reading that the dawn chorus intensity is in proportion to the left-over energy budget from the night). Plus, I imagine that they save energy by being able to relax the neck muscles which are no longer working to hold up the head. And like humans, when they sleep their head would droop if not supported. 78.146.67.27 (talk) 10:53, 21 May 2009 (UTC)[reply]

I suspect that it helps keep the biting insects at bay. If you've ever seen a bird on the nest being swarmed by mosquitoes and black fly. You would understand. 67.193.179.241 (talk) 20:44, 21 May 2009 (UTC) Rana sylvatica[reply]

On the topic of thunderstorms and humans

I am personally very fond of thunderstorms. They bring about a certain "charged" smell in the air, a heated and moist atmosphere and indeed, the rumbling from afar is tad soothing. I know a dozen more of my friends share this with me, and while lightning striking nearby does affect us all in similar way, I was keen on asking a question more related to thunderstorms from afar: Shouldn't I feel less enthusiastic when I know a thunderstorm is about to come? Most animals seek shelter, I can't imagine too many of them enjoy the ordeal, but while some humans retain the fright of thunderstorms, many (most, according to my original research) don't. The precautionary signs come off in a comfortable way. What is to be attributed for my experiences? Simply that I've never experienced anything to teach me otherwise, or that a more built-in switch has been dulled over generations of having shelter? Thank you in advance. 90.149.144.55 (talk) 10:34, 21 May 2009 (UTC)[reply]

I'm just speculating here, but it seems to me that if you're a human-sized animal, you really don't worry about thunderstorms too much. The chance of encountering a lightning strike in our natural habitat (the savannahs) is very small, and there isn't much shelter to speak of around. It's probably even a good time to hunt for certain animals. Dcoetzee 10:39, 21 May 2009 (UTC)[reply]
Well, one of the cool (and really comparatively advanced) things about us humans is that we are capable of ignoring or overriding our instincts based on the circumstances and rational thinking. Animals aren't very good at that. I mean, when we get angry, horny or scared, we can still function properly -- we don't start fighting, screwing or running away at the drop of a hat as soon as the urge strikes us. (Well, some people do, but people with that poor impulse control are generally trouble, and in trouble.) I was a little scared of thunderstorms when I was a kid; these days, like you, I love 'em. A part of it is probably the fact that I know I'm safe, and while I haven't examined my feelings very closely, I wouldn't be surprised to find that I'm still a little scared somewhere deep inside, but knowing that I'm safe, I can enjoy the thrill -- not unlike going on a roller coaster ride at an amusement park. -- Captain Disdain (talk) 11:17, 21 May 2009 (UTC)[reply]

Captain, i respectfully disagree with "animals aren't very good at that". Any plant or animal can be aclimatized, habituated or get used to, any stimulus within thier physiological limits. Again, HUMANS ain't so special. 67.193.179.241 (talk) 20:51, 21 May 2009 (UTC)Rana sylvatica[reply]

Sure, animals can get used to all kinds of things, but there's a great conceptual -- and cognitive -- difference between becoming acclimatized to something and making an actual case-by-case decision to disobey the instinct. The latter tends to require abstract reasoning skills. -- Captain Disdain (talk) 23:32, 21 May 2009 (UTC)[reply]
I suspect (without evidence) that we're 'wired' to seek shelter in the case of storms - which means that something in our brains makes us feel uncomfortable out in bad weather and comfortable when we reach shelter. If our emotional systems "reward" us for achieving an instinctual goal, that would be no surprise. We feel similarly comfortable after eating, sex and other such activities. It seems reasonable then that when we hear a storm and are sheltered - that we feel good about that. SteveBaker (talk) 12:31, 21 May 2009 (UTC)[reply]
Maybe I'm not like other people, but personally I like being outside rather than in when a big storm is coming. Then again, I also want to go tornado chasing, so I think my fight-or-flight instinct is broken.-RunningOnBrains(talk page) 14:18, 21 May 2009 (UTC)[reply]
I just got myself thinking: maybe more of us are adrenaline junkies than we give ourselves credit for. Maybe that sense of impending danger is what gives you a pleasurable rush. -RunningOnBrains(talk page) 14:22, 21 May 2009 (UTC)[reply]
I think all the answers above are very good. I would only add that thunderstorms may engage our higher brain functions as well. We live in an age that allows us unprecedented communications abilities; we can vicariously experience almost anything we care to imagine, but there's something to be said for the first hand, totally immersive experience of being within a powerful thunderstorm. For an old atheist like myself, it's probably as close as I'll ever come to experiencing something of that kind of terrible power. The kind of awe that power can elicit is probably not too far from an old-time fire-n-brimstone religious experience. Matt Deres (talk) 13:38, 21 May 2009 (UTC)[reply]
The "charged" smell you sense is probably ozone. It is said that some people react positively to the ionised air before thunderstorms. Cuddlyable3 (talk) 14:04, 21 May 2009 (UTC)[reply]
The fear factor is partly situational, I think. I ordinarily don't have any fear of thunderstorms, but I've been caught in exposed positions a couple of times -- once on a mountain pass with lightning striking all around and numerous lightning-blasted trees beside the trail -- and it definitely got my adrenaline going. Regarding the smell, I think it comes more from rain on dry ground than from ozone. Looie496 (talk) 16:31, 21 May 2009 (UTC)[reply]
Rain on dry ground happens in any sudden downpour. If the smell is unique to thunderstorms then it is more likely from the ozone. That would require lightning to be nearby, though, you wouldn't get any ozone from a thunderstorm several miles away. --Tango (talk) 23:25, 21 May 2009 (UTC)[reply]

Avoiding sexual arousal

I'm not sure if this is science or not. I took a guess. Anyway, my friend has two problems. One, she gets aroused very easily by vibrating buses - something which she finds intensely uncomfortable, especially as she has to take a bus to university every day. Secondly, she has wet dreams, which again, she doesn't enjoy. Is there any way to control or prevent either of these? Vimescarrot (talk) 10:28, 21 May 2009 (UTC)[reply]

Some people just happen to be very easily sexually stimulated by certain stimuli. I'm not aware of any treatment for this, although she can talk to her doctor about it if she feels comfortable doing so (I do know that there are some drugs that decrease libido - this is generally listed as a negative side effect - but I don't know if she'd want it to be impeded all the time). Generally, we can't give medical advice here. The only suggestion I can think of is to bring along a book or a music player or something to help divert her attention, and if possible to seek a more comfortable alternative to the bus. Dcoetzee 10:35, 21 May 2009 (UTC)[reply]
Maybe she just needs more sex, duo or solo? Or does she have religious reasons for abstaining? If you are a male friend, are you sure this is genuine and isnt just a come-on? 78.146.67.27 (talk) 10:56, 21 May 2009 (UTC)[reply]
She has reasons for abstaining, but she won't tell me what they are. Vimescarrot (talk) 11:09, 21 May 2009 (UTC)[reply]
Generally, if someone is abstaining from sex for reasons they aren't willing or able to disclose and gets very disturbed by arousal or wet dreams, I would consider that a kind of a warning sign. I could speculate, but what with the information available to us being second-hand and especially that whole medical advice thing, I'm gonna pass. Instead, I would recommend that she talk to a good therapist. -- Captain Disdain (talk) 11:23, 21 May 2009 (UTC)[reply]


Why the hell wouldn't she enjoy sexual dreams?? As for the bus thing,, see PSAS —Preceding unsigned comment added by 82.44.54.169 (talk) 11:23, 21 May 2009 (UTC)[reply]

82.44 - Just because you enjoy your sexuality, doesn't mean everyone does. Vimescarrot (talk) 11:33, 21 May 2009 (UTC)[reply]
True enough, but when the body and mind are at odds, it's an indication that one side or the other needs to be brought into alignment. The one requires a therapist, the other requires a physician; neither would benefit particularly from strangers on the internet making guesses based on second-hand information. The anon's comment was a little insensitive, but speaks to the general truth that the mind is often confused while the body seldom is. Matt Deres (talk) 13:43, 21 May 2009 (UTC)[reply]

I have heard two advices given to celibate monks troubled by sexual desire: A) Imagine that everyone is your brother or sister, and B) Picture everyone without their skin. Cuddlyable3 (talk) 13:42, 21 May 2009 (UTC)[reply]

The OP's friend isn't being aroused by specific people, though. Avoiding being sexually stimulated by the bus could be as simple as sitting it a different position. Cross or uncross your legs, maybe. Being made uncomfortable by wet dreams isn't something we can help with, though, that needs a therapist. --Tango (talk) 14:29, 21 May 2009 (UTC)[reply]
Check to see if she's figiting with her legs; bouncing one leg up and down is a common indication that someone is sexually aroused. It's considered by some to be an almost unconscious form of masturbation. If she's moving her legs all the time then she's likely sexually aroused more often than just on her way to school, and it's not just the bus (although that would indicate that the bus does a better job at pleasing her than her own legs). -Pete5x5 (talk) 16:07, 21 May 2009 (UTC)[reply]
Restless leg syndrome, though an invented reason to sell a drug, is not an unconscious form of masturbation. Tempshill (talk) 16:08, 21 May 2009 (UTC)[reply]
Unconsciously figiting is not restless leg syndrome. --Tango (talk) 16:27, 21 May 2009 (UTC)[reply]

Check to see if she's figiting with her legs; bouncing one leg up and down is a common indication that someone is sexually aroused[citation needed]. It's considered by some to be an almost unconscious form of masturbation[citation needed].

Wikipedia articles are fine. Vimescarrot (talk)
Actually, I don't know why I responded to that. I don't need to know when she's sexually aroused (she tells me anyway), and we already know why. I was just after how to stop it. Thanks for trying, though. Vimescarrot (talk) 17:02, 21 May 2009 (UTC)[reply]
To me, this looks like a request for medical advice, right? Axl ¤ [Talk] 21:09, 21 May 2009 (UTC)[reply]
At least partially, yes, which is why people have been suggesting going to a therapist. --Tango (talk) 23:23, 21 May 2009 (UTC)[reply]

The rules of thumb I know are that if its got four petals, its a member of the mustard/cabbage family, and if its got five petals then its a member of the Mallow family. Are these two rules of thumb reliable, and are there any more? I am in the UK, but such rules of thumb might apply worldwide. 78.146.67.27 (talk) 11:01, 21 May 2009 (UTC)[reply]

No, they are not reliable in that form. For example, a flower with four petals may belong to a plant in Papaveraceae and not only in Brassicaceae (=Cruciferae), and a flower with five petals may belong to a plant in Rosaceae, Oxalidaceae, Violaceae, Geraniaceae, etc., and not only Malvaceae. --Dr Dima (talk) 19:58, 21 May 2009 (UTC)[reply]

Those are not reliable rules and to each rule there are exceptions, BUT if the flower consists of many "florets" it is generally in the composite family. If the stem of the plant is roughly squared, then it is usually in the mint family. Can't think of exceptions to either rule but learning to use botanical keys quickly, is learning to cheat. That is, if you recognize that a plant is in family "X" then you can go to the index and find where that family is in the key. Good luck and have fun. 67.193.179.241 (talk) 21:19, 21 May 2009 (UTC) Rana sylvatica.[reply]

Thanks. "The family Asteraceae or Compositae (known as the aster, daisy, or sunflower family) is the second largest family of flowering plants, in terms of number of species." Cleavers or Goose grass is one of the exceptions to the square-stem rule. 78.146.162.232 (talk) 11:30, 22 May 2009 (UTC)[reply]

Biology - water

In cold countries such as Alaska, the surface of the sea freezes first or becomes solid ice, while the bottom of the surface still remains in liquid state. Why? —Preceding unsigned comment added by 60.53.212.205 (talk) 11:12, 21 May 2009 (UTC)[reply]

Because ice floats. 78.146.67.27 (talk) 12:07, 21 May 2009 (UTC)Rkr1991 (talk) 13:04, 21 May 2009 (UTC)[reply]

Exactly. Ice has a density of only approximately 9/10th of water, and hence when water freezes, ice being lighter rises to the top. This is of vital importance to the survival of fish and other aquatic animals. If ice had frozen from the bottom, they would have been pushed to the top, an a more dangerous environment. Now since ice freezes from the top, they can swim underneath the ice sheet. Also, don't forget to sign your posts by typing 4 '~' signs at the end :) Rkr1991 (talk) 13:04, 21 May 2009 (UTC)[reply]

Also, the sheet of ice on the top tends to insulate the water underneath and prevents total freezing. If it froze from the bottom up you would end up with the whole thing frozen. This peculiar feature of water (most substances constantly contract as they cool, water has a little bump in the graph near its freezing point) is one of the things that makes it so good for life. They have been theories about life on the moons of the outer planets based on ammonia, but ammonia doesn't have this advantage (it also lacks certain other advantages). --Tango (talk) 14:33, 21 May 2009 (UTC)[reply]
A crucial point is that (for fresh water) the maximum density of water occurs at 3.98 °C (39.16 °F), that is, several degrees above freezing. This means that the deep water is never colder than that, because colder water will rise. The same thing holds for sea water, except that the temperatures are different. Looie496 (talk) 16:40, 21 May 2009 (UTC)[reply]
Is that point of maximum density universal, or does it depend on pressure? --Tango (talk) 16:43, 21 May 2009 (UTC)[reply]
Here is a graph of water density vs. temperature and pressure and based on the maximums, the point of greatest density does indeed depend on pressure. Regarding ice density, there are some other forms of ice made using varied temperatures and pressures which have higher densities than liquid water like Ice III (1.16g/cm3 (at 350 MPa)). I'm not sure if Ice III is more dense than water at 350 MPa though. Sifaka talk 00:35, 22 May 2009 (UTC)[reply]
No, the same thing does not hold for sea water. For sea water, due to its salt content, the maximum density of the liquid is at the freezing point. Dragons flight (talk) 17:31, 21 May 2009 (UTC)[reply]
Interesting. I didn't know that. I wonder what's the minimum amount of salt necessary in order to make the point of maximun density coincide with the freezing point. Dauto (talk) 23:01, 21 May 2009 (UTC)[reply]
If what you are saying is true D.F., I'm not sure that implies that saltwater ice is equally or more dense than the saltwater it's in. To rule out the brine effect (the tendency for ice to exclude solutes as it freezes), I'm making a solid icecube of 125mM NaCl and I'm going to see if it floats or sinks in the same liquid solution it is made from. I'll report the results once the cube freezes entirely solid. (Later Edit:) I went and spilled my liquid solution and was too lazy to make some more so I tested my salty ice cube in tap water and it still floated, just barely. Sifaka talk 00:35, 22 May 2009 (UTC)[reply]
The solid is still less dense than liquid. It is just that the point of maximum density of the liquid shifts to the freezing point of the liquid rather than several degrees above the freezing point as is seen for pure water. It is also worth noting that the solid-to-liquid density shift is very large in all cases. By contrast the temperature dependence of density within the liquid state is relatively quite small. Dragons flight (talk) 00:51, 22 May 2009 (UTC)[reply]
DF, are you saying tha the maximum density of saltwater is precisely at the freezing point, or just much nearer the freezing point than freshwater? Is there a graph for the salinity/density/freezing point relationship? I'm asking because of my fascination with the very weird properties of water in all its phases. Franamax (talk) 09:34, 22 May 2009 (UTC)[reply]
AFAIK Freezing also changes the concentration of salt because it gets squeezed into channels and oozes out. You then get water with less salt in it which again floats. Drift ice. 71.236.24.129 (talk) 09:49, 22 May 2009 (UTC)[reply]

I'm sure I'm not the first person to say the suggestibility article reads poorly and looks to be mostly about hypnotic suggestibility. (Or, maybe the editors are just very suggestible and have been told it's god like it is. :-) )

I was wondering how it was that some - especially children - are able to think that things they imagine are actually real. Is this self-suggestibility due to confabulation? Do their minds naturally envision that, "Since I thought/envisioned it, it must be true?" What about adults? I'm not talking necessarily about the criminal who - desipte all the evidence they did it - convinces him or herself they didn't, though that may be part of it.

I would even be tempted to try to edit the article to include mention of confabulation, but I wouldn't have real sources. But, there has to be more to suggestibility than hypnotic states, doesn't there? 209.244.30.221 (talk) 13:29, 21 May 2009 (UTC)[reply]

I'll give this a shot. First off, when you say above that editors have been "told it's god" - have you been staring at any rotating coins lately? ;)
The confabulation article seems to be about manifestations of neurological impairment. This is not the case with children, rather there is an ongoing process of distinguishing imagination from reality. Recall that for the first several months of a human life, the baby is not able to even recognize the difference between itself and its environment. For many years, one depends on one's parents to explain that your dreams were only that, that the scary movie you watched was all made up, and that it's OK to walk down the stairs into a dark basement. It takes a long time for a brain to settle down and get on with the boring process of plain old living a life. Even then, many people are still compelled by their imagination and they imagine crazy realities like light bulbs and microprocessors. That's just normal neural development,
Looking at the Suggestibility article, I'm struck by the section discussing the differences between suggestible, susceptible and gullible. Confabulation may play a part there too as a pathological manifestation, but yes, you would need to assemble some sources. If you do find them, please be bold and edit away! Franamax (talk) 08:54, 22 May 2009 (UTC)[reply]

Carbon Monoxide Poisoning

Hi, I've always been curious about some of the specific points relating to carbon monoxide poisoning and have been unable to find answers elsewhere, I have five questions, any help or answers would be greatly appreciated:

  1. Many of the long term effects of co poisoning(CoP) can take several weeks to show up, I've read that this is due to the long term effects being caused by CO debinding from hemoglobin, then becoming toxic. Is it true that this is the cause of longterm effects, and if so would being treated with hyperbaric oxygen after 48 hours, but prior to several weeks, have any benefit?(HBO has a large benefit if administered within 48 hours)
  2. CoP can have permenant/long term effects of cognition and memory formation; in the event that these effects occur, how often are they permenant?
  3. Related to the above, I've read that some cases of CoP present mild symptoms so that the victim is unaware that have been poisoned; supposing that such a case did cause cognitive/memory problems, would the victim be aware that they had such problems? In other words, would the person be able to tell that they something was wrong, or would they be oblivous to such things, perhaps thinking that they have become somewhat absented minded...in short, how obvious are the brain based effects of CoP?
  4. Again relating to the above; I've read that CoP causes demylination to occur in the white matter of the brain, and I've also read that their are medicines for other disorders that can reverse the effects of demylination; given that this medicine was developed for other disorders, would it be effective in reversing the effects of CoP also?
  5. My final question, again related to the above; in the event of long term cognitive impairments would a CT/MRI be able to detect such damage? Are there any means to estimate the ammount of damage, the possiblity of recovery, or the chance of further degradation? I've read that in the first few weeks after exposure that small pockets of atrophy can be detected in the brain, though I would imagine that in time these atrophied cells would be removed by the body; is this accurate?

Thank you for any help. P.S. I do realize that this question could be considered "medical" in nature, to avoid giving the wrong impression; I am asking out of curiousity, not because I think I may have CoP or any other such thing:) Phoenix1177 (talk) 14:43, 21 May 2009 (UTC)[reply]

I have adjusted the formatting of your question slightly, I hope you don't mind! --Tango (talk) 16:30, 21 May 2009 (UTC)[reply]

To answer question 1: Carbon monoxide washout follows a biphasic pattern due to myoglobin binding. From this article, half times while breathing air are 236 minutes and 302 minutes; breathing 100% oxygen they are 87 minutes and 160 minutes. The CO level will be back to baseline within a couple of days. Our article "Carbon monoxide poisoning" describes the mechanisms of toxicity. The main effect is binding to haemoglobin, which reduces the oxygen-carrying capacity of blood, leading to tissue hypoxia. The other mechanisms are (probably) less important. Release of CO bound to myoglobin does indeed slow the rate at which CO in the blood falls. However this is not enough to actually raise the blood CO level. I am unconvinced that hyperbaric oxygen has "a large benefit". This study did not demonstrate superiority over high flow oxygen (FiO2 100%) at normal barometric pressure. Axl ¤ [Talk] 21:42, 21 May 2009 (UTC)[reply]

Scanning western blots randomises results...

Is there an advantage to using built-for purpose gel/blot scanners from biotech companies over a regular scanner for the purpose of scanning in western blots stained with AEC stain?

We've been using an ordinary scanner, and then quantifying the bands but I notice that the same band from two different scans of the same blot divided by the first band in the blot (for normalisation purposes) can differ by as much as two-fold. It depends on the contrast settings used. Default settings don't always result in fair representations of the actual blot. How does one overcome this? ----Seans Potato Business 15:03, 21 May 2009 (UTC)[reply]

If you mean an ordinary scanner in the sense of an off-the-shelf document scanner, I think that's a remarkably bad idea. Their nonlinearities are hideous. Looie496 (talk) 16:47, 21 May 2009 (UTC)[reply]
You shouldn't need anything fancy to scan in your gels and quantitate the bands. The commercial scanners are usually bundled with software that facilitates the analysis process and they come with other bells and whistles that make it worthwhile for some people to buy them. You can test for nonlinearity by just scanning the same gel a few different times and checking to make sure that you're getting similar results. Granted, if you're doing ultrasensitive measurements, Looie has a good point.
However, I think you already have a clue about why you're getting disparate results -- it sounds to me that the software you're using to adjust contrast is actually changing the information content of the image instead of just representing it differently on your display. This can be a huge problem because you're basically saturating the darkest bands to be able to see the lightest ones. If your image software is doing this, you need to either perform the quantitation on the unmodified image (you may not be able to see the band very well but the computer will be able to count up the gray levels for each pixel without a problem), or get new software!
If you just want to contrast the blot for the purpose of presentation or publication, you can do so but you should be honest and report that the image was contrasted using such-and-such program. Image manipulation can be very misleading and is pretty rampant in the scientific literature! --- Medical geneticist (talk) 16:57, 21 May 2009 (UTC)[reply]

Energy from walking feet

I read somewhere that now-a-days, many scientists have developed methods to produce and use energy in many forms, from walking human steps! Can anyone tell me how is this done? many thanks. 59.103.63.74 (talk) 23:08, 21 May 2009 (UTC)[reply]

I'm not sure what you are talking about, but I know that some wrist-watches use energy from a person's movement to self wind. This is not a new technology. Dauto (talk) 23:17, 21 May 2009 (UTC)[reply]
There are two technologies you could be describing. There are some devices that are powered by being shaken as a person moves. I also recall a design for a floor that uses the vibrations caused by people walking over it to generate power (possibly the floor of a subway station...). Which one are you interested in? --Tango (talk) 23:21, 21 May 2009 (UTC)[reply]
I think he is thinking of piezoelectrics which generate a tiny amount of electricity when mechanically compressed. Placed in everyday areas subject to compression, such as the sole of your shoe, and rectified in the right way this can be used to do small amounts so work. The available energy is usually quite small though. See also: Energy harvesting. Dragons flight (talk) 00:58, 22 May 2009 (UTC)[reply]

There are also floors that generate energy from the friction of those walking around on it. http://www.groovygreen.com/groove/?p=1867 has more info on them. 194.221.133.226 (talk) 09:08, 22 May 2009 (UTC)[reply]

It's not friction, it's impact. That's the kind of thing I was trying to describe above. --Tango (talk) 11:08, 22 May 2009 (UTC)[reply]

May 22

does THC cause weed's smell?

I'm curious. If it doesn't, I imagine there's some future genetic engineering in the works that will be the delight of young people everywhere, and I imagine it could be very lucrative for street hustlers with biotech qualifications. John Riemann Soong (talk) 02:55, 22 May 2009 (UTC)[reply]

The large leaves have little THC content but still have the herb smell, so I'd say no, the smell is due to other chemical compounds. Of course, that's only what I've heard through my church group, I never touch the stuff. :) One of the attractions of cannabis is that it is a natural plant product that springs from the earth all by itself and delivers a gift of nature - I'm not so sure there is a big market for GMO's in that space. There are already lots of synthetic chemical products to whack your head with if you're so inclined. (I'm not) Franamax (talk) 08:19, 22 May 2009 (UTC)[reply]
If you're thinking of sniffing dogs, I would imagine expect they could be trained to smell THC if they're not already. In any case, removing every single compound likely to be detected by humans is an extremely difficult task. In other words, you could change the smell but it would likely still have a fairly unique smell. Indeed, I would expect the current smell is a combination of a large number of compounds not one single one although some may be more important then others. Most importantly perhaps, removing most of the compounds giving it a smell is likely to result I would expect in a product that doesn't 'taste' as good when the marijuana is smoked or whatever Nil Einne (talk) 08:52, 22 May 2009 (UTC)[reply]
As an analogous scenario, think about decaffeinated coffee. While some enthusiasts insist it's as tasty as regular coffee, I can tell the difference. However, removing the caffeine does not create a colorless, odorless liquid. Many other constituent chemicals contribute to the total flavor and aroma. Nimur (talk) 09:54, 22 May 2009 (UTC)[reply]

Hypermetropia

How to find power of corretive lens? why do we consider normal near point as object distance and the defective near point as image distance while calculating the power of convex corrective lens for Hypermetropic eye? —Preceding unsigned comment added by 61.2.178.70 (talk) 03:10, 22 May 2009 (UTC)[reply]

I assume you are asking this question with some knowledge of lenses and optics. Now consider a hypermetropic eye. The problem is that things too close are not focused on the retina. The objective is to give normal vision, where the least distance of distinct vision (D) is about 25cm. That is, if an object is placed in front of the eye as close as D, even then the eye must be able to see it. That is the limiting case, so if we satisfy for the limiting, we (theoretically at least) satisfy for all cases. So now the object is at D. But the eye can see only its own least distance of distinct vision (d) which is greater than D. Therefore, the image must be formed there. So now applying the lens formula, as you correctly suggested, we consider normal near point as object distance and the defective near point as image distance. Also don't forget to sign your posts by typing four '~'s at the end. Rkr1991 (talk) 05:07, 22 May 2009 (UTC)[reply]

Lilac bushes

I have a few lilac bushes in my front yard. I'd like to have more. Is there an easy way for me to take a cutting from one of the bushes and get another bush out of it? Dismas|(talk) 05:15, 22 May 2009 (UTC)[reply]

I have no answer for you (other than talking about all my different-coloured lilac bushes) but I will say that if you're in the northern hemisphere, this is probably the time to clip off some green shoots and test directly. You could also try using a rooting hormone in the water, and while you're at the garden centre buying it, ask one of those helpful people. Franamax (talk) 09:42, 22 May 2009 (UTC)[reply]
Buddleia seeds can be easily bought, where I am at least, and grow into shrubs that look very similar to lilac bushes in my opinion. Perhaps you could harvest some lilac seeds from your bushes and grow those in pots of compost before planted them out. Other shrubs with lots of blossom also exist - I sometimes see for example a shrub that is completely covered in blue, no idea what its name is. 78.146.162.232 (talk) 11:50, 22 May 2009 (UTC)[reply]

BTU'S produced by a conventional 4 or 6 cylinder engine

Can anybody advise how much heat is produced on the exhaust system of a conventioal 4 -6 cylinder engine at the exhaust pipe near the catalytic convertor? —Preceding unsigned comment added by Sustain6996 (talkcontribs) 07:36, 22 May 2009 (UTC)[reply]

List of longest rivers in Poland

I have only found top 29 longest rivers in Poland. Do you know top 50 longest rivers in Poland? —Preceding unsigned comment added by 213.158.197.100 (talk) 11:44, 22 May 2009 (UTC)[reply]

  1. ^ Epstein, L.C. Thinking Physics. San Francisco: Insight Press. ISBN 0-935218-06-8